Ευκλειδης Β 62

84
, ΟΚΤΟΒΡΙΟΣ·ΝΟΕΜΒΡΙΟΣ·ΩΕΚΕΜΒΡΙΟΣ 2006 EYPD 3,5 Γενική ΚΑΤΑΣΤΑΤΙ Συνέλευση κ , 28 I , 2007 Μοναδικό Θέμα υρtα Κη αVOυαptOV ΤΡΟΠΟΠΟΙΗΣΗ ΚΑΤΑΣΤΑτΙΚΟΥ n - . Γ Γενική Απολογιστική l Συνέλευση 2005-2007 ' Κυριακή 11 Μαρτίου 2007 ΕΟΓΕΣ Κυριακή 18 Μαρτίου 2007 -- -=� Ελληνική Μαθηματική Εταιρεία

description

 

Transcript of Ευκλειδης Β 62

Page 1: Ευκλειδης Β 62

,

ΟΚΤΟΒΡΙΟΣ·ΝΟΕΜΒΡΙΟΣ·ΩΕΚΕΜΒΡΙΟΣ 2006 EYPD 3,5

Γενική ΚΑΤΑΣΤΑΤΙΚΗ Συνέλευση κ , 28 I , 2007 Μοναδικό Θέμα υρtαΚη αVOυαptOV ΤΡΟΠΟΠΟΙΗΣΗ ΚΑΤΑΣΤΑτΙΚΟΥ

n - � . Γ Γενική Απολογιστική

l Συνέλευση 2005-2007

' Κυριακή 11 Μαρτίου 2007

ΕΚΛΟΓΕΣ

Κυριακή 18 Μαρτίου 2007 � �-----==�

Ελληνική Μαθηματική Εταιρεία

Page 2: Ευκλειδης Β 62

•• Άλγεβρα

Α' Ενιαίου Λυκείου

Θ. Τσιούμας, I. Σιάχος

Μεθοδολογία Άλγεβρας Β' Ενιαίου Λυκείου

Γ ενικής Παιδείας Ε. Πρωτοπαπάς

1111 I � 111111 11111 I 11 I I� IIIH8 111111 11111 I 11 I I I ! / I �--· r i f I f I i I i ' f ' ' I ' !_,_ι I I i ' i / ' I ! 1) [j l ) I ; �-- · - I ' _} .. ) ..... .--!' '_; ---· ' .>

ΕΚΠΑΙΔΕΠΙΚΑ ΒΙΒΛΙΑ yιa μαβπτίς και εκπαιδευτικσιίς

Μεθοδολογία Άλγεβρας Α' Ενιαίου Λυκείου

Μ. Ευσταθίου, Ε. Πρωτοπαπός

μαθηματικά r· Eνraiou Λιικε!οu

Μαθηματικά Γ' Ενιαίου Λυκείου

Γενικής Παιδείας Μ. Τσιλπιρίδης

Γεωμετρία Α' Ενια ίου Λυκείου

Γεωμετρία Β' Ενιαίου Λυκείου

Γ ενικης Παιδείας Ε. Πρωτοπαπάς

Μαθηματικά Γ Ενιαίου Λυκείου

Αρχές οικονομικής θεωρίας Γ Ενιαίου Λυκείου

Μάθημα επιλογής Θετικής και Τεχνολογικής κατεύθυνσης α · τόμος: Ε. Πρωτοπαπάς για όλες τις κατευθύνσεις

Π. Μηλίτσης β' τόμος: Ε. Πρωτοπαπάς, Σ. Γκούμaς

ΕΚΔΟΕΕΙΣ ΠΑΤΑΚΗ

σ ε: ό r't a τ a e. ιe. rt ι o n ω r't ε:ia ΒΙΒΛΙΟΠΩΛΕΙΟ ΠΑΤΑΚΗ: ΑΚΑΔΗΜΙΑΣ 65, 106 78 ΑΘΗΝΑ. ΤΗΛ. 210.38.11.850

KENTPIKH ΔΙΑΘΕΣΗ: ΕΜΜ. ΜΠΕΝΑΚΗ 16, 106 78 ΑΘΗΝΑ. ΤΗΛ. 210.38.31.078

ΥΠΟΚ!ΜΑ: Ν. MONAHHPIOY 122. 563 34 ΘΕΣΣΑΛΟΝΙΚΗ. ΤΗΛ. 2310.70.63.54

Web site: http:/fwww.patakis.gr · e-mail: [email protected]. [email protected] nρωιonopiu οιψ• ι-σιιιίδει•οιι

Page 3: Ευκλειδης Β 62

ΕΛΛΗΝΙΚΗ ΜΑΘΗΜΑΤΙΚΗ ΕΤΑΙΡΕΙΑ ----------------------ΠΡΟΣΚΛΗΣΗ

Καλούνται όλα τα τακτικά μέλη της Ε.Μ.Ε. σε ΚΑΤΑΣΤΑΤΙΚΗ Γενική Συνέλευση

την Κυριακή 28 Ιανουαρίου 2007

Αθήνα, 23 - 1 1 - 2006

ώρα 10.00 το πρωί, στο Μεγάλο Αμφιθέατρο στο Κτίριο του Νέου Χημείου του Πανεπιστημίου Αθηνών (Ναυαρίνου 13α, Αθήνα).

Με μοναδικό Θέμα: Τροποποίηση Καταστατικού Για να υπάρξει απαρτία σύμφωνα με το καταστατικό πρέπει να παρίσταται τουλάχιστον

το 1/2 των τακτικών μελών της Ε.Μ.Ε., που έχουν εκπληρώσει τις ταμειακές τους υποχρεώσεις για το 2006 .

Με συναδελφικούς χαιρετισμούς

Ο Πρόεδρος Θεόδωρος Εξαρχάκος

Αγαπητοί φίλοι Συνάδελφοι,

Για το Δ.Σ.

Σε όλους εσάς και στις οικογένειές σας ευχόμαστε:

Ο Γενικός Γραμματέας Ιωάννης Τυρλής

«Καnά ΧριστούγΕννα, ΕυτυχισμιΞνο και Δnμιουpγικό το 2007>> Αγαπητοί Συνάδελφοι, Στη Γενική συνέλευση της 27ης Φεβρουαρίου 2005, όπως γνωρίζετε, αποφασίστηκε

ομόφωνα η συγκρότηση οκταμελούς επιτροπής για να προτείνει τροποποιήσεις του ισχύοντος από το 1 977 καταστατικού.

Η οκταμελής επιτροπή , ύστερα από αρκετές συνεδριάσεις της, αποφάσισε να θέσει υπόψη του Διοικητικού Συμβουλίου την «Ομόφωνη απόφασή της για την αλλαγή του εκλογικού συστήματος, σε ένα ψηφοδέλτιο που επιτρέπει την αναλογική εκπροσώπηση των κινήσεων, καθώς και των μεμονωμένων συναδέλφων, που δραστηριοποιούνται στην Ελληνική Μαθηματική Εταιρεία .

Το Διοικητικό Συμβούλιο της Ελληνικής Μαθηματικής Εταιρείας αποφάσισε ομόφωνα να αποδεχτεί την πρόταση της Επιτροπής όπως αυτή διαμορφώθηκε ύστερα από νομοτεχνική επεξεργασία και να θέση υπόψη σας στην Καταστατική Συνέλευση που ορίστηκε για τις 28 Ιανουαρίου 2007.

Επειδή το ισχύον καταστατικό είναι αρκετά παλαιό και δεν ανταποκρίνονται στις σύγχρονες απαιτήσεις και ανάγκες της Ελληνικής Μαθηματικής Κοινότητας, καλούνται τα τακτικά μέλη της Ελληνικής Μαθηματικής Εταιρείας να παρεβρεθούν στην Γενική Καταστατική Συνέλευση για να εγκρίνουν με την ψήφο τους ένα σύγχρονο, λειτουργικό και αντιπροσωπευτικό εκλογικό σύστημα.

Το πλήρες κείμενο της πρότασης τροποποίησης του καταστατικού βρίσκεται αναρτημένο στο κόμβο της Ε.Μ.Ε. στη διεύθυνση:

http://www .hms .gr/gr/eme/index pin3 .html .

Και πάλι θερμές ευχές και Ραντεβού στις 28 Ιανουαρίου 2007 στο Νέο Χημείο.

ΕΥΚΛΕΙΔΗΣ Β ' τ.2/1

Page 4: Ευκλειδης Β 62

--------------- Καταστατική Γενική Συνέλευση ---------------

�-11'®��® �W�·��·

Στη συνεδρίαση του Δ.Σ. της Ε.Μ.Ε. την 1 η Φεβρουαρίου 2006 και σύμφωνα με την ημερήσια διάταξη, συζητήθηκε η ομόφωνη πρόταση που κατέθεσε η 8μελής Επιτροπή που είχε οριστεί με ομόφωνη απόφαση της Γενικής Συνέλευσης της 27ης Φεβρουαρίου 2005.

Το Δ.Σ. σε συνεδρίαση του αποφάσισε ομόφωνα να αποδεχθεί την πρόταση, όπως αυτή διαμορφώθηκε μετά από νομοτεχνική επεξεργασία και να την θέσει υπόψη της Καταστατικής Γενικής Συνέλευσης που ορίστηκε για την 12η Φεβρουαρίου 2006 στην οποία όμως δεν επιτεύχθηκε η προβλεπόμενη αυξημένη απαρτία.

Το Δ.Σ. στη συνεδρίαση της 10ης Νοεμβρίου 2006 αποφάσισε ομόφωνα να καλέσει εκ νέου Καταστατική Γενική Συνέλευση στις 28 Ιανουαρίου 2007 και να θέσει προς συζήτηση τη πρόταση.

Το πρωτοποριακό εκλογικό σύστημα που προτείνεται είναι μία σύζευξη του ενιαίου ψηφοδελτίου και της απλής αναλογικής. Στην πρόταση περιλαμβάνονται τα εξής: Α) Συγκρότηση ενιαίου ψηφοδελτίου. Ο κάθε υποψήφιος δηλώνει μία από τις τρεις παρακάτω δυνατότητες:

α) Συμμετέχω στο συνδυασμό Α ή Β ή Γ κλπ. και αποδέχομαι την προσμέτρηση των σταυρών προτίμησης που θα λάβω με τους σταυρούς των υπολοίπων μέχρι και δεκατεσσάρων υποψηφίων του ίδιου συνδυασμού.

β) Δεν συμμετέχω σε κανένα συνδυασμό, είμαι μεμονωμένος υποψήφιος και αποδέχομαι την προσμέτρηση των σταυρών προτίμησης που θα λάβω στους σταυρούς των 15 πρώτων σε σταυρούς μεμονωμένων υποψηφίων, εφόσον υπάρχουν, προκειμένου να συμμετέχουν στη Β' κατανομή των εδρών.

γ) Δεν συμμετέχω σε κανένα συνδυασμό, είμαι μεμονωμένος υποψήφιος, η υποψηφιότητά μου αποτελεί αυτοτελή συνδυασμό και δεν αποδέχομαι την προσμέτρηση των σταυρών που θα λάβω με τους σταυρούς άλλων μεμονωμένων υποψηφίων ή υποψηφίων συνδυασμών προκειμένου να συμμετέχω στη Β' κατανομή των εδρών.

Στα πλαίσια του ενιαίου ψηφοδελτίου είναι δυνατόν να δημιουργηθούν συνδυασμοί, ο καθένας από τους οποίους περιλαμβάνει από ένα έως δεκαπέντε υποψηφίους.

Β) Κατανομή εδρών:

α) Η Εφορευτική Επιτροπή διεξαγωγής των Αρχαιρεσιών εκλέγεται και λειτουργεί όπως προβλέπεται από το Καταστατικό. Μετά την ολοκλήρωση της ψηφοφορίας αθροίζει όλους τους σταυρούς όλων των υποψηφίων, το δε άθροισμα το διαιρεί με το 15, οπότε το πηλίκο που προκύπτει αποτελεί το εκλογικό μέτρο.

β) Για την Λ κατανομή των εδρών προσθέτει τους σταυρούς των υποψηφίων κάθε συνδυασμού και το άθροισμα το διαιρεί με το εκλογικό μέτρο, οπότε ο κάθε συνδυασμός εκλέγει τόσους υποψηφίους όσους δηλώνει το ακέραιο μέρος του πηλίκου της αντίστοιχης διαίρεσης, σύμφωνα με τη σειρά σταυροδοσίας τους. Έτσι ολοκληρώνεται η διάθεση των εδρών στην Α κατανομή, εφόσον δεν υπάρχει μεμονωμένος υποψήφιος της (lγ) περίπτωσης ο οποίος να έχει λάβει αριθμό σταυρών τουλάχιστον ίσον με το εκλογικό μέτρο.

γ) Στη Β κατανομή συμμετέχουν όλοι οι συνδυασμοί ανεξάρτητα από την εκλογή ή μη υποψηφίου στην Α κατανομή. Επιπλέον συμμετέχει και η ομάδα των 15 πρώτων σε σταυρούς μεμονωμένων υποψηφίων (σαν παραγόμενος συνδυασμός), οι οποίοι εξ' αρχής έχουν αποδεχθεί την προσμέτρηση των σταυρών που θα λάβουν, για να συμμετέχουν στη Β κατανομή των εδρών. Αν μετά τη Β κατανομή παραμείνουν αδιάθετες έδρες, τότε αυτές κατανέμονται σύμφωνα με τα υπόλοιπα των συνδυασμών (στους οποίους συμπεριλαμβάνεται και ο παραγόμενος συνδυασμός).

Ακριβώς αντίστοιχη διαδικασία με την παραπάνω εφαρμόζεται για την εκλογή των μελών της Ε.Ε., (Για την Ε.Ε. εκλέγονται 3 υποψήφιοι). Ώρα λήξης της ψη φοφορίας: 19.00'

Η παραλαβή των φακέλων από το ταχυδρομείο θα γίνεται, στις 13.00' την πρώτη εργάσιμη μέρα μετά τις εκλογές. Το πλΙ1ρες κείμενο της πρότασης τροποποίησης του καταστατικού βρίσκεται αναρτημένο στο κόμβο της

Ε.Μ.Ε. στη διεύθυνση: http://www.hms.gr/gr/eme/index pin3.html.

Η προθεσμία υποβολής υποψηφιοτήτων για το Δ.Σ. και την Ε.Ε.

είναι 15 Ιανουαρίου 2007 Απολογιστική Γενική Συνέλευση 2005-2007

Κυριακή 11 Μαρτίου 2007 Οι εκλογές θα πραγματοποιηθούν

18 Μαρτίου 2007 ημέρα Κυριακή

Ο Πρόεδρος Καθηγητής Θεόδωρος Εξαρχάκος

ΓΙΑ ΤΟ Δ.Σ. ΤΗΣ Ε.Μ.Ε.

ΕΥΚΛΕΙΔΗΣ Α ' τ.2/2

Ο Γενικός Γραμματέας Ιωάννης Τυρλής

Page 5: Ευκλειδης Β 62

ΕΛΛΗΝΙΚΗ ΜΑΘΗΜΑΤΙΚΗ ETAIPEIA Τεύχος 62 . Οκτώβριος · Νοέμβριος • Δεκέμβριος 2006· Έτοc; Αθ' • Ευρώ: 3,50

e-mail: [email protected] www.hms.gr

ΜΑθΗΜΑΤΙΚΟ ΠΕΡΙΟΔIΚΟ ΓΙΑ ΤΟ ΛΥΚΕΙΟ

ΠΕΡΙΕΧΟΜΕΝΑ .ι' Γράμμα από το Δ.Σ. της Ε.Μ.Ε. • Πρόσκληση yια ΚΑΤΑΣΤΑJΙΚΗ Συνέλευση .... . .. 1 ../ Ιστορικές Μαθηματικές Αναφορές • ΠροΕυκλείδεια Μαθηματικά .............. 4 ../ Μαθηματικοί Διαγωνισμοί- Μαθηματικές Ολυμπιάδες ...................... 9 ../ Το Βήμα του Ευκλείδη .................................. . ........... 20 ../ Homo Mathematicus ............................................. . ............ 24

Μαθηματικά Α' Τάξης ../ Άλyεβρα

../ Γεωμετρία

....................................................... 28 34

Μαθηματικά Β' Τάξης ../ Άλyεβρα ....................................................... 38 ../ Γεωμετρία ........................................................ 41 ../ Κατεύθυνση 45

Μαθηματικά Γ' Τάξης ../ Μαθηματικά Γενικής Παιδεία ......................................... 51 ../ Μαθηματικά Κατεύθυνσης ................. . ........................ 60

../ Το Sketchpad σχεδιάζει, μετράει και ... προκαλεί ......................... 72 ../ Τα Μαθηματικά μας Διασκεδάζουν ..................... . .............. 75 ../ Ο Ευκλείδης προτείνει ... Ευκλείδη . . . . . . . . . . . . . . . . . . . . . . . . . . . . . . . . . . . 77

•••• • ••••••••••••••••••••••••••••••••••••••••••••••••••••••••• • •••• • ••••••••••

ΕΚΔΟΣΗ ΤΗΣ ΜΑΘΗΜΑ1ΙΚΗΣ ΗΑΙΡΕΙΑΣ

ΠΑΝΕΠΙΠΗΜΙΟΥ 34 - 1 06 79 ΑΘΗΝΑ Τηλ.: 21 Ο 3617784 • 361 6532 Fax: 2 1 03641025 Εκδότης: Εξαρχάκος Θεόδωρος Διευθυντής: Τυρλής Ιωάννης

Κωδικός ΕΛ.ΤΑ.: 2055 ISSN: 1105- 7998 Επιμέλεια · Εκδοοης: Κυριακόπουλος Αντώνης Ευσταθίου Βαγγέλης

Εκτελεστική Γραμματεία

Πρόεδρος: Κυριακόπουλος Αντώνης

Αντιπρόεδροι: Α·: Ευσταθίου Βαγγέλης ι·: Τασσόπουλος Γιώργος Γ

ραμματέας: Χριστόπουλος Παναγιώτης

Μέλη: Αργυράκης Δ. Δρούτσας Π. Λουρίδας Σ. Ταπεινός Ν.

ΙΔιΟΚΤΗΣιΑ ΤΗΣ ΕΜΗΝιΚΗΣ ΜΑΘΗΜΑΥΙΚΗΣ ΕΥΑΙΡΕΙΑΣ

Η ύλη των τάξεων του τεύχους 62 έγινε με

συνεργασία των Παραρτημάτων της Ε.Μ.Ε. Κοζάνης, Γρεβενών, Ημαθίας

Συντακτική Επιτροπή

Αθανασόπουλος Γεώργιος Ανδρουλακάκης Νίκος Αντωνόπουλος Νίκος Αργυράκης Δημήτριος Βακαλόπουλος Κώστας Γράψας Κων/νος Δρούτσας Παναγιώτης Ευσταθίου Βαγγέλης Ζαχαρόπουλος Κων/νος Ζώτος Βαγγέλης Καλίκας Σταμάτης Κανέλλος Χρήστος Καραγκούνης Δημήτρης Καρακατσάνης Βασίλης Καρκάνης Βασίλης Κατσούλης Γιώργος Κερασαρίδης Γιάννης Καρδαμίτησ Σπύρος Κηπουρός Χρήστος Κλάδη Κατερίνα Κόντζιας Νίκος Κοτσιφάκης Γιώργος Κυριακόπουλος Αντώνης Κυριακόπουλος Θανάσης Κυβερνήτου Χρυστ .

Λαζαρίδης Χρήστος Λουρίδας Σωτήρης Μαλαφέκας Θανάσης Μενδρινός Γιάννης Μεταξάς Νικόλαος Μυλωνάς Δημήτρης Μώκος Χρήστος Πανουσάκης Νίκος Ρέγκλης Δημήτρης Σαtτη Εύα Στα'ίκος Κώστας Στά'ίκος Παναγιώτης Στρατής Γιάννης Ταπεινός Νικόλαος Τασσόπουλος Γιώργος τζιώρζιος Θανάσης Τριάντος Γεώργιος Τσαγκάρης Ανδρέας Τσατούρας Ευάγγελος Τσικαλουδάκης Γιώργος Τσιούμας Θανάσης Τυρλής Ιωάννης; Χαραλαμποπούλου Λίνα Χαραλάμπους Θάνος Χριστόπουλος Παναγιώτης

• • • • • • • • • • • • • • • • • • • • • • • • • • • • • • • • • • • • • • • • • • • • • • • • • • • • • • • • • • • • • • • • • • • • • • • • • • • • • • • Τα διαφημιζόμενα βιβλία δε σημαίνει ότι πρστείνοvtαι από την Ε.Μ.Ε. • Οι συνεργάτες, τα άρθρα, οι πρστεινόμενες ασκήσεις, οι λύσεις ασκήσεων κτλ. πρέπει να στέλνοvtαι έγκαιρα, στα γραφεία της

Ε.Μ.Ε. με την ένδειξη "Για τον Ευκλείδη β'". Τα χειρόγραφα δεν επιστρέφονrαι. Τιμή Τεύχους ευρώ 3,50 Ετήσια συνδρομή (10,00 + 4,00 Ταχυδρομικά= ευρώ 14,00) Εηjοια ουνδρομή για Σχολεία ευριδ 10,00 Το αντίτιμο για τα τεύχη που παραγγέλνοvtαι στέλνεται με απλή επιταγή σε διαταγή Ε.Μ.Ε. Ταχ. Γραφείο Αθήνα 54 Τ. Θ. 30044 ή πληρώνεται στα γραφεία της Ε.Μ.Ε.

Εκτύπωση: ΙΝΤΕΡΠΡΕΣ Α.Ε. τηλ.: 210 8160330 Υπειίθυνος τuπογpαφείοu: Β. Σωτηριάδης

Page 6: Ευκλειδης Β 62

Επιμέλεια: Χρήστος Κηπουρός

Τα Προ - Ευκλείδεια Μαθηματικά

Δημόκριτος και οι Σοφιστές ήταν οι πρώτοι που ασχολήθη­καν με τη γεωμετρία των aπειροστών μεγεθών, εγκαινιάζο­ντας έτσι ένα κλάδο των μαθηματικών εντελώς πρωτοπορια­

κό για τον 5° π.Χ. αιώνα. Το δυστύχημα ήταν ότι μ· αυτό το είδος των •

μαθηματικών διαφωνούσε ο Πλάτων, με αποτέλεσμα οι αληθινά πρω­τοποριακές αυτές θεωρίες να μείνουν για καιρό στο περιθώριο και να μη βρουν άξιους συνεχιστές.

Στέλιος Λαμνής

Ευτυχώς για τα μαθηματικά, που οι πλατωνικές ιδεολογικές προκαταλή­ψεις δεν υιοθετήθηκαν από τον Εύδοξο τον Κνίδιο και τον Αρχιμήδη. Οι δύο κορυφαίοι Έλληνες μαθηματικοί της αρχαιότητας είχαν την τόλμη να αποδεχτούν τις πρωτοποριακές ιδέες του Δημόκριτου και των Σοφιστών, αλ­λά και την ευφυΊα να τις εμπλουτίσουν και να τις εξελίξουν. Στηριζόμενος άλλωστε πάνω σ· αυτές ακριβώς τις ιδέες, λίγες δεκαετίες αργότερα ο μεγά­λος Αρχιμήδης κατανόησε πρώτος την έννοια του «απειροστού μεγέθους» και έφτασε να βήμα πριv από την επινόηση του Απειροστικού Λογισμού . Πως οδηγήθηκαν όμως ο Δημόκριτος και οι Σοφιστές σ' αυτές τις επινοήσεις; Τ ο πιθανότερο είναι ότι αυτό έγινε στην προσπάθειά τους να αντιμετωπίσουν τα τρία περίφημα κλασικά προβλήματα της αρχαίας ελληνικής γεωμετρίας. Ήδη στην εποχή του Δημόκριτου ήταν βάσιμες οι υποψίες των γεωμετρών ότι

Πλάτων τα τρία αυτά προβλήματα ήταν αδύνατο να λυθούν με τη χρήση μόνο ευθειών και κύκλων.

Έτσι λοιπόν, ορισμένοι από αυτούς, σκέφτηκαν να αναζητήσουν λύσεις που απαιτούσαν τη χρήση και άλλων καμπύλων, όπως είναι η «τετραγωνίζουσα του Ιππία του Ηλείου», με την οποία λύνονται τα προ­βλήματα της τριχοτόμησης της γωνίας και του τετραγωνισμού του κύκλου. Αργότερα επινοή­θηκε η «σπείρα ή έλιξ του Αρχιμήδη» για τον τετραγωνισμό του κύκλου, η «κογχοειδής» του Νικομήδη για την τριχοτόμηση της γωνίας και για τον τετραγωνισμό του κύκλου, αλλά και η «κισσοειδής» του Διοκλή για τον διπλασιασμό του κύβου. Πριν από την εποχή του Δημόκρι­του, ο Ιπποκράτης ο Χίος (450 π.Χ.) είχε επι­νοήσει την αναγωγή του προβλήματος του δι­πλασιασμού του κύβου στην ανεύρεση δύο μέ­σων αναλόγων χ και ψ ώστε να ισχύουν οι ανα­λογίες:

�=�=_y_· χ ψ 2α

Άν α είναι η πλευρά του δοσμένου κύβου και χ του ζητουμένου, θα πρέπει από τις παρα-

ΕΥΚΛΕΙΔΗΣ Β' τ.2/4

Page 7: Ευκλειδης Β 62

-------------- Ιστορικές Μαθηματικές Αναφορές -------------­

πάνω αναλογίες να προκύπτει η εξίσωση χ3=2α3 . Από τις αναλογίες α χ ψ , ) 2 2 Μ

, - = - = - · παιρνουμε: χ- = α ψ και ψ = αχ. ε τετραγωνισμο της χ ψ 2α

πρώτης, έχουμε : χ4 = α2ψ2 .Αντικαθιστώντας σ ' αυτήν την ψ2 = 2αχ βρί­

σκουμε : χ4 = 2α3 χ<::::> χ3 = 2α3 . Η λύση του προβλήματος λοιπόν, ανάγεται στην κατασκευή δύο ευθυ­

γράμμων τμημάτων χ και ψ, μέσων αναλόγων των τμημάτων α και 2α. Η ιδέα του Ιπποκράτη του Χίου χρησίμευσε σαν βάση για μερικές πάρα

πολύ σημαντικές νέες μαθηματικές έννοιες, όπως για παράδειγμα οι κωνικές Αρχιμήδης τομές. Η επινόησή τους οφείλεται στον Μέναιχμο (4°ς αι. π.Χ.) , ο οποίος τις χρησιμοποίησε για να λύσει το πρόβλημα του διπλασιασμού του κύβου . Έ-δειξε ότι η κατασκευή των δύο μέσων αναλόγων επιτυγχάνεται με τον προσδιορισμό των σημείων τομής δύο κωνικών τομών, συγκεκριμένα μιας υπερβολής και μιας παραβολής. Αργότερα βέβαια τη θεωρία των κωνικών τομών ανέπτυξε ακόμη περισσότερο και ολοκλήρωσε ο κορυφαίος γεωμέτρης της αρχαιότητας Απολλώνιος ο Περγαίος (3°ς αι. π .Χ.) .

Ο Πλάτων ποτέ δεν είδε με καλό μάτι την ενασχόληση των Ελλήνων γεωμετρών με τα τρία παραπάνω προβλήματα. Α ναφέρει σχετικά ο Πλούταρχος ("Συμποσιακά Προβλήματα", 8° κεφάλαιο) : <<Ιlλάτων ε­μέμ ψατο τους περί Εύδοξον και Αρχύταν και Μέναιχμ ον εις οργανικάς και μηχανικάς κατασκευάς τον του στερεο ύ διπλα­σιασμ όν απάγει επιχειρ ο ύντας ... απόλλυσθαι γαρ ο ύτω και διαφ θείρεσθαι το γεωμετρίας αγαθόν αύ θις επί τα αισθητά παλινδρ ομ ο ύ σης και μη φερ ομένης άνω μηδ' αντιλαμβανομέ ­νης των αϊδίων και ασωμ άτων εικόνων, πρ ος αίσπερ ών ο Θεός Θεός έστιν». Δηλαδή , ο Πλάτων κατηγόρησε τον Εύδοξο, τον Αρχύτα και τον Μέναιχμο επειδή προσπάθησαν να λύσουν το πρόβλημα του διπλασιασμού του κύβου με μηχανικά εργαλεία και κατασκευές. Έ­τσι όμως, ισχυρίζεται ο Πλάτων, καταστρέφεται η γεωμετρία γιατί οπι-

Αρχύτας σθοχωρεί προς τα αισθητά και δεν προχωρά προς τα άνω, ούτε προσπα­θεί να έρθει σε επαφή με τις αιώνιες και aσώματες εικόνες, με τις οποίες

διαρκώς ασχολούμενος ο Θεός είναι Θεός. . Διαπιστώνουμε δηλαδή ότι οι παραινέσεις του Πλάτωνα δεν ελαμβάνοντο σοβαρά υπόψιν από τους

μαθηματικούς της εποχής του, αφού άλλωστε δεν έπειθαν ούτε καν τους φίλους και άμεσους συνεργάτες του, όπως ήταν ο Εύδοξος, ο Αρχύτας και ο Μέναιχμος.

ο άλλο μεγάλο ζήτημα που είχε απασχολήσει τα ελληνικά Μαθηματικά κατά την προευκλείδεια περίοδο είναι οι άρρητοι ή aσύμμετροι αριθμοί. Η ύπαρξή τους, σύμφωνα με τον Πάππο τον Α­λεξανδρινό, διαπιστώθηκε για πρώτη φορά από τους Πυθαγορείους. Επειδή όμως οι Πυθαγόρει­οι είχαν θεμελιώσει τα μαθηματικά τους και τη φιλοσοφία τους, αποκλειστικά πάνω στα σύμ­

μετρα μεγέθη , ήταν επόμενο η ανακάλυψη της aσυμμετρίας να συγκλονίσει τα θεμέλια της κοσμοθεω­ρίας τους.

Αυτό το αρκετά πιθανό ενδεχόμενο, ορισμένοι εκπρόσωποι της σύγ­χρονης παραδοσιακής μαθηματικής ιστοριογραφίας (B .L.Van der Waerden κ.α.) , όχι μόνο το αποδέχτηκαν ως βέβαιο αλλά το εμφάνισαν και ως δομική κρίση ολόκληρου του οικοδομήματος των αρχαίων ελλη­νικών μαθηματικών. Το περίεργο είναι ότι αυτή είναι σήμερα η επικρα­τούσα εκδοχή της μαθηματικής ιστοριογραφίας για την ανακάλυψη της aσυμμετρίας, η οποία, με ελάχιστες παραλλαγές, επαναλαμβάνεται σε πάρα πολλές σύγχρονες ιστορίες των αρχαίων ελληνικών μαθηματικών. Ωστόσο, παρά την ομοφωνία των σύγχρονων ιστορικών, θεωρώ ότι υ­πάρχει ανάγκη να επανεξεταστούν ορισμένες πτυχές της παραπάνω ι­στορίας, που αφορούν το μέγεθος της κρίσης και τη χρονολογία ανακά-λυψης των άρρητων αριθμών. Ιπποκράτης

Αν οι διαστάσεις της κρίσης ήταν τόσο μεγάλες ώστε να επηρεάσουν στο σύνολό τους τα αρχαία ελληνικά μαθηματικά, τότε θα έπρεπε αυτό να είχε αναφερθεί από κάποιον

ΕΥΚΛΕΙΔΗΣ Β' τ.2/5

Page 8: Ευκλειδης Β 62

-------------- Ιστορικές Μ αθη ματικές Αναφορές --------------

αρχαίο συγγραφέα. Ούτε ο Πρόκλος όμως, ούτε και κανείς άλλος αρχαίος συγγραφέας. αναφέρουν ο­τιδήποτε σχετικά.

Η χρονολογία ανακάλυψης της ασυμμετρίας συνδέεται άμεσα με το όνομα του προσώπου που έκανε την ανακάλυψη . Το ερώτημα είναι αν το πρόσωπο αυτό ήταν ο ίδιος ο Πυθαγόρας ή κάποιος μαθητής του . Αν και κανείς δεν μπορεί να απαντήσει με βεβαιότητα σ' αυτό το ερώτημα, το πιθανότερο είναι ότι την ανακάλυψη δεν την έκανε ο Πυθαγόρας. Κι αυτό γιατί είναι απίθανο ο Πυθαγόρας να είχε ανακαλύψει την ασυμμετρία και ταυτόχρονα να είχε διατυ­πώσει τη φιλοσοφική του θεωρία, στην οποία όπως είναι γνωστό κυριαρχούν οι φυσικοί αριθμοί και η έννοια των σύμμετρων μεγεθών.

Ποιους λόγους όμως είχε η παραδοσιακή μαθηματική ιστοριογραφία να μετατοπίσει χρονικά την πραγματοποίηση της ανακάλυψης μερικές δεκαετίες

Απολλώνιος προς τα πίσω; Η απάντηση είναι ότι μ' αυτόν τον τρόπο επιχειρήθηκε να θεμε-λιωθεί η άποψη ότι τα ελληνικά μαθηματικά, αμέσως μετά την υποτιθέμενη

φάση της πρόσκτησης των γνώσεων από την Ανατολή (Αίγυπτο και Βαβυλώνα), βρέθηκαν μπροστά σε μια απροσδόκητα σοβαρή κρίση . Έτσι λοιπόν για να αποφευχθούν οι συνέπειές της, οι Έλληνες μαθημα­τικοί υποτίθεται ότι αναγκάστηκαν να ντύσουν με γεωμετρικό μανδύα τις βαβυλωνιακές αλγεβρικές γνώσεις. Παράλληλα όμως, ο Van der Waerden και οι άλλοι εκπρόσωποι της παραδοσιακής μαθηματικής ιστοριογραφίας, αντιμετώπιζαν και το πρόβλημα της παραδοσιακής ιστοριογραφίας να δώσει μια ικανο­ποιητική εξήγηση για την παντελή απουσία από την αρχαία ελληνική μαθηματική βιβλιογραφία κάθε ίχνους των βαβυλωνιακών μαθηματικών γνώσεων, δηλαδή των γνώσεων που υποτίθεται ότι μετέφεραν στην Ελλάδα οι Θαλής και Πυθαγόρας.

Για να ευσταθήσει όμως η παραπάνω ερμηνεία ήταν απαραίτητο να υποβαθμιστεί η προευκλείδεια ελληνική μαθηματική παραγωγή, ή σωστότερα η προευκλείδεια ελληνική μαθηματική παραγωγή που δεν ήταν συμβατή με την πυθαγόρεια και την πλατωνική αντίληψη για τα μαθηματικά. Σ' αυτό το ζήτημα η σύγχρονη παραδοσιακή μαθηματική ιστοριογραφία βρήκε απρόσμενο στήριγμα στην αρχαία ελληνική μαθηματική ιστοριογραφία. Η τελευταία, όπως είδαμε, αποσιώπησε πλήρως τη μαθηματική συνεισφορά του Δημόκριτου και των Σοφιστών. Δεν έκανε την παραμικρή αναφορά στις προσεγγιστικές μεθόδους των αρχαίων Ελλήνων, ενώ αποσιώπησε ως κι αυτές ακόμη οι προσπάθειες των ίδιων των Πυθαγορείων να ξεπεράσουν την κρίση της ασυμμετρίας. [Καταβλήθηκε δηλ. μεγάλη παραπληροφόρηση ώστε να πε­ράσει το μήνυμα ότι τα Ελληνικά Μαθηματικά είναι προϊόντα που μεταλαμπαδεύτηκαν στον Ελλαδικό χώρο από . . . βαρβαρικές χώρες] ,

Θαλής Πυθαγόρας

Ποιες ήταν όμως οι προσπάθειες των ίδιων των Πυθαγορείων να ξεπεράσουν την κρίση της aσυμμε­τρίας και γιατί παρέμειναν άγνωστες;

Οι Πυθαγόρειοι προσπάθησαν να ξεπεράσουν το πρόβλημα της ασυμμετρίας με τη λεγόμενη θεωρία των πλευρικών και διαμετρικών αριθμών. Ονόμαζαν πλευρικούς τους αριθμούς που παριστάνουν τα μή­κη πλευρών τετραγώνων και διαμετρικούς τους αριθμούς που παριστάνουν τα μήκη των αντίστοιχων δι­αμέτρων (διαγωνίων) των ίδιων τετραγώνων. Τις πληροφορίες μας για τη σχετική θεωρία τις αντλούμε από δύο κυρίως πηγές: από τον νεοπυθαγόρειο μαθηματικό Θέωνα τον Σμυρναίο (έζησε περίπου το 1 30 μ.Χ.) και από τα <<Σχόλια εις Πλάτωνος Πολιτεία» του Πρόκλου.

Οι πλευρικοί και οι διαγώνιοι αριθμοί, σύμφωνα με τον Θέωνα, δημιουργούνται αν θεωρήσουμε ένα απειροελαχίστως μικρό τετράγωνο, που έχει πλευρά ίση με τη μονάδα και διάμετρο (δηλαδή διαγώνιο)

ΕΥΚΛΕΙΔΗΣ Β' τ.2/6

Page 9: Ευκλειδης Β 62

-------------- Ιστορικές Μαθηματικές Αναφορές --------------

επίσης ίση με τη μονάδα, και κατασκευάζουμε τετράγωνα των οποίων οι πλευρές και οι διαγώνιοι (διά­μετροι) παίρνουν τις εξής τιμές :

Πλευρά Διάμετρος (Διαγώνιος) 1 ο τετράγωνο 1 1 2° τετράγωνο 1 + 1 =2 2+ 1=3 3° τετράγωνο 2+3=5 5+2=7 4° τετράγωνο 5+7= 1 2 1 2+5= 1 7 5° τετράγωνο 1 2+ 1 7=29 29+ 1 2=4 1 6° τετράγωνο 29+4 1 =70 70+29=99 7° τετράγωνο 70+99= 1 69 1 69+70=239 8° τετράγωνο 1 69+239=408 408+ 1 69=577 ............... . . . . . . . . . . . . . . . . . . . . . . . . . . . . . . . . . . . . ν0 τετράγωνο αν- Ι +δv- 1 =αν 2αv-Ι+δv- Ι=δv

Δηλαδή , η πλευρά κάθε τετραγώνου ισούται με το άθροισμα της πλευράς και της διαγωνίου του προ­ηγουμένου τετραγώνου, ενώ η διαγώνιός του ισούται με το άθροισμα της διαγωνίου και του διπλάσιου της πλευράς του προηγουμένου τετραγώνου.

Οι ανωτέρω πλευρικοί και διαμετρικοί αριθμοί εύκολα μπορούν να αποδεί­θουν ότι αποτελούν τις ακέραιες λύσεις της διοφαντικής εξίσωσης ψ2

= 2 χ2 ± 1. Αυτή καθεαυτή η εξίσωση δεν μας βοηθά να κατανοήσουμε καλύτερα τη σχετι­κή θεωρία, μας βοηθά όμως να προσδιορίσουμε τη χρονική περίοδο κατά την οποία διατυπώθηκε. Κι αυτό γιατί την ίδια εξίσωση μνημονεύει ο Πλάτων στην «Πολιτεία» (546 c) του, και άλλωστε αυτό ακριβώς το χωρίο αναλαμβάνει να σχολιάσει και να ερμηνεύσει ο Θέων, παρουσιάζοντάς μας τη θεωρία των πλευ­ρικών και διαμετρικών αριθμών. Συμπεραίνουμε λοιπόν ότι οι πλευρικοί και οι διαμετρικοί αριθμοί ήταν γνωστοί στον Πλάτωνα, συνεπώς θα πρέπει να είχαν Θέων ο Σμυρναίος επινοηθεί πριν από την εποχή του .

Το εύλογο ερώτημα τώρα είναι πως ακριβώς ξεπερνιέται το πρόβλημα της ασυμμετρίας με τη θεωρία των πλευρικών και διαμετρικών αριθμών. Αν σχηματίσουμε το λόγο των διαμέτρων προς τις αντίστοιχες πλευρές αυτών των τετραγώνων, τότε έχουμε :

λ1 = 1 / 1 = 1 λ2 = 3/2= 1 ,5 λ3 = 7/5= 1 ,4 λ4 = 1 7/ 1 2= 1 ,4 1 666 λ5 = 4 1 /29= 1 ,4 137 λv= δvfαv � J2.

Παρατηρούμε ότι οι λόγοι περιττής τάξης αποτελούν μια αύξουσα ακολουθία που έχει ανώτερο φράγμα το J2 , ενώ οι λόγοι άρτιας τάξης αποτελούν μια φθίνουσα ακολουθία που έχει κατώτερο φράγ­μα επίσης το J2 , δηλαδή παρατηρούμε ότι:

� < 2 < � < 239 < . . . J2 . . . < 577 < 99 < .!2 < �. 1 5 29 1 69 408 70 1 2 2 Αν τώρα δοθεί μια αριθμητική τιμή που απέχει ελάχιστα από τη J2, είτε από αριστερά είτε από δε­

ξιά, μπορούμε να καταστήσουμε τη διαφορά των λόγων αυτών από τη J2 όσο μικρή θέλουμε, σχηματί­ζοντας τους επόμενους πλευρικούς και διαμετρικούς αριθμούς.

Η πρώτη παρατήρησή μας αφορά στην επιλογή του αρχικού τετραγώνου, το οποίο έχει πλευρά και διαγώνιο ίσες με τη μονάδα. Ακόμη και οι μη εξοικειωμένοι με τα μαθηματικά γνωρίζουν ότι στην κλα­σική ευκλείδεια γεωμετρία δεν υπάρχει τετράγωνο με πλευρά ίση με τη διαγώνιο . Κάτι τέτοιο θα μπο­ρούσε να συμβεί μόνο σε τετράγωνα με απειροελάχιστα μήκη πλευρών και διαγωνίων, τα οποία ας ονο­μάσουμε απειροστά τετράγωνα. Έτσι λοιπόν, μ' αυτόν τον έμμεσο τρόπο, συμπεραίνουμε ότι οι Πυθαγό­ρειοι είχαν προχωρήσει τις έρευνές τους και στον πολύ ριζοσπαστικό για τα δεδομένα τους τομέα των απειροστών μεγεθών.

Συμπεραίνουμε επίσης ότι οι Πυθαγόρειοι είχαν προχωρήσει τις έρευνές τους και σε έναν άλλο, ακό­μη πιο ριζοσπαστικό τομέα: τις προσεγγιστικές μεθόδους. Τόσο η επιλογή του αρχικού απειροστού

ΕΥΚΛΕΙΔΗΣ Β' τ.2/7

Page 10: Ευκλειδης Β 62

-------------- Ι στορικές Μαθηματικές Αναφορές --------------

Δημόκριτος

τετραγώνου όσο και η χρησιμοποίηση της μεθόδου για τοΥ προσδιορισμό της.fi , είναι φανερό ότι προϋποθέτουν μια προσεγγιστική λογικ-ή. \1ε τη μέθοδο των πλευρικών και διαμετρικών αριθμών δεν προσδιορίζεται επα­κριβώς η .fi (αυτό πρέπει να ήταν σίγουρα γνωστό σ' αυτούς που επινό­ησαν τη μέθοδο), επιτυγχάνεται όμως μια ικανοποιητικότατη προσέγγισή της.

Ακόμη πιο περίεργο είναι ότι οι Πυθαγόρειοι, που εμφανίζονται ως φανατικοί πολέμιοι των προσεγγίσεων, είχαν επινοήσει μια θαυμάσια προ­σεγγιστική μέθοδο για τον υπολογισμό της τετραγωνικής ρίζας ενός αριθ­μού .

Τη μέθοδο επινόησε ο Πυθαγόρειος Αρχύτας ο Ταραντίνος, είναι απλή και εξαιρετικά αποτελεσματική . Εφαρμόζεται δε ως εξής:

Έστω ότι θέλουμε να υπολογίσουμε την .J40. • Κατ' αρχάς πρέπει να βρούμε τον πλησιέστερο τετράγωνο αριθμό

προς το 40, που βεβαίως είναι το 36, διότι J36 =6. Σ , �ο , �ο [40 Η ' θ , , , , τη συνεχεια η -ν 4u γραφεται -ν 4u = ν 6. 6 . προσπα εια μας τωρα ειναι να αντικαταστησουμε τους αριθμούς 6 και 40/6 με μια ικανοποιητική προσέγγισή τους.

• Μια καλή σκέψη είναι να αντικαταστήσουμε καθέναν από τους δύο αριθμούς με τον μέσο όρο τους μ, γιατί τότε η .J40 γράφεται .J40 = � = μ.

6 + 40 • Ήδη έχουμε πετύχει μια πρώτη προσέγγιση της .J40 αφού .J40 = μ = --6- = 6,25 . 2 • Αν θέλουμε μια ακόμα καλύτερη προσέγγιση, μπορούμε να βάλουμε στη θέση του 6 το 6,25 και να

εφαρμόσουμε εκ νέου τη μέθοδο . Έτσι θα πάρουμε την τιμή .J40 = 6 .3250, την οποία αξίζει να συ­γκρίνουμε με την τιμή 6 .3245 που βρίσκουμε εμείς σήμερα. Η μέθοδος του Αρχύτα διασώθηκε από τον Ήρωνα τον Αλεξανδρινό, ο οποίος την χρησιμοποιεί για

τον υπολογισμό τετραγωνικών ριζών. Παρ' όλο που ο Ήρων δεν αναφέρει τίποτα για την αιτία επινόησής της, είναι φανερό ότι ο ουσιαστικός λόγος για την επινόηση της μεθόδου ήταν η εξυπηρέτηση της θεωρί­ας των πλευρικών και διαμετρικών αριθμών. Διαφορετικά η μέθοδος του Αρχύτα εμφανίζεται σαν να μην έχει σκοπό ύπαρξης.

Φαίνεται λοιπόν ότι αρκετά από τα ακλόνητα συμπεράσματα της σύγχρονης μαθηματικής ιστοριο­γραφίας δεν είναι πάντοτε αδιαμφισβήτητα ιστορικά γεγονότα. Α ν χωρίς προκαταλήψεις και υστεροβου­λίες ριχθεί φως σ' αυτές τις σκοτεινές πλευρές της ιστορίας των αρχαίων ελληνικών μαθηματικών, τότε είναι βέβαιο ότι κάθε ενδιαφερόμενος θα σχηματίσει πιο σφαιρική εικόνα γι' αυτήν την τόσο ενδιαφέ­ρουσα περίοδο της ιστορίας της επιστήμης. Μ' αυτή την έννοια, η έρευνα πάνω στην ιστορία των αρχαί­ων ελληνικών μαθηματικών όχι μόνο δεν έχει κορεστεί, αλλά αντιθέτως μόλις τώρα αρχίζει.

ΕΥΚΛΕΙΔΗΣ Β ' τ.2/8

Page 11: Ευκλειδης Β 62

,..' ::--� ;·:;;. 1� ·�,�:�,:Ji; .g: ",�;

Επιμέλεια: Σωτήρης Ε. Λουρίδας 1

ΣΑΒΒΑΤΟ, 9 ΔΕΚΕΜΒΡΙΟΥ 2006

Η Α' τάξη ενός Λυκείου έχει 5 τμήματα που το καθένα έχει τουλάχιστον 20 μαθητές. Σε κα­θένα από τους μαθητές των τμημάτων αυτών δίνουμε 10 €. Έτσι δώσαμε 1090€. Να αποδείξε­τε ότι δύο τουλάχιστον από τα τμήματα αυτά έχουν τον ίδιο αριθμό μαθητών.

Έστω ότι α,β,γ,δ,ε είναι οι αριθμοί των μαθη­τών των πέντε αυτών τμημάτων. Έτσι έχουμε :

1 Ο( α + β + r + δ + ε) = 1 090 � α + β + r + δ + ε = 1 09 (1)

Έστω ότι οι αριθμοί των μαθητών των τμημά­των αυτών είναι ανά δύο διαφορετικοί και έ­στω ότι: α < β < r < δ < ε. Επειδή α 2::20 και α,β,γ,δ,ε φυσικοί έχουμε : β > α 2::20 � β > 20� β2::2 1 r > β;;::: 2 1 � r > 2 1 � r ;;::: 22 δ > r ;;::: 22 � δ > 22 � δ;;::: 23 ε > δ 2:: 23 � ε > 23 � ε 2:: 24 Συνεπώς α +β + y + δ + ε 2:: 110 , άτοπο λόγω

της ( 1 ) . Άρα, δύο τουλάχιστον από τα τμήματα αυ­τά έχουν τον ίδιο αριθμό μαθητών.

Ν α λυθεί η εξίσωση λ ( λχ + 3) = λ 3

+ 2λχ - 2

για τις διαφορές πραγματικές τιμές της παρα­μέτρου λ.

Η εξίσωση γράφεται:

λ 2χ + 3λ =λ 3 + 2λχ - 2 � λ 2χ - 2λχ = λ 3 - 3λ - 2 � χ(λ 2 - 2λ ) =λ 3 - 3λ - 2

λ 2 - 2λ = ο � λ (λ - 2 ) = ο � λ = ο ή λ = 2 1 . Α ν λ =Ο είναι αδύνατη 2 . Αν λ=2 είναι αόριστη 3 . Αν λ:;tΟ και λ;t:2 τότε

λ 3 - 3λ - 2 χ =--:: ---λ 2 - 2λ (λ - 2) (λ + 1 )2 (λ + 1 )2 � χ = � χ = -'-------''---λ (λ - 2 ) λ

Αν α, β, γ πραγματικοί αριθμοί διάφοροι του μηδενός να αποδείξετε ότι: (α +Q.+1.)2 2::3 (α +1.+�)

β γ α

γ β α

Η aνίσωση γράφεται : ( )2 ( )2 ( )2 α β r α r β α r β - + - + - + 2- + 2-+ 2-2::3- + 3- + 3-, β r α r β α r β α αρκεί (; )' + (�) ' + (: ) ' - ; - ; -� �0 , αρκεί

Ξ[(; -�J' + (� - : )\(:-; J}o, η οποία ισχύει.

Αποδεικνύεται και άμεσα αν κανείς γνωρίζει την ανισότητα

( χ + y + z )2 2:: 3xy + 3yz + 3zx

ΕΥΚΛΕΙΔΗΣ Β ' τ.2/9

Page 12: Ευκλειδης Β 62

----------Μαθη ματικοί Διαγωνισμοί - Μαθη ματικές Ολυμπιάδες ----------

Λ Λ

'11. Σε τρίγωνο ΑΒΓ με Α> Β οι διχοτόμοι των γωνιών Α, Β τέμνονται στο Ι. Στην πλευρά ΑΒ

παίρνουμε τμήμα ΒΔ = ΒΓ -ΑΓ. Να αποδείξετε ότι: ΙΔ = ΙΑ.

Ε

Γ

Αν ΓΕ=α τότε ΑΕ=α-β=ΒΔ και ΓΙ η τρίτη δι­χοτόμος.

Δ Δ

Έχουμε ΙΓΕ = IΓΒ διότι ΙΓ=ΙΓ, ΓΕ=ΓΒ και Λ Λ Λ Α Γ 1 = Γ2 άρα Ε = Β1 , ΙΕ=ΙΒ .

Δ Δ Άρα ΙΑΕ = ΙΒΔ διότι ΒΔ=ΑΕ, ΙΕ=ΙΒ και Λ Λ Λ Ε = Β1 = Β2 , άρα ΙΑ=ΙΔ.

!Β' ηJ{ιπος Αρκεί το Ι να ανήκει στη μεσοκάθετο του ΑΔ.

Αν δηλαδή ΙΚ.lΑΔ, αρκεί ΚΑ=ΚΔ. Πράγματι ΚΑ=τα και ΚΔ=ΙΒΚ -ΒΔΙ=Ιτα-β-( α-β) i=Ιτα-α Ι=τα-α, αφού τ>α.

Β' τάξη Λυκείου i . Ν α εξετάσετε αν η εξίσωση

χ2 - (2006κ + l)x + 2007 = Ο

όπου κ ε Ζ, έχει δύο ακέραιες ρίζες.

Αν χ 1 , χ 2 οι ρίζες , τότε χ 1 +χ2 = 2006 κ+ 1 (1) και χ 1 • χ 2 = 2007 (2) .

Από την (2) προκύπτει ότι οι χ 1 , χ 2 θα είναι περιττοί. Αλλά τότε το άθροισμα τους χ 1 + χ 2 θα είναι άρτιος, οπότε δεν θα ισχύει η (1 ) . 2 . Δίνεται ορθογώνιο ΑΒΓ Δ με ΑΒ=4, ΒΓ=2 και σημείο Μ στο εσωτερικό του με ΜΓ=l και MB= .J3 . Να βρείτε το εμβαδόν του τριγώνου

ΜΑΒ.

λ1'1ση Παρατηρούμε ότι ( J3)2 + 1 2 = 4 = 2 2 οπότε το

τρίγωνο ΜΒΓ είναι ορθογώνιο στο Μ . Λ -1

11

\1 �---11

.\ \I.

Επειδή ΜΓ = ΒΓ έχουμε ΜΒΓ = 30° , οπότε 2 Λ Λ ΜΓΒ = 60° και ΜΓΔ = 30° .

Έστω ΜΜ · .l ΔΓ , τότε ΜΜ . = _!_ 2 1 3 Δ v= 2 -- = - . Το εμβαδόν του ΜΑΒ 2 2

1 3 Ε = - · 4 · - = 3 . . 2 2

Β' τρόπος ΜΒ2 = ΒΓ · ΒΗ οπότε 3 = 2 · υ άρα υ = � 2

άρα

είναι

.:ι Έστω κ = 2 + 2 2 + 23 + . . . + 2 2008 • Να αποδεί­ξετε ότι ο 30 διαιρεί τον κ.

.\τί1ση Είναι

κ=(2+i +i +i)+i(2+i +i +i)+ ... +?(2+i +i +i)= =3CX1+i +i + .. . +?)=l1lli30

αφού2+ 22+ 23+24=30 . 4. α) Να αποδείξετε ότι: ifi + if3 > ifl9

β) Να λύσετε την εξίσωση :

- \ - \ 58 2 χ + χ =1r;; 1r;; · {12 + {13

λlJ(Hl ) Α , , , α + β r::Dp

α πο τη γνωστη ανισοτητα: -- > ν α μ 2 όπου α, β θετικοί με α -:f. β, έχουμε:

ifi + ifj > � ifi ifj = If6 2 Οπότε ifi + if3 > 2If6 .

ΕΥΚΛΕ ΙΔΗΣ Β ' τ.2/1 0

Page 13: Ευκλειδης Β 62

----------Μαθη ματικοί Διαγωνισμοί- Μ αθηματικές Ολυμπιάδες ----------

Αρκεί λοιπόν 2� 2':ifl9 , ή 26 ·62':1cJ, ή 3842':361 που ισχύει.

Αν λ= if38 τότε ifi+</3

λ=� </3 . ifi < ifi < J2 οπότε λ2 < 2 . 3 2 + 3 3

Η εξίσωση για χ =F Ο είναι ισοδύναμη με την χ2 - 2λχ+2=0 με Δ=4(λ2 - 2) < 0 .

Άρα η εξίσωση είναι αδύνατη . �3 r τρ6πος

χ2 - 2λχ + 2 = χ2 - 2λχ + λ 2 -λ 2 + 2 = =(χ-λ)2 + (2 -λ2 )2': 2 -λ2 > 0

Γ' τάξη Λυκεi'Ιl! 1 . Έστω συνάρτηση f : IR ---+ IR με την ιδιότητα

f(f(x + y)) =χ- f(y) για κάθε x,y Ε IR. Να

αποδείξετε ότι η h : IR ---+ IR με h(x) = f(x)+ f(-x) είναι σταθερή.

Για χ= Ο έχουμε f(.f(y)) =-f(y) για κάθε Υ EIR.

Για y = Ο έχουμε f(f(x)) =χ- f(O) για κά-θε χ Ε IR.

Άραf(.f(χ)) =-f(x) και f(f(x)) =χ-f( O),

για κάθε χ Ε IR . Επομένως -f(x) =χ - f(O) ή f(x) = f(O)- χ ( 1 ) και /(-χ)= f(O) +χ (2), για κάθε χ Ε IR

Οπότε από τις ( 1 ) και (2) έχουμε : f(x) + f( -χ)= 2/(0) , rια κάθε χ Ε IR.

Άρα η h είναι σταθερή . Ν α βρείτε τις ακέραιες λύσεις της εξίσωσης

3x+l -Χ· 3χ - 4χ -1 =Ο. .\{JGffl

Έστω ότι η εξίσωση έχει μια ακέραια λύση ρ. Τότε γ+ Ι -ρ · 3Ρ - 4ρ - 1 = 0 � 3Ρ (3 - ρ)= 4ρ + 1

3ρ 4ρ + 1 ( ' ' 3 ) 4ρ + 1 � = -- αφου προφανως ρ =F � -- > Ο 3 -ρ 3 -ρ 1 � (4ρ + 1)(ρ - 3) < 0 � -- <ρ < 3 �ρ Ε {0, 1 , 2} . 4

Όπως βρίσκουμε εύκολα, οι αριθμοί ρ=Ο και ρ= 1 δεν είναι λύσεις της εξίσωσης. Ο αριθμός ρ=2 όμως είναι λύση της εξίσωσης. Άρα η εξίσωση έ­χει τη μοναδική λύση ρ=2 . 3. 'Εστω οι μιγαδικοί αριθμοί z1 ,z2 και θ Ε (Ο,�). Να αποδείξετε ότι:

lzι l 2 lz2 l 2 I 1 2 I 1 2 -___.!..__:_c_

2 - + -2- 2': Ζ1 + z2 + 2 Re( z1 z2 ) •

συν θ ημ θ

ι\iJση Επειδή

lz112 +lz2j2 +2Re (�z2)=z1;ι +z2;2 +;ιz2 +;2z1

αρκεί να δείξουμε ότι:

= lzι + 2212

lzιl: + jz2 ( 2': lzι + z212 .

συν θ ημ θ

4. Δίνεται ευθύγραμμο τμήμα ΒΓ και τα σημεία Κ, Λ, Μ προς το ίδιο μέρος της ευθείας ΒΓ.

Αν ΒΚr = ΒΑΓ = ΒΜΓ, τότε να αποδείξετε ότι δύο τουλάχιστον από τα γινόμενα ΚΒ· ΚΓ,

ΛΒ · ΛΓ και ΜΒ · ΜΓ είναι άνισα.

Λ1Jση Από την ισότητα ΒΚΓ = ΒΑΓ = ΒΜΓ έχουμε

ότι τα Κ, Λ, Μ βρίσκονται στο ίδιο τόξο χορδής ΒΓ. Έστω ΒΚΓ = ΒΑΓ = ΒΜΓ = φ.

Αν ΚΒ · ΚΓ= ΛΒ· ΛΓ= ΜΒ· ΜΓ, τότε 1 1 1 -ΚΒ · ΚΓημφ = -ΛΒ · ΑΓημφ = -ΜΒ · ΜΓημφ 2 2 2 � (ΚΒΓ) = (ΛΒΓ) = (ΜΒΓ)

Αυτό σημαίνει ότι τα Κ, Λ, Μ θα βρίσκονται σε ευθεία παράλληλη στην ΒΓ, άτοπο αφού το μέ­γιστο πλήθος κοινών σημείων ευθείας και κύκλου είναι 2 .

ΕΥΚΛΕΙΔΗΣ Β ' τ.2/1 1

Page 14: Ευκλειδης Β 62

---------Μαθη ματικοί Διαγωνισμο ί - Μαθηματικές Ολυμπιάδες ---------

Η 23η Βαλκανική Μαθηματική Ολυμπιάδα Κύπρος, 23 Απριλίου -3 Μα"ίΌυ 2006

Η 23η Βαλκανική Μαθηματική Ολυμπιάδα διοργανώθηκε από την Κύπρο στη Λευκωσία και στον Αγρό, μια ορεινή περιοχή κάτω από το όρος Τρόοδος. Στο διαγωνισμό συμμετείχαν όλες οι Βαλκανικές χώρες πλην της Τουρκίας καθώς επίσης η Αγγλία και το Καζακστάν.

Η Ελληνική ομάδα αποτελούνταν από τους μαθητές Φ. Κουφογιάννης Γ' Λυκείου- Αργυρό Μετάλλιο Ν. Ράπανος Α' Λυκείου- Αργυρό Μετάλλιο Ν. Μπουντάς Β' Λυκείου- Αργυρό Μετάλλιο Κ. Ψύχας Γ' Λυκείου - Χάλκινο Μετάλλιο Σ. Δεσποτάκης Γ' Λυκείου - Εύφημη Μνεία Ν. Παπαγιαννάκης Γ' Λυκείου - Συμμετοχή

Συνοδοί της Ελληνικής ομάδας ήταν ο σύμβουλος από το Παιδαγωγικό Ινστιτούτο Δη μήτριος Κοντογιάννης και ο μαθηματικός Βαγγέλης Ζώτος.

Ο καλός καιρός και η θερμή φιλοξενία των αδελφών Κυπρίων έκανε την παραμονή της Ελ­ληνικής αποστολής αξέχαστη.

Θέματα της 23 ης Βαλκανιάδας ι. Αν α,β,γ, θετικοί αριθμοί να αποδείξετε ότι

1 1 1 3 α ( 1 + β) + β ( 1 + γ) + γ ( 1 + α) � 1 + αβγ ( 1 )

2. Έστω m μια ευθεία που τέμνει τις πλευρές ΑΒ, ΒΓ, Γ Α τριγώνου ΑΒΓ στα σημεία Δ, Ε, Ζ, αντίστοιχα, όπου το Ε βρίσκεται στην προέκταση της ΒΓ. Από τις κορυφές Β, Γ φέρουμε παράλληλες στη m που τέ­μνουν τον περιγεγραμμένο κύκλο c του τριγώνου ΑΒΓ στα Β1, Γι αντίστοιχα. Αν Ρ η τομή των ΒιΖ, Γ ιΔ ενώ Αι η τομή της ΡΕ και του κύκλου c, να αποδείξετε ότι AAιllm.

3. Να προσδιορίσετε όλες τις τριάδες των θε­τικών ρητών αριθμών m, η, p ώστε οι α-

θ ' 1 1 1 ' ' λ ρι μοι m +-, η +-, p +-να ει ναι ο οι

ακέραιοι. np mp mn

4. Έστω m θετικός ακέραιος. Να προσδιορί­σετε όλους τους θετικούς ακεραίους α, ώστε η ακολουθία {α η } �=ο η οποία ορίζεται { 1 '

_2 _ -α11 , αν α" αρτιος ως ασ- και αn+l - 2 αn+m' αν α η περιττός

για n=O, 1 . . . είναι περιοδική ακολουθία, δηλαδή αν οι όροι της επαναλαμβάνονται.

Λύσεις θεμάτων ι. Η ( 1 ) γράφεται

1 + αβγ 1 + αβγ 1 + αβγ 3 , -�..:._ + + > η α(1 + β ) β(1 + γ) γ(1 + α) -( 1 + αβγ + 1) + ( 1 + αβγ + 1) + ( 1 + αβγ + 1) ;::: 6 . α ( 1 + β ) β(1 + γ) γ ( 1 + α) Όμως 1 + αβγ + 1 = 1 + αβγ + α + αβ = 1 + α + αβ ( 1 + γ) = α( 1 + β) α(1 + β) α( 1 + β) α(l + β)

(1 + α

) + β ( 1 + γ) , οπότε η (2) γράφεται α 1 + β 1 + β

[ 1 + α + β ( l + γ) ] + [ 1 + β + γ(1 + α) ] + α ( 1 + β ) 1 + β β(1 + γ) 1 + γ

+ [ 1 + γ + α ( 1 + β ) ] ;::: 6 γ ( 1 + α) 1 + α

ή

[ 1 + α + α ( 1 + β) ] + [ 1 + β + β(1 + γ ) ] + α ( 1 + β ) 1 + α β(1 + γ) 1 + β

+ [ 1 + γ + γ(1 + α) ] ;::: 6 γ(1 + α) 1 + γ Αλλα, 1 + α + α(1 + β) > 2 λ ,

( ) _ κ. .π. οποτε η α l + β 1 + α ( 1 ) ισχύει. 2. Πρώτα θα αποδείξουμε ότι το Ρ ανήκει

στον κύκλο c. Έστω Ρ ι η τομή της Β ιΖ με τον c.

ΕΥΚΛΕΙΔΗΣ Β ' τ.2/12

Page 15: Ευκλειδης Β 62

---------- Μαθη ματικοί Διαγωνισμοί - Μαθηματικές Ολυμπιάδες ----------

Αφού ΒΒι I/ Γ Γι τότε m\ = BJ'. Είναι ΔΖΡι + ΔΑΡι = 1 80° , οπότε ΑΡΖΔ εγ­

γράψιμο. Ας υποθέσουμε ότι η Ρ 1Δ τέμνει τον c στο

σημείο Γ 2· Αφού ΑΔΡι = ΑΖΡι τότε _....--.._ � _....--.._

ΒΓ2 = ΒιΓ2 -ΒΒι = ΒιΓ2 -ΒΓ+ΒιΓ Γι =Γ2 και άρα Ρι =Ρ .

οπότε

Αν rn//AA2, όπου Α2 σημείο του c, τότε οι ευθείες Β1Ζ, ΕΑ2 τέμνονται σε σημείο του c, και αφού η Β1Ζ διέρχεται από το Ρ, τότε Αι =Α2

Β' τρόπος Αν θεωρήσουμε τον περιγεγραμμένο κύ­

κλο στο τρίγωνο ΑΔΓ έστω Ρ η τομή του με το C. Τότε Δι = Ζι.

ΡΖΕ = ΔΑΡ = ΡfΈ οπότε το ΡΖΓΕ εγγρά-ψιμο οπότε Δι = Ζι = Ε ι .

Επίσης Βι = 1 80° -Α=ΡΖΔ. Άρα ΒΒι//m�Βι =ΒΊ �Ρ=Ρι Έτσι έχουμε: ΡΕΖ = ΡfΖ = ΡΑιΑ τότε

ΑΑι I/ m. Προφανώς Γf2p = ΓΑΡ = zΔΡ οπότε ΓΓ 1/ Im οπότε Γ 2=Γ ι.

Ο περιγεγραμμένος κύκλος στο τρίγωνο ΑΔΖ δεν εφάπτεται στο c αφού ΔΖ n ΒΓ = Ε .

3. Αρχικά παρατηρούμε ότι μια λύση είναι η m=n=ρ= 1

α) Έστω α m = ­, χ

b η = ­ ' Υ

α, b, c, χ, z Ε Ν* για του οποίους

c p = -2 όπου

ΜΚΔ ( α, χ) = ΜΚΔ (b, y) = ΜΚΔ ( c, z ) = 1 . τ, θ , θ , α yz b zx οτε α πρεπει οι αρι μοι -+-, -+-,

χ βc y cα c xy , , , -+- να ειναι ακεραιοι οποτε z αb

xbclαbc+xyz ( 1 ) , αyzlαbc+xyz (2), αbzlαbc+xyz (3).

Από την ( 1 ) προκύπτει ότι x/abc και αφού ΜΚΔ (a,x) = 1 τότε x/bc και όμοια από τις (2), (3) έχουμε y/ca, z/ab. Πλλαπλασιάζουμε κατά μέλη τος σχέσεις αυτές και παίρνουμε xyz/(abc)2 (4)

Από την ( 1 ) έχουμε ακόμα bc/xyz και ό­μοια από τις (2), (3) τα xyz, ab/xyz συνεπώς (abc)2/(xyz)2. (5)

Από τις (4) , (5) συμπεραίνουμε ότι οι abc, xyz έχουν τους ίδιους πρώτους παράγοντες των abc, xyz. Έστω ότι αbc * 1 * xyz και ρι,ρ2, . . . ,ρκ οι πρώτοι παράγοντες των αbc, xyz οπότε xyz = ρ � ' ρ�' . . . ρ :' και αbc = ρ ;' ρ �' . . . ρ> ό­που τ;,s; Ε Ν. Τότε ο εκθέτης λ του ρι στο ά­θροισμα abc + xyz είναι λ:?:. (τi, Si) με 1 �i�k.

Όμως από τις ( 1 ) , (2), (3) παίρνουμε η+2si�3min(η, Si) οπότε η,= Si συνεπώς abc =

αbc xyz και άρα mnp =-= 1 (6). Ώστε η (α) ι-xyz σχύει (β) Από την (β) και τις ( 1 ) , (2), (3) προ­κύπτει ότι χ/2α και αφού ΜΚΔ( χ, α)= Ι , τότε χ/2 και όμοια y/2, z/2 άρα xyz/8 και αbc/8 . Παρατηρούμε ότι δεν είναι δυνατόν όλοι οι αριθμοί a, b, c, χ, y, z να είναι άρτιοι, οπότε abc = xyz/4 .

Με άλλους υπολογισμούς στη συνέχεια βρίσκουμε (a, b, c, χ, y, z)=(4, 1 , Ι, Ι, 2, 2) εί­τε (a, b, c, χ, y, z)=(2 , Ι, 1 , 1, 2, 1 ) είτε (a, b, c, χ, y, z)=( 1 , 1 , 1 , Ι, 1 , 1 ), οπότε (m, η, p) =

=( 1 , 1 , 1 ) είτε ( 4,1-,Ξ) είτε ( 2,1-,ι) καθώς

και όλες τις μεταθέσεις τους.

4. Αρχικά παρατηρούμε ότι αν m άρτιος και α=2κt με t περιττό, αk+j= 1+jm, για κάθε j�O, δηλαδή η ακολουθία δεν είναι περιοδική . Αν ο m είναι περιττός θα αποδείξουμε ότι α Ε { 1 , 2, . . . , m } υ {m + 1, m + 3, . . . , 2m - 2, 2m} , δη-λαδή το δεύτερο σύνολο αποτελείται από τους άρτιους από m+ Ι ως 2m. Προφανώς η ακο­λουθία αν είναι περιοδική , αν και μόνο αν, ο α παίρνει δυο τουλάχιστον φορές την ίδια τιμή , οπότε αυτό θα γίνεται άπειρες φορές. τότε εύ­κολα μπορούμε να αποδείξουμε επαγωγικά, ότι αv=2m για κάθε ν και επιπλέον αv�m ( 1 ) , όταν ο αν είναι περιττός. Συνεπώς η ακολουθία είναι φραγμένη, οπότε υπάρχουν όροι της οι οποίοι συμπίπτουν. Έστω k ο μικρότερος μη αρνητικός ακέραιος για τον οποίο ακ= για κά-

ΕΥΚΛΕΙΔΗΣ Β' τ.2/13

Page 16: Ευκλειδης Β 62

----------Μαθη ματικοί Διαγωνισμοί - Μαθη ματικές Ολυμπιάδες ---------

ποιον r>k. Θα αποδείξουμε ότι k=O. Υποθέ­τουμε ότι k=O. Έστω ότι αυτό δεν ισχύει, αλλά

που είναι προφανές ότι ο αj θα είναι περιττός . α. + m Τ' < = -J__ ' < Α '

ότι k>O. Αν ακ:5m, τότε ακ = ακ-Ι και ατ = ατ-Ι οτε αj _ αj+2 2 , οποτε αj _ m . πο τις

2 2 άρα ακ_1 = α τ-I , πράγμα άτοπο. Α ν ακ>m τότε αφού ακ:5m συνεπάγεται ότι ακ=ακ-ι+m και αr=αr-ι+m, οπότε πάλι έχουμε ακ-ι=αr-1 πράγμα άτοπο.

Έστω α�{1,2, . . . ,m}υ{m+ l,m+3, . . . , 2m-2, 2m}

( 1 ) μπορούμε να αποδείξουμε ότι αγ::j:.α όταν ν�j (αυτό είναι φανερό αv2:2m+ 1 αλλιώς ο α θα είναι ένας από τους περιττούς {m+2, m+4, . . . , 2m- 1 } , οπότε δεν μπορεί να είναι ίσος με έναν περιττό αv) . Άρα η ακολουθία είναι περι­οδική .

και έστω αj ο ελάχιστος όρος της ακολουθίας,

ΣημΕίωση Ένας Βούλγαρος μαθητής απέδειξε το θέμα με τη βοήθεια της Θεωρίας των Ομάδων, δη­

λαδή με μη στοιχειώδη τρόπο. Για τη λύση αυτή πήρε ειδικό βραβείο. Αυτό έγινε άλλη μια φο­ρά στην 4η ΒΜΟ όπου ένας Έλληνας μαθητής είχε πάρει επίσης ειδικό βραβείο.

47η Διεθνής Μαθηματική Ολυμπιάδα IMO 2006 Λούμπινα Σλοβενία

Στη Σλοβενία, από 6 έως 1 8 Ιουλίου 2006 διεξήχθη η 47η Διεθνής Μαθηματική Ολυμπιάδα. Οι Λιεθνείς Μ αθηματικές Ολυμπιάδες είναι ένας θεσμός υψηλοτάτου επιστημονικού επι­

πέδου όπου συμμετέχουν τα μεγαλύτερα ταλέντα στο χώρο των μαθηματικών από όλο σχεδόν τον κόσμο.

Η Ελλάδα συμμετείχε με εξαμελή ομάδα την οποία συνόδεψαν ο Καθηγητής του Πανεπιστη­μίου Ιωαννίνων Θεόδωρος Μπόλης και ο μαθηματικός Δημήτριος Κοντοκώστας.

Οι Έλληνες μαθητές πήραν: Καραμιχάλης Ράλλης Ψύχας Κωνσταντίνος Ράπανος Νικόλαος Ρωμανίδης Νικόλαος Σταυροπούλου Ελένη Μπουντας Νικόλαος

Χάλκινο Μετάλλιο Χάλκινο Μετάλλιο Εύφημη Μνεία Εύφημη Μνεία Εύφημη Μνεία Συμμετοχή

Θέματα 47ης Διεθνοί>ς Μαθηματικής Ολυμπιάδας Πρόβλημα 1. Έστω ABC ένα τρίγωνο με έγγεντρο το Ι. Ένα σημείο Ρ στο

εσωτερικό του τριγώνου LPBA + LPCA = LPBC + LPCB.

ικανοποιεί την

Να αποδειχθεί ότι ΑΡ �ΑΙ, και ότι η ισότητα ισχύει αν και Ω μόνο αν P=l .

Λύση. Έστω LA = α, LB = β, LC = γ. Αφού LPBA + LPCA + LPBC + LPCB = β + γ, η συνθήκη στην Β εκφώνηση του προβλήματος είναι ισοδύναμη με την

LPCB + LPCB = (β + γ) I 2, δηλαδή LB PC = 90° + α I 2 . Από την άλλη, LBIC = 180° - (β + γ) I 2 = 90° + α I 2 . Συνεπώς

ΕΥΚΛΕΙΔΗΣ Β' τ.2/ 1 4

Α

Ρ Ι

c ω

Μ

Page 17: Ευκλειδης Β 62

-----------Μαθη ματικοί Διαγωνισμοί - Μαθη ματικές Ολυμπιάδες ----------

LB PC = LBIC, και αφού τα Ρ και Ι βρίσκονται στην ίδια πλευρά ως προς την BC, τα σημεία B ,C,I και Ρ

είναι ομοκυκλικά. Με άλλα λόγια, το Ρ βρίσκεται πάνω στον κύκλο ω του τριγώνου BCI . Έστω Ω ο περιγεγραμμένος κύκλος του τριγώνου ABC Είναι αρκετά γνωστό το γεγονός πως το κέ­

ντρο του ω είναι το μέσο Μ του τόξου BC του Ω. Ετούτο είναι επίσης και το σημείο όπου η διχοτόμος ΑΙ τέμνει τον Ω.

Από το τρίγωνο ΑΡΜ έχουμε ΑΡ + ΡΜ ;:::: ΑΜ = ΑΙ + lM = ΑΙ + ΡΜ. Συνεπώς ΑΡ ;:::: ΑΙ. Η ισότητα ισχύει αν και μόνο αν το Ρ ανήκει στο ευθύγραμμο τμήμα ΑΙ, που

συμβαίνει αν και μόνο αν Ρ = Ι.

J!ll ρί 'ψλη μο 2 . Έστω Ρ ένα κανονικό 2006 - γωνο. Μια διαγώνιος του Ρ καλείται καλή αν τα άκρα της διαιρούν

το σύνορο του Ρ σε δύο μέρη, το καθένα από τα οποία αποτελείται από περιττό πλήθος πλευρών του Ρ. Οι πλευρές του Ρ επίσης καλούνται καλές.

Υποθέτουμε πως το Ρ έχει διαμεριστεί σε τρίγωνα από 2003 διαγώνιες, οποιεσδήποτε δύο από τις οποίες δεν έχουν κοινό σημείο στο εσωτερικό του Ρ. Να βρεθεί το μέγιστο πλήθος ισοσκελών τριγώνων με δύο καλές πλευρές που μπορούν να εμφανιστούν σε μια τέτοια διαμέριση.

,\ �) ι(ίη , Ας καλέσουμε ένα ισοσκελές τρίγωνο καλό αν έχει δύο καλές πλευρές. Ας υποθέσουμε πως μας δίνε­

ται μια διαμέριση όπως στην εκφώνηση του προβλήματος. Ένα τρίγωνο της διαμέρισης που είναι καλό και ισοσκελές θα αποκαλείται εν συντομία ως ισο-καλό .

. ι Έ·� μ μα . Έστω ΑΒ μια από τις διαγώνιες της διαμέρισης, και έστω .C το μικρότερο μέρος του συνό­ρου του 2006 - γωνου με άκρα τα Α, Β. Έστω πως το .C αποτελείται από η τμήματα. Τότε το πλήθος

των ισο-καλών τριγώνων με κορυφές στο .C δεν είναι μεγαλύτερο του η I 2. Λπ;')δειξη . Αυτό είναι προφανές για η = 2. Έστω η με 2 < η � 1 003 και ας υποθέσουμε πως ο ισχυ­

ρισμός αληθεύει για κάθε .C μήκους μικρότερου του η. Έστω τώρα ένα .C (με άκρα τα Α, Β) που απο­τελείται από η τμήματα. Έστω PQ η μεγαλύτερη διαγώνιος που είναι πλευρά ενός ισο-καλού τριγώνου PQS με όλες τις κορυφές του στο .C (αν δεν υπάρχει τέτοιο τρίγωνο, τότε δεν χρειάζεται να αποδείξουμε τίποτε) . Κάθε τρίγωνο με κορυφές στο .C είναι είτε αμβλυγώνιο είτε ορθογώνιο, με την LS τη μεγαλύ­τερη γωνία. Μπορούμε να υποθέσουμε πως τα πέντε σημεία A, P , S, Q, B κείτονται στο .C με αυτή τη

σειρά και διαμερίζουν το .C σε τέσσερα κομμάτια .CAP , Lr.s , 4,Q , �Β (με τα εξωτερικά κομμάτια πιθα­

νώς να συρρικνώνονται σε σημεία) . Από τον ορισμό του PQ ένα ισο-καλό τρίγωνο δεν μπορεί να έχει όλες τις κορυφές του και στο .CAr

και στο �Β . Συνεπώς κάθε ισο-καλό τρίγωνο στο .C έχει όλες του τις κορυφές σε ένα μόνο από τα τέσ­

σερα κομμάτια. Εφαρμόζοντας την επαγωγική υπόθεση σε κάθε ένα από τα κομμάτια και προσθέτοντας τις τέσσερις ανισότητες που προκύπτουν βρίσκουμε πως το πλήθος των ισο-καλών τριγώνων εντός του

.C των διαφορετικών του PQS δεν υπερβαίνει το η I 2 . Και καθώς τα Lr.s , 4,Q αποτελούνται από περιττό

πλήθος πλευρών το καθένα, οι ανισότητες για αυτά τα δύο κομμάτια είναι γνήσιες, επιτρέποντας ένα συ­νολικό πλεόνασμα ίσο με 1 I 2 + 1 I 2. Συνεπώς το άνω φράγμα η I 2 καλύπτει και το τρίγωνο PQS . Αυτό ολοκληρώνει το επαγωγικό βήμα και αποδεικνύει το Λήμμα. D

Στο υπόλοιπο μέρος της λύσης πρόκειται στην πραγματικότητα να επαναλάβουμε το σκεπτικό της προηγούμενης απόδειξης. Ας θεωρήσουμε τη μεγαλύτερη διαγώνιο της διαμέρισης, έστω τη ΧΥ. Ας είναι .ζγ το μικρότερο από τα δύο μέρη του συνόρου με άκρα τα Χ, Υ και έστω ΧΥΖ το τρίγωνο της διαμέρι-

σης με τη κορυφή Ζ όχι επί του .ζγ . Παρατηρήστε πως το τρίγωνο ΧΥΖ είναι αμβλυγώνιο ή ορθογώνιο,

αφού διαφορετικά κάποιο από τα τμήματα ΧΖ, ΥΖ θα ήταν μεγαλύτερο από το ΧΥ. Ονομάζοντας

Lxz , Lyz τα δύο τμήματα του .C που ορίζονται από το Ζ και το Χ ή Υ αντιστοίχως (και που δεν ανήκουν

στο .ζγ ) και εφαρμόζοντας το Λήμμα σε καθένα από τα Lxz , Lyz , .ζγ , καταλήγουμε πως συνολικά δεν

υπάρχουν περισσότερα από 200612 ισο-καλά τρίγωνα, εκτός ίσως κι εάν το ΧΥΖ είναι ένα τέτοιο τρίγω­νο. Όμως σε αυτή την περίπτωση , οι ΧΖ και ΥΖ είναι καλές διαγώνιες και οι αντίστοιχες ανισότητες του

ΕΥΚΛΕΙΔΗΣ Β" τ.2/15

Page 18: Ευκλειδης Β 62

----------Μαθηματικοί Διαγωνισμοί - Μαθη ματικές Ολυμπιάδες ----------

Λήμματος είναι γνήσιες. Ετούτο δείχνει πως και στη ειδική αυτή περίπτωση το συνολικό πλήθος των ισο­καλών τριγώνων στη διαμέριση, συμπεριλαμβανομένου του ΧΥΖ, δεν υπερβαίνει τα 1 003 .

Το φράγμα των 1 003 ισο-καλών τριγώνων μπορεί να επιτευχθεί. Αρκεί να επιλέξουμε μια κορυφή του 2006 - γώνου και ξακινώντας από αυτή να χαράξουμε μια σπαστή γραμμή που ενώνει κορυφή παρά κορυφή . Καθώς το 2006 είναι άρτιος, η γραμμή αυτή κλείνει επιστρέφοντας στην επιλεγμένη κορυφή . Ετούτο μας δίνει ήδη μια διαμέριση του Ρ (όχι όμως σε 2003 τρίγωνα) στην οποία υπάρχουν 1 003 ισο­καλά τρίγωνα. Για να έχουμε μία από τις διαμερίσεις της υπόθεσης με 2003 τρίγωνα, μπορούμε κατόπιν να συμπληρώσουμε τη διαμέριση αυτή με αυθαίρετο τρόπο.

Να βρεθεί ο μικρότερος πραγματικός αριθμός Μ τέτοιος ώστε η ανισότητα I ab(a2 - b2

) + bc(b2 - c2) + ca(c

2 - a2 ) ι� M(a2 + b2 + c2 )2

να ισχύει για όλους τους πραγματικούς και c.

Παρατηρούμε πρώτα πως το αριστερό μέλος παραγοντοποιείται ως

Ι ab( a 2 - b2 ) + bc(b2 - c2 ) + ca( c2 - a 2 ) 1= 1 ( a - b )(b - c )( c - a )( a + b + c) I Ένας τρόπος για να καταλήξουμε εκεί είναι για παράδειγμα να θεωρήσουμε το τριτοβάθμιο πολυώ­

νυμο tb( t2 - b2 ) + bc(b2 - c2 ) + ct( c2 - t2 ) και να ελέγξουμε πως P(b) = Ρ( c) = Ρ( -b - c) = Ο, οπότε

Ρ( t) = (b - c )( t - b )( t - c )( t + b + c ) , μιας και ο συντελεστής του τριτοβάθμιου όρου είναι ο b - c. Συνε­

πώς το αριστερό μέλος Ι ab( a 2 - b2 ) + bc(b2 - c2 ) + ca( c2 - a 2 ) 1=1 Ρ( a) Ι της προτεινόμενης ανισότητας

γράφεται πράγματι ως Ι ( a - b )(b - c )( c - a )( a + b + c) Ι όπως ισχυριστήκαμε.

Συνεπώς το πρόβλημά μας ανάγεται στην εύρεση του μικρότερου Μ που ικανοποιεί την ανισότητα

I (a - b)(b - c)(c - a)(a + b + c) ι� M(a2 + b2 + c2 )2 ( 1 ) Ας παρατηρήσουμε πως αυτή η έκφραση είναι συμμετρική ως προς τα a, b , c και συνεπώς μπορούμε

χωρίς βλάβη της γενικότητας να υποθέσουμε πως a :::; b :::; c. Με αυτήν την υπόθεση , είναι

Ι (a - b)(b - c) I= (b - a)(c - b) :::; ( (b - a) ; (c - b) ) = (c�a)2 , (2)

με την ισότητα να ισχύει αν και μόνο αν b - a = c - b; δηλαδή 2b = a + c. Επίσης,

( ( c - b) + (b - a) )2 � ( c - b )2 + (b - a )2

2 2 '

ή ισοδυνάμως 3(c - a2 / :::; 2[(b - a)2 + (c - b)2 + (c - a)2 ] , (3)

με την ισότητα να ισχύει και πάλι μόνο όταν 2b = a + c. Από τις (2) και (3) παίρνουμε

Ι ( a - b )(b - c )( c - a )( a + b + c) ι::; _!_ Ι ( c - a )3 ( a + b + c) Ι 4 ( 2 2 2 )3 1 �( )ι; ( b )2 1 2 1 (b - a) + ( c - b) + ( c - a) I ( b )2 =- c - a a + + c � - a + + c = 4 4 3 J2[ ( (b - a)2 + (c - b)2 + (c - a)2 )3 ( b ) )2 = - 4 a + + c 2 3

Λόγω της ανισότητας του σταθμικού γεωμετρικού - αριθμητικού μέσου, αυτή η ανισότητα συνεχίζει ως εξής:

I ( a - b )(b - c )( c - a )( a + b + c) ι::; J2 ( (b - a )2 + ( c - b )2 + ( a - c )2 + ( a + b + c )2 )2 =

9J2 ( a 2 + b2 + c2 )2 . 2 4 32

Βλέπουμε λοιπόν πως η ανισότητα ( 1 ) ικανοποιείται για Μ = J__J2 , με ισότητα αν και μόνο αν 32

(b - a)2 + (c - b)2 + (a - c)2 , , , , 2b = a + c και = ( a + b + c )2 . Θετοντας στην τελευταια ισοτητα οπου b το 3

ΕΥΚΛΕΙΔΗΣ Β ' τ.2/ 1 6

Page 19: Ευκλειδης Β 62

-----------Μαθη ματικοί Διαγωνισμοί - Μαθη ματικές Ολυμπιάδες ----------ίσο του ( a + c) I 2 , τη φέρνουμε στην ισοδύναμη μορφή της 2( c - a )2 = 9( a - c ) 2 •

Οπότε οι συνθήκες για την ισότητα μπορούν να διατυπωθούν εκ νέου ως 2b = b + c και ( c - a )2 = 1 8b2 •

Θέτοντας b= 1 βρίσκουμε a = 1 -�.J2 και c = 1 + 'i.J2 . Συνάγουμε λοιπόν πως το M = J___.fi είναι πράγ-2 2 32

ματι η ελάχιστη σταθερά που ικανοποιεί την ανισότητα, και πως η ισότητα ισχύει για κάθε τριάδα (a,b,c)

ανάλογη στην (1 - 'i .fi, 1 , 1 + '}__ .fi) , έως μεταθέσεως. 2 2

rΙ ! i . , ,_, ; ; " ' ι ; i ' "' · Να βρεθούν όλα τα ζευγάρια ακεραίων (x, y) έτσι ώστε 1 + 2' + 22'+ 1 = y 2

Αν (χ, y) είναι μια λύση τότε προφανώς χ � Ο και το (x , -y) είναι επίσης λύση . Για χ = Ο παίρνου­

με τις δύο λύσεις (0, 2) και (0, -2).

Έστω τώρα (χ, y) μια λύση με χ > Ο . Χωρίς βλάβη της γενικότητας ας περιορίσουμε την προσοχή

μας στα y > Ο . Ξαναγράφοντας την εξίσωση ως 2' (1 + 2χ+ ι ) = (y - 1)(y + 1)

βλέπουμε πως οι παράγοντες y - 1 και y + 1 είναι άρτιοι και ακριβώς ένας από αυτούς διαιρείται με το

4 . Συνεπώς χ � 3 και ένας από τους παράγοντες αυτούς διαιρείται με το 2χ-Ι αλλά όχι με το 2Ό Άρα

λοιπόν y = 2x- 1 m + ε, m περιττός ( > Ο ), ε = ± 1 . ( 1 ) Τοποθετώντας τη σχέση αυτή στην αρχική εξίσωση βρίσκουμε

2χ (1 + 2χ+ Ι) = (2 χ-ι m + ε)2 - 1 = 22χ-2 m2 + 2χ mε,

ή ισοδυνάμως 1 + 2χ+Ι

= 22 x-2 m2 + mε.

Οπότε 1 - εm = 2x-z (m2 - 8). (2)

Για ε = 1 , το αριστερό μέλος της (2) γίνεται 1 - m � Ο ( αφού m περιττός και θετικός), οπότε το δεξί μέλος ικανοποιείται μονάχα για m = 1 Όμως για ε = 1 και m = 1 , η (2) δεν ικανοποιείται.

Για ε = - 1 η σχέση (2) μας δίνει τη 1 + m = 2χ-2 (m2 - 8) � 2(m2 - 8),

που συνεπάγεται τη 2m2 - m - 1 7 � Ο . Συνεπώς m � 3 . Από την άλλη όμως το m δεν μπορεί να ισούται με 1 εξαιτίας της (2). Καθώς ο m είναι περιττός, πρέπει m = 3 , που συνεπάγεται χ = 4 . Τότε από την ( 1 ) βρίσκουμε y = 23 . Αυτές οι τιμές των χ , y ικανοποιούν πραγματικά τη δοσμένη εξίσωση . Ας ανα-

καλέσουμε πως τότε η τιμή y = -23 είναι επίσης καλή . Συνεπώς η πλήρης λίστα των λύσεων είναι η : (0, 2), (0, -2), ( 4, 23), ( 4 , -23) .

1 ; ; rM) ': . • I : ' , J'

Έστω Ρ(χ) ένα πολυώνυμο βαθμού η > 1 με ακέραιους συντελεστές και έστω k ένας θετικός α­κέραιος. Θεωρήστε το πολυώνυμο Q(x) = Ρ(Ρ( . . . Ρ(Ρ(χ)) . . . )), όπου το Ρ εμφανίζεται k φορές. Να αποδειχθεί πως υπάρχουν το πολύ η ακέραιοι t τέτοιοι ώστε Q(t) = t.

Ο ισχυρισμός του προβλήματος είναι προφανής στην περίπτωση που κάθε ακέραιο σταθερό σημείο του Q είναι και σταθερό σημείο του ίδιου του Ρ. Ακολούθως ας υποθέσουμε πως αυτό δε συμβαίνει.

Σαν πρώτο βήμα θα δείξουμε πως κάθε ακέραιο σταθερό σημείο χ0 του Q(x) , είναι και ακέραιο

σταθερό σημείο του πολυωνύμου Ρ(Ρ(χ)) .

Πράγματι, αν χ0 είναι ένα σταθερό σημείο του Q που είναι και σταθερό σημείο του Ρ, τότε φανερά

το χ0 θα είναι και σταθερό σημείο του Ρ(Ρ(χ)) . Ας θεωρήσουμε λοιπόν τώρα έναν οποιοδήποτε ακέραιο

χ0 τέτοιο ώστε Q(x0 ) = χ0 , Ρ(χ0 ) :;t: χ0 (υποθέσαμε προηγουμένως πως τέτοιο χ0 υπάρχει) και ας ορί­

σουμε αναδρομικά χ i+ Ι = P(x i ) για i = 0, 1 , 2, . . . . Τότε x k = χ0 •

Είναι προφανές πως το Ρ( u) - Ρ( ν) διαιρείται με το u - ν για διακριτές τιμές u, ν. ( 1 )

(Πραγματικά, αν Ρ(χ ) = Σ a i xi

τότε κάθε a i (u i - ν

i ) διαιρείται με το u - ν ) .

Ας παρατηρήσουμε τώρα πως οι διαφορές χ0 - χ 1 , χ 1 - χ2 , • • · , χ k- Ι - x k , x k - χ k+ Ι είναι όλες

ΕΥΚΛΕΙΔΗΣ Β ' τ.2/17

Page 20: Ευκλειδης Β 62

-----------Μαθη ματικοί Διαγωνισμοί - Μ αθη ματικές Ολυμπιάδες ----------μη μηδενικές. Πραγματικά, αν για κάποιο i είχαμε Χ ; = χ i + Ι , τότε αφού χ0 = Ρ(Ρ( . . . Ρ(χ 0 ) · · · ) ) ( το Ρ k φορές), εφαρμόζοντας και στα δύο μέλη το πολυώνυμο Ρ k - i φορές θα καταλήγαμε στην ισότητα χ0 = Ρ(χ0 ) σε αντίθεση με την υπόθεσή μας χ0 ::ι:. Ρ(χ0 ) .

Συνεπώς μπορούμε να εφαρμόσουμε την ( 1) για τις διακριτές τιμές Χ ; , χ i+ Ι για κάθε i � k , καταλήγοντας

πως κάθε όρος στην ακολουθία των διαφορών χ0 - χ 1 , χ 1 - χ 2 , · · · , xk_ 1 - x k , x k - x k+ Ι (2)

είναι διαιρέτης της επόμενης, και αφού xk - x k+ l = χ0 - χ 1 , όλες αυτές οι διαφορές έχουν ίσες απόλυτες

τιμές. Για Χ 111 = miη(x 1 , - · · , x k ) , ετούτο σημαίνει πως χ m- Ι - χ ιη = -(χ ιη - χ ιη+ 1 ) . Οπότε

χ m- Ι = χ ιη+ Ι (::ι:. χ ιη ). Έπεται πως διαδοχικές διαφορές στην ακολουθία (2) έχουν αντίθετα πρόσημα. Συνε­

πώς, χ0 , χ 1 , . . . είναι μια ακολουθία δύο διακεκριμένων τιμών. Άρα λοιπόν χ0 = χ2 = Ρ(Ρ(χ0 )) .

Δείξαμε πράγματι πως κάθε ακέραιο σταθερό σημείο χ0 του Q(x) , είναι και ακέραιο σταθερό ση­

μείο του πολυωνύμου Ρ(Ρ(χ)) . Αρκεί τώρα να δείξουμε πως υπάρχουν το πολύ η σε πλήθος ακέραια

σταθερά σημεία του Ρ(Ρ(χ)).

Έστω a ένα ακέραιο σταθερό σημείο του Ρ(Ρ(χ)) τέτοιο ώστε b = P(a) ::ι:. a (υποθέσαμε πιο πάνω

πως τέτοιο a υπάρχει) . Φυσικά ισχύει a = P(b) και το b είναι επίσης σταθερό σημείο του Ρ(Ρ(χ)) . Ας

θεωρήσουμε ένα οποιοδήποτε άλλο ακέραιο σταθερό σημείο a ι του Ρ(Ρ(χ)) , διαφορετικό από τα και b και ας θέσουμε b ι = P(a ι) , οπότε φυσικά a ι = P(b ') . Οι ακέραιο a ', b ' δεν είναι υποχρεωτικό να είναι δι­

ακεκριμένοι (ο a ' ίσως και να είναι σταθερό σημείο του Ρ( χ) ), αλλά καθένας από τους a ', b ' είναι δια­

φορετικός από καθέναν από τους a, b . Εφαρμόζοντας την ιδιότητα ( 1 ) στα τέσσερα ζευγάρια των ακε­ραίων (a, a ') , (b, b '), (a, b ') , (b, a ') προκύπτει πως οι ακέραιοι a - a ' και b - b ' διαιρούν ο ένας τον άλλον,

και πως ομοίως οι ακέραιοι a - b ' και b - a ' . Οπότε b - a ' = ±(a - b '), a - a ' = ±(b - b ') . (3 )

Α ν και στις δύο σχέσεις το πρόσημο είναι το συν : b - a ' = a - b ' και a - a ' = b - b ' . Αφαιρώντας κατά μέλη έχουμε πως b - a = a - b δηλαδή b = a , άτοπο λόγω της υπόθεσής μας πως b ::ι:. a . Άρα λοιπόν σε μία τουλάχιστον σχέση της (3) το πρόσημο είναι το μείον. Όποια από τις δύο και αν είναι αυτή η σχέση , ισοδυναμεί με την a + b = a '+ b ' ή ισοδυνάμως με την a + b - a '- Ρ( a ') = Ο .

Δείξαμε πως κάθε ακέραιο σταθερό σημείο a ' της Ρ(Ρ(χ)) άλλο από τα a , b είναι ρίζα του πολυω­

νύμου F(x) = a + b - χ - Ρ( χ). Φυσικά και τα a, b είναι ρίζες του F(x). Αφού ο βαθμός η του Ρ είναι

> 1, το πολυώνυμο F θα έχει ομοίως βαθμό η. Συνεπώς το F δεν μπορεί να έχει περισσότερες από η ρί­ζες και το αποτέλεσμα αποδείχθηκε.

Π ρόβλημα 6. Σε κάθε πλευρά b ενός κυρτού πολυγώνου Ρ aντιστοιχούμε το μέγιστο εμβαδόν ενός τριγώνου

που έχει τη b σαν μια πλευρά και περιέχεται στο Ρ. Να αποδειχθεί πως το άθροισμα των εμβαδών που ανιστοιχούν στις πλευρές του Ρ είναι τουλάχιστον διπλάσιο του εμβαδού του Ρ.

λu'Jen� . . fl ι!μμα. Κάθε κυρτό (2η) - γωνο (Q), εμβαδού S έχει μια πλευρά και μια κορυφή που μαζί ορίζουν

ένα τρίγωνο εμβαδού όχι λιγότερο από S/n Α πriδπξη. Λέγοντας κύριες διαγώνιες για το (2η) - γωνο (Q) θα εννοούμε εκείνες που το διαμερί-

ζουν σε δύο πολύγωνα με ίσο πλήθος πλευρών. Για κάθε πλευρά r = ΑΒ του (Q) ας συμβολίσουμε με Δr το τρίγωνο ΑΒΟ όπου Ο είναι το σημείο τομής των κυρίων διαγωνίων ΑΑ ', ΒΒ ' . ( Φυσικά οι διαγώ-

νιες αυτές τέμνονται) . Ισχυριζόμαστε πως η ένωση όλων των τριγώνων Δr , καθώς η r διατρέχει όλες τις

πλευρές του (Q), καλύπτει ολόκληρο το πολύγωνο (εδώ θεωρούμε τα τρίγωνο και το πολύγωνο μαζί με τα εσωτερικά τους) εκτός ίσως από κάποια σημεία των κύριων διαγωνίων. (τα σημεία των κύριων δια­γωνίων μπορούμε να τα εξαιρέσουμε με ασφάλεια όσον αφορά τους συλλογισμούς μας για τα εμβαδά) .

Για να το δείξουμε αυτό ας θεωρήσουμε για οποιαδήποτε κορυφή Υ, την ευθεία της κύριας διαγωνί­ου ΥΥ ' ως άξονα με προσανατολισμό από το Υ προς το Υ ' . Έστω Χ ένα οποιοδήποτε σημείο του πολυ­γώνου, μη κείμενο σε οποιαδήποτε από τις κύριες διαγωνίους.

Ας επιλέξουμε οποιαδήποτε κορυφή Α του 2η - γωνου (Q) και ας θεωρήσουμε πως το Χ βρίσκεται στην αριστερή πλευρά του άξονα ΑΑ ' . Θεωρούμε την ακολουθία των αξόνων των κύριων διαγωνίων

ΕΥΚΛΕΙΔΗΣ Β' τ.2/ 1 8

Page 21: Ευκλειδης Β 62

----------Μ αθη ματικοί Διαγωνισμοί - Μ αθη ματικές Ολυμπιάδες ----------

AA1, BB, CC 1, . . . όπου A, B, C, - · · είναι διαδοχικές κορυφές του πολυγώνου, δεξιά του άξονα ΑΑ ' . Το

η - στο μέλος της ακολουθίας αυτής είναι ο άξονας Α 1 Α (δηλαδή ο ΑΑ 1 αντεστραμμένος), ο οποίος έ­χει το Χ στη δεξιά του πλευρά. Συνεπώς υπάρχουν δύο διαδοχικές κορυφές Κ, L στην ακολουθία Α, Β, C, · · · πριν φτάσουμε στο σημείο Α 1 έτσι ώστε το Χ να εξακολουθεί να βρίσκεται αριστερά του ά­

ξονα ΚΚ 1 , αλλά δεξιά του LL 1 • Αυτό σημαίνει πως το Χ βρίσκεται στο τρίγωνο Δ1 όπου t = Κ 1L 1 • Α­

νάλογο σκεπτικό εφαρμόζεται και στα σημεία Χ που βρίσκονται αριστερά του άξονα ΑΑ 1 • Οπότε πράγ­ματι η ένωση όλων των τριγώνων Δ, , καθώς η r διατρέχει όλες τις πλευρές του 2η - γώνου (Q) καλύ­

πτει ολόκληρο το (Q). Β' Β'

Α' C ' Α'

(Q) ο χ

Α r Β Α Β c

Κ'

' Δt ι(_ __ L' ,:_ . . . - - ·

Ν L s./ �-Δ�'''< _ "··-c

Μ κ

z Μ'

Δ5, s' Ν '

Άρα λοιπόν άθροισμα όλων των Δ, είναι τουλάχιστον ίσο με S Συνεπώς μπορούμε να βρούμε δύο

απέναντι πλευρές, έστω r = MN και r 1 = M 1N 1 (με ΜΜ ', ΝΝ 1 κύριες διαγώνιες) ώστε

[Δ, ] + [Δ, . ] � S / η , όπου οι αγκύλες [· . . ] δηλώνουν το εμβαδόν ενός χωρίου . Έστω πως οι διαγώνιες

ΜΜ 1, ΝΝ 1 τέμνονται στο Ζ, και πως ισχύει (δίχως βλάβη της γενικότητας) ΖΝ � ΖΝ ' . Τότε

[ΜΝΜ 1] = [ΜΝΖ] + [ΖΝΜ 1] � [ΜΝΖ] + [ΖΜ 1Ν 1] = [Δ, ] + [Δ, . ] � S / η, και το Λήμμα έχει αποδειχθεί. D Έστω τώρα S το εμβαδόν του κυρτού πολυγώνου (Ρ) της εκφώνησης, και

a 1 , a 2 , . . . , a ln οι πλευρές του . Ας είναι Si το εμβαδόν του μεγαλύτερου τρι­

γώνου εντός του πολυγώνου με μία πλευρά την a i . Ας υποθέσουμε αντίθε­

τα με το ζητούμενο πως

s τότε θα υπάρχουν ρητοί αριθμοί q 1 , q2 , • • · , qm τέτοιοι ώστε και q i > _.!_ για

s κάθε ί. Έστω η ένας κοινός παράγοντας των παρονομαστών των κλασμά-

_ ki των q 1 , q2 , . . . , qm , οπότε qi - - για κάποιους θετικούς ακέραιους με

η

(Q)

(Ρ)

τ

b a . ι

Η

Σki = 2η. Διαμερίζουμε κάθε πλευρά a i του πολυγώνου (Ρ) σε ki ίσα τμήματα και δημιουργούμε ένα

κυρτό 2η - γωνο (Q) (με κάποιες γωνίες ίσες με 1 80° ) εμβαδού S . Το Λήμμα ισχύει για (Q) σαν και το δικό μας όπου κάποιες από τις γωνίες του ίσες με 1 80° . Εφαρ­

μόζοντάς το συμπεραίνουμε πως το (Q) έχει μια πλευρά b και μια κορυφή Η που ορίζουν τρίγωνο Τ εμβαδού [τ] � S / η. Αν η πλευρά b του (Q) είναι κομμάτι της πλευράς a i του Ρ για κάποιο i, τότε το

τρίγωνο Τ 1 με βάση a i και απέναντι κορυφή Η έχει εμβαδόν [Τ 1] = ki [τ] � ki · S I η = qi · S > Si , δηλαδή

[τ '] > Si , πράγμα που αντίκειται στην ιδιότητα που κουβαλάει το Si εξ'ορισμού.

Οπότε κακώς υποθέσαμε Σ§_ < 2, και η απόδειξη ολοκληρώθηκε. s

Τα ονόματα των επιτυχόντων μαθητών Γυμνασίου - Λυκείου στο διαιyωνισμό «0 ΘΑΛΗΣ» θα ανακοινωθούν κατά νομό, σχολείο στο επόμενο τεύχος 6� του i

ΕΥΚΛΕΙΔΗΣ Α ' - ΕΥΚΛΕΙΔΗΣ Β ', αντίστοιχα . . � �

ΕΥΚΛΕΙΔΗΣ Β' τ.2/1 9

Page 22: Ευκλειδης Β 62

π I

Η στήλη αυτή έχει ως στόχο την ανάπτυξη μαθηματικού διαλόγου. Φιλοδοξούμε να συμμετάσχουν όλοι όσοι έχουν ένα γενικότερο ενδιαφέρον για τα Μαθηματικά.

Επιμέλεια: Γιάννης Στρατής - Βαγγέλης Ευσταθίου

Η Άλγεβρα των Δεικτών του Ρολογιού (Το ρολόι κτυπά και εξισώσεις)

Ε ισ αγωγ ι κά - ο ρ ισμο ί Σε ορισμένα βιβλία, κυρίως Άλγεβρας Α ' Λυ­

κείου, σχολικά και μη , υπάρχουν μεμονωμένα α­πλά προβλήματα που οδηγούν σε εξισώσεις α ' βαθμού και είναι σχετικά με το ρολόι.

Αναφερόμαστε στα κλασικά ρολόγια με τα λε­πτά και τους τρεις δείκτες (ωροδείκτης, λεπτοδεί­κτης, δευτερολεπτοδείκτης), και όχι στα σύγχρονα ηλεκτρονικά.

Στο άρθρο αυτό θα δούμε μια γενίκευση των προβλημάτων αυτών καθώς και άλλα προβλήματα που σχετίζονται με το ρολόι και τους δείκτες του .

Τα προβλήματα αυτά παρουσιάζουν ενδιαφέ­ρον για τους μαθητές, επειδή αναφέρονται σε ένα χρήσιμο και γνωστό όργανο και συνδυάζουν γνώ- . σεις Άλγεβρας και Γεωμετρίας, καθώς και πρακτι­κής αριθμητικής, χωρίς να είναι απλά, όπως έχω διαπιστώσει ακόμη και για μαθητές Γ Λυκείου.

Αναφέρουμε κατ ' αρχήν δυο, λίγο - πολύ γνω­στούς κανόνες, που θα μας χρησιμεύσουν παρακά­τω και έχουν σχέση με τα 60 διαστήματα-γραμμές που, ως γνωστόν, χωρίζεται ο κυκλικός δίσκος του ρολογιού.

1 . Όταν ο μικρός δείκτης (ωροδείκτης) κινεί­ται από μια ώρα στην επόμενη , διαγράφει 5 δια­στήματα-γραμμές και στον χρόνο αυτό, δηλαδή σε μια ώρα, ο μεγάλος δείκτης (λεπτοδείκτης) δια­γράφει 60 γραμμές- λεπτά. Άρα ο μικρός δείκτης διαγράφει 1 γραμμή σε χρόνο 12 λεπτών.

2 . Μια γραμμή στον κύκλο του ρολογιού αντι­στοιχεί σε τόξο 360/60 = 6° . Άρα γωνία δεικτών γωνία ω (οι γωνίες αναφέρονται σε μοίρες) αντι­στοιχεί σε ω/6 γραμμές.

Έστω ΟΜ ο μεγάλος δείκτης (λεπτοδείκτης), ΟΚ ο μικρός δείκτης (ωροδείκτης) και ΟΔ ο δευ­τερολεπτοδείκτης του ρολογιού. Την γωνία ω =

Λ ΜΟΚ που διαγράφει ο μεγάλος δείκτης κινούμε­

νος κατά την κανονική φορά κίνησής του ( αρνητι-

Δημήτρης Ι. Μπουνάκης

κή φορά) μέχρι να συμπέσει με τον μικρό δείκτη ΟΚ λέμε (στο άρθρο αυτό) γωνία των δεικτών του ρολογιού. Επίσης όταν αναφερόμαστε σε δυο δεί­κτες ή απλά δείκτες, θα εννοούμε τον μικρό και τον μεγάλο δείκτη του ρολογιού.

Πρόβλημα ι Να βρεθεί ποια ώρα μεταξύ ν και ν+l ώρας

ο μεγάλος και ο μικρός δείκτης του ρολογιού συναντιούνται, ν = Ο, 1, 2, . . . , 1 1 .

Λύση Κατ' αρχήν από ώρα Ο ( 1 2) μέχρι 1 οι δείκτες

δεν συναντιούνται (συμπίπτουν) , οπότε ν= 1 ,2, . . . , 1 1 . Ο μικρός δείκτης ξεκινά από το ν και ο μεγάλος από το 1 2 (Σχήμα 1 ) . Έστω ότι, μέχρι την θέση σύμπτωσης έχουν παρέλθει λv λεπτά (m) , δηλαδή ο μεγάλος έχει κινηθεί κατά λv γραμ­μές . Σ ' αυτόν τον χρόνο ο μικρός έχει διαγράψει (λv - 5ν) γραμμές (κάθε ώρα ο μικρός κινείται κατά 5 γραμμές) . Όμως κάθε γραμμή του μικρού αντι­στοιχεί σε χρόνο 1 2 λεπτών, άρα

60 ν λv = 1 2( λv -5ν) <=> λv = -

1 1

1 2

Σχήμα 1

ΕΥΚΛΕΙΔΗΣ Β ' τ.2/20

Page 23: Ευκλειδης Β 62

Το Βήμα του Ευκλείδη

Άρα υπάρχουν 1 1 σημεία συνάντησης των δει­h 60 ν m κτών τα Σv = ν -- , ν = 1 ,2 , . . . , 1 1 .

1 1 τα οποία διαφέρουν μεταξύ τους κατά 1 ώρα 60 5 5 λ ' ( θ ' ' δ ' και U = U επτα αρι μητικη προο ος με πρω-

το όρο και διαφορά 1 ώρα και 5 2._ λεπτά) . Την 1 1 στιγμή της συνάντησης η γωνία

ΑΟΜ = 6. 60 ν = 360ν ο.

π

1 1 1 1

i , Από τις 7 μέχρι τις 8 οι δυο δείκτες συμπί­πτουν στις 7 και

λ 7 . 60 420 3 8 2 λ , τ , 1 = -'1-1- = U = U επτα. ην στιγμη αυ-

, ' ΑΟΛΜ ' 360' 7 229 1 ο ο δ τη η γωνια ει ναι -- = - . ευ-1 1 1 1 τερολεπτοδείκτης ΟΔ την στιγμή αυτή μετρά

2 60 1 20 1 0 1 0 δ ' λ ' U · = U = U ευτερο επτα και η γωνια

ΟΛ , 1 20 6 6 5 0 Α Δ ειναι - · = 5- . 1 1 1 1

1 , Από τις 4 μέχρι τις 5 οι δυο δείκτες συνα-ντιούνται την ώρα Σ4 = 4 1' 2 1 _2._ m • Την στιγμή 1 1

' ' ΑΟΛΜ ' 360· 4 1 30 1 0 ο αυτη η γωνια ει ναι -- = -

1 1 1 1 Μετά την 1 ώρα οι δείκτες συναντιούνται

στις ν = 1 και λ 1= 60 = 52_ λεπτά. 1 1 1 1

r. ι Από τις 1 1 μέχρι τις 1 2 οι δείκτες συμπί­πτουν στις 1 1 και λ 1 1 = 60 λεπτά, δηλαδή στις 1 2 . Την στιγμή αυτή συμπίπτουν και οι τρεις δείκτες.

r! ι) �Η σ η ο Ο μικρός και ο μεγάλος δείκτης του ρολογιού

συναντιούνται σε ακέραιο αριθμό λεπτών μόνο την ώρα 1 2 .

Πράγματι, πρέπει το 1 1 να διαιρεί το 60ν, οπότε ν = 1 1 και λ = 60.

Π ρ�φιη μ α 2 Να βρεθεί ποια χρονική στιγμή μεταξύ της

ν και ν+ 1 ώρας ο μεγάλος και ο μικρός δείκτης του ρολογιού σχηματίζουν γωνία ω, ν= 0, 1 , . . . , 1 1 .

Την ώρα ν , ν = 0, 1 , . . . , 1 1 (ώρα Ο είναι η 1 2) ο μικρός δείκτης βρίσκεται στο ν και μεγάλος στο 1 2 . Καθώς ο μικρός κινείται από το ν στο ν+ l , δι­αγράφοντας 5 γραμμές, ο μεγάλος κινείται από το

1 2 στο 1 2 και στην αρχή προηγείται ο μικρός, ενώ μετά την συνάντησή τους προηγείται ο μεγάλος (πάντα κατά την φορά κίνηση των δεικτών­αρνητική φορά) . Διακρίνουμε λοιπόν δυο περι­πτώσεις

Π φ iπτωση •\ . Ο μικρός δείκτης προηγείται του μεγάλου ή συμπίπτει με αυτόν.

Λ Αυτό συμβαίνει όταν η γωνία ΑΟΜ (με αρχι-κή πλευρά ΟΑ και αρνητική φορά διαγραφής) εί­ναι μικρότερη από την γωνία που αντιστοιχεί στον

' ' δ λ δ ' ' 60 ν 6° Α ' δ χρονο συμπτωσης, η α η απο U · . υτο εν

μπορεί να συμβεί από την ώρα 0( 1 2) μέχρι την 1 , αφού προηγείται πάντα ο μεγάλος, οπότε εδώ ν= 1 ,2 , . . . , 1 1 .

Σχήμα 2 Έστω ότι, όταν οι δείκτες σχηματίζουν γωνία ω

(Σχήμα 2) ο μεγάλος έχει κινηθεί κατά λ λεπτά (από την θέση 1 2) . Σ' αυτόν τον χρόνο ο μικρός έχει κινηθεί από την θέση ν και έχει διαγράψει (λ+ ω -5ν) γραμμές (βλέπε και κανόνα 2 στην ει-6 σαγωγή) .

, ω 60 ν - 2ω Άρα λ = 1 2(λ+6 -5ν) <:::> λ = 1 1 ( 1 ) Άρα οι δείκτες σχηματίζουν γωνία ω την ώρα ν

60 ν - 2ω , και λ = λεπτα. 1 1 Την στιγμή αυτή η γωνία

ΑΟΜ = 60 ν - 2ω . 6 = 1 2(30ν - ω) . 1 1 1 1 Η ( 1 ) , δ , 60 ν - 1 1λ ειναι ισο υναμη με την ω = 2

που εκφράζει την ω συναρτήσει του λ. Όταν οι δείκτες συμπίπτουν, δηλαδή ω = Ο, τό-

, , 60 ν λ , , δ τε η ωρα ειναι ν και -- επτα, που ειχαμε ει 1 1 και στο προηγούμενο πρόβλημα.

ΕΥΚΛΕΙΔΗΣ Β ' τ.2/2 1

Page 24: Ευκλειδης Β 62

Το Βήμα του Ευκλείδη

Δ ιφι:ι} ιιηση Πρέπει Ο�λ� 60 ν οπότε Ο�ω�30ν, ν= 1 ,2 , . . . , 1 1 1 1 Άρα υπάρχει μια μέγιστη γωνία ω = 30ν από

την ν μέχρι την ν+ 1 ώρα την οποία σχηματίζουν ακριβώς την ώρα ν. Στην συνέχεια η γωνία ω μι­κραίνει μέχρι να γίνει Ο στο σημείο συνάντησης. Έτσι, π.χ. στις 7 έχουμε γωνία ω = 2 1 0° η οποία μικραίνει μέχρι να γίνει Ο, όταν συμπέσουν οι δεί­κτες. Η μέγιστη τιμή του ω είναι 330° στις 1 1 . Από την ώρα αυτή μέχρι τις 1 2 προηγείται πάντα ο μι­κρός μέχρι να συμπέσουν (ω = 0).

Παρατηρούμε ότι, με σταθερό ν, ο λ είναι γνη­σίως φθίνουσα συνάρτηση της ω.

Π α ρ α δ Σ iγματα i ο Μεταξύ της ώρας 4 και 5 και πριν συνα­

ντηθούν, οι δείκτες σχηματίζουν γωνία ω = 20° στις

ν = 4 λ 60 ν - 2ω 200 1 8 2 λ , και = = - επτα. 1 1 1 1 1 1 Την στιγμή αυτή η γωνία είναι

Λ 200 1 ' ΑΟΜ =- · 6 = 1 09- 0, ο δευτερολεπτοδεικτης 1 1 1 1 μετρά 2 · 60

= 1 0!Q δευτερόλεπτα και η γωνία 1 1 1 1 ΑΟΔ = 720 = 652_ 0 • 1 1 1 1

2 0 Την ώρα 7 .30 προηγείται ο μικρός, οπότε (ν=7, λ=30) οι δείκτες θα σχηματίζουν γωνία ω με

30 = 60 · 7 - 2 ω ή ω = 45 ο ο 1 1 3 ο Την ώρα 1 0 . 1 5 προηγείται ο μικρός δεί­

κτης, οπότε (ν = 1 0, λ = 1 5) οι δείκτες θα σχηματί­ζουν γωνία ω με 1 5= 60 " 1 0 - 2 ω ή ω = 2 1 7,5° ή κυρτή γωνία 1 42,5° . 1 1

4 . Ποια ώρα μετά τις 8 οι δείκτες σχηματίζουν γωνία 1 80° ; Είναι ν = 8, ω = 1 80° οπότε ( αφού προηγείται ο μικρός) αυτό συμβαίνει στις 8 και λ 60 ° 8 - 360 1 20 1 0 1 0 λ , Γ , , = = - = - επταο ενικα οταν 1 1 1 1 1 1 προηγείται ο μικρός οι δείκτες είναι σε ευθεία γω-, , 60 . (ν - 6) λ , 6 1 1 νια τις ωρες ν και επτα , ν = , . . . , . 1 1

Ο αριθμός αυτός των λεπτών είναι ακέραιος μόνο για ν = 6.

5 ο Την ώρα 1 1 030 οι δείκτες σχηματίζουν γω­νία ω με

60° 1 1 - 2ω 30 ή ω = 1 65° . 1 1

Ας σημειωθεί ότι από τις 1 1 μέχρι και τις 1 2 προηγείται πάντα ο μικρός (μέχρι να συμπέσουν) .

6. Αμέσως μετά τις 9 οι δείκτες σχηματίζουν ευθεία γωνία, στις 9 και

λ= 60 · 9 - 2 · 1 80 = 1 80 = 1 6_i_ λεπτά. 1 1 1 1 1 1 Π i ερ ίπτωση Β . Ο μεγάλος δείκτης προηγείται

του μικρού. Αυτό συμβαίνει μετά την συνάντηση των δει-

Λ κτών, δηλαδή όταν η ΑΟΜ (με αρχική πλευρά ΟΑ και αρνητική φορά διαγραφής) είναι μεγαλύ­τερη της γωνίας που αντιστοιχεί στον χρόνο σύ-μπτωσης, δηλαδή της γωνίας 6�1ν ·6° με ν=Ο, 1 ,2 , . . . , 1 1 . Αυτό δεν μπορεί να συμβεί από τις 1 1 μέχρι τις 1 2 , οπότε εδώ ν = 0, 1 ,2, . . . , 1 0 .

1 2

Σχήμα 3 Έστω ότι, όταν οι δείκτες σχηματίζουν γωνία

ω (Σχήμα 3), ο μεγάλος έχει κινηθεί κατά λ λεπτά (από την θέση 1 2) . Σ ' αυτόν τον χρόνο ο μικρός έχει κινηθεί από την θέση ν και έχει διαγράψει (λ- 360 - ω -5ν) γραμμές (βλέπε και κανόνα 2) . 6 Άρα λ=(λ- 360 - ω _5ν) · 1 2<=:>λ= 60(ν + 1 2) - 2ω (2) 6 1 1 Επομένως οι δείκτες σχηματίζουν γωνία ω, με­

ταξύ της ώρας ν και ν+ 1 , την ώρα 60(ν + 1 2) - 2ω , ν και λ = λεπτα , ν = 0, 1 , . . . , 1 0 . 1 1

Το ν = Ο αντιστοιχεί στις 1 2 . Λ .ιr:ρ:: ι) Ι 'ηση Επειδή ΟΟν �λ�60 προκύπτει 30( ν+ 1 )�ω�3 60° . 1 1

Άρα κατά την διάρκεια της ώρας από ν σε ν+ 1 , όσο προηγείται ο μεγάλος, η γωνία ω ξεκινά από τις 360° , την στιγμή της συνάντησης των δεικτών

ΕΥΚΛΕΙΔΗΣ Β' τ.2/22

Page 25: Ευκλειδης Β 62

Το Βήμα του Ευκλείδη

(μέγιστη τιμή) , και στην συνέχεια ελαττώνεται μέ­χρι να πάρει την ελάχιστη τιμή της 30(ν+ 1 ) . Για ν = Ο έχουμε την μικρότερη γωνία ω=30° από τις 1 2 μέχρι τις 1 . Παρατηρούμε ότι, με σταθερό ν, ο λ είναι γνησίως φθίνουσα συνάρτηση της ω.

Σημείωση Αν στην περίπτωση Β θεωρήσουμε σαν γωνία

Λ δεικτών την ω = ΚΟΜ (με αρχική πλευρά ΟΚ και Λ αρνητική φορά διαγραφής και όχι την ΜΟΚ ) τότε

η σχέση (2) γίνεται απλούστερη λ= 60(ν + 1 2) - 2(360 - ω) = 60 ν + 2ω . 1 1 1 1

j ::.:υνi:zπu ι π ιJ ετrό μ : :νο τειΊzο.; Ι

Χριίσιμη Παραηίρηση Εκ παραδρομής δεν είχαμε δημοσιεύσει την

παρατήρηση του συναδέλφου Παναγιώτη Οικονο­μάκου πάνω στο πρόβλημα 3 της Α ' Λυκείου στο τεύχος 53 (Αύγουστος, Σεπτέμβριος, 2004) . Γι ' αυτό επανορθώνουμε δημοσιεύοντάς το .

Το π ρ ίJ βλη μ α 3 : (Θ . Κυρ ιακόπουί.ο; ) Αφού αποδειχθεί η ταυτότητα:

( α2+β2+γ2)(χ2+y2+z2)-( αχ+βy+γz)2= =(βχ-αy)2+(γy-βz)2+(γχ-αz)2 να βρεθεί στο εσωτερικό τριγώνου ΑΒΓ σημείο

Ρ έτσι ώστε «αν χ, y, z είναι οι αποστάσεις του Ρ από τις πλευρές του τριγώνου και x+y+z=c όπου c>O σταθερός, το άθροισμα x2+y2+z2 να είναι το μικρότερο δυνατό»

Α

Β

Η ωτι'iντηση τ ο υ π�:ρωδ ικο{J Για το πρώτο :

Γ

Εκτελούμε τις πράξεις και στα δυο μέλη της αποδεικτέας ταυτότητας και οι παραστάσεις που προκύπτουν είναι ίδιες. Για το δεύτερο: Από την ταυτότητα για α=β=γ= 1 παίρνουμε :

3 (χ2 + y2 + z2 ) - ( x + y + z)2 = = ( x - y)2 + ( y - z)2 + ( x - z)2

χ2 + / + z2 =�[(( x - y)2 + ( y- z)2 + (z - x)γ ] +� c2 Συνεπώς: χ 2 + y2 + z2 :?: .!.c2 3

Το «=» ισχύει μόνον όταν x=y=z. Άρα το ζη­τούμενο σημείο είναι το σημείο που συναντιούνται οι διχοτόμοι των γωνιών του τριγώνου . Θυμίζουμε ότι: «η διχοτόμος μιας γωνίας αποτελείται από όλα τα σημεία του επιπέδου, που ισαπέχουν από τις πλευρές της γωνίας» .

Αυτό συμβαίνει μόνον όταν το τρίγωνο είναι ισόπλευρο με πλευρά α και τότε.

αJ3 x + y + z = c =-- . 2 Σχόλιο : Το σημείο αυτό λέγεται έγκεντρο (κέ­

ντρο του εγγεγραμμένου κύκλου) , οπότε x=y=z=ρ (ρ η ακτίνα του εγγεγραμμένου κύκλου . Συνεπώς c=3ρ)

Η πα ρ ατi1 ρ η ση του Π. Ο ι κονο μ ά κου Το πρόβλημα 3 της σελίδας 19 (περιόδου Ιου­

λίου-Αυγούστου-Σεπτεμβρίου 2004) έχει λανθα­σμένη υπόθεση , διότι αν Ρ μεταβλητό σημείο στο εσωτερικού τυχαίου τριγώνου ΑΒΓ και χ, y, z εί­ναι οι αποστάσεις του από τις πλευρές του, τότε το άθροισμα x+y+z δεν είναι σταθερό .

Στο τυχαίο τρίγωνο ΑΒΓ ακυρώνουμε τη λαν­θασμένη και περιττή υπόθεση x+y+z=c, αφού μας αρκεί ότι ισχύει πάντα αχ+βy+γz=2Ε= σταθερό, όπου Ε το εμβαδόν του τριγώνου ΑΒΓ

Ακόμα: (x2+/+z2) ( α2+β2+γ2)-( αχ+βy+γz)2= =(αy-βχ)2+(αz-γχ)2+(βz-γy)2 (ταυτότητα του

Lagrange) οπότε και αντικαθιστώντας έχουμε : (x2+/+z2) (α2+β2+γ2)-4Ε2= =( αy-βχ )2+( αz-γχ )2+(βz-γy )2�0 οπότε 2 2 2 > 4Ε2 χ + y + z - 2 2 2 α + β + γ

Άρα η ελάχιστη τιμή για το άθροισμα x2+/+z2 4Ε2 είναι η 2 2 2 α + β + γ

Και επιτυγχάνε-, χ Υ z ται οταν - = - = -α β γ ,

δηλαδή όταν το ση­μείο Ρ συμπέσει με το σημείο από το οποίο περνούν οι 8

Α

τρεις συμμετροδιά- Γ μεσοι του τριγώνου.

Στη συνέχεια ο συνάδελφος Π . Ο ι κονο μ ά κος κάνει αναφορές στο γνωστό θεώρημα των συμμε­τροδιαμέσων και των ιδιοτήτων του .

Βιβλία που Λάβαμε: ΑΛΓΕΒΡΑ Α' Ενιαίου Λυκείου,

Θ. Τσιούμας και I. Σιάχος.

ΕΥΚΛΕΙΔΗΣ Β' τ.2/23

Page 26: Ευκλειδης Β 62

{�_ ,... ·: ι Α f: ι ; -�-!:tr (Ί l, .. �1'" i·· /.�

/�; .-{--5 : "fi!$ ,(, ,Α Λι#),..,.:.�ι Λl7 ��t R ;if :1-"�� : t' '<," ί<f� :� ,.t'f·�}# Αί {t.<ι [3>4'ι ' �#,�· '

Η Homo Mathematicus είναι μια στήλη στο περιοδικό μας, με σκοπό την ανταλλαγή απόψεων και την ανάπτυξη προβληματισμού πάνω στα εξής θέματα: 1 ) Τι είναι τα Μαθηματικά, 2) Πρέπει ή όχι να διδάσκονται, 3) Ποιοι είναι οι κλάδοι των Μαθηματικών και ποιο το αντικείμενο του καθενός, 4) Ποιες είναι οι εφαρμογές τους, 5) Ποιες επιστήμες ή κλάδοι επιστημών απαιτούν καλή γνώση των Μαθηματι­κών για να μπορέσει κάποιος να τους σπουδάσει.

�, ι . Ι · Ι -. ·' : Ι

παράκληση ! τα κείμενα της στήλης αυτής, ως προς το περιεχόμε­νό τους και ως προς το επίπεδό τους, θα πρέπει να είναι συμβιβαστά με τα ενδιαφέροντα και το επίπεδο κατανόησης από μέρους των παιδιών.

� � Ι c < > � Ι . ι ψ ι" ., - , , , ., : ι · ' ·; Η στήλη μας ανταποκρινόμενη στην επιθυμία πολλών αναγνωστών, καθιερώνει από το τεύχος αυτό και στο εξής, ένα νέο υπόθεμα με τίτλο «τι είναι τα Μαθηματικά; ». Εί­ναι σε όλους γνωστό πως δεν υπάρχει μοναδική απάντηση σ' αυτό το ερώτημα. Οι απαντήσεις ποικίλουν ανάλογα με την εποχή που διατυπώθηκαν και τη φιλοσοφική τοποθέτηση αυτού που τη διατύπωσε. Εμείς θα καταβάλουμε σοβαρή προσπάθεια να αποδώσουμε όσο γίνεται πιο πιστά τις διάφορες τοποθετήσεις. Θα θεωρήσουμε, όμως, πολύ σημαντικό γεγονός τις δικές σας παρεμβάσεις. Με τη βοήθειά σας και τις υποδείξεις σας, όπως πάντα, είμαστε βέβαιοι ότι θα πετύχουμε το στόχο μας.

Είναι σε όλους γνωστό πως ο Πυθαγόρας (και γε­νικότερα η σχολή των πυθαγορείων) δεν άφησαν γραπτά κείμενα και τις όποιες πληροφορίες τις έ­χουμε «εκ πλαγίου». Απ' όλες τις «πλάγιες» μαρ­τυρίες, εμείς θεωρούμε πιο έγκυρες αυτές του Αρι­στοτέλη . Διαβάζουμε : « . . . οι λεγόμενοι Πυθαγόρειοι κατα­

πιάστηκαν πρώτοι με τα μαθηματικά και όχι μόνο τα ανέπτυξαν αλλά καθώς αυτά τους έγιναν δεύτε­ρη φύση, θεώρησαν ότι οι αρχές τους είναι αρχές των όντων. Επειδή λοιπόν στα μαθηματικά οι αριθ­μοί έρχονται από τη φύση τους πρώτοι και επειδή θεωρούσαν ότι στους αριθμούς βλέπουν πολλές ομοιότητες με τα όντα και τα γεγονότα, περισσότε­ρες απ' ότι στη φωτιά, στη γη και στο νερό, ότι ο τάδε αριθμός με τις συγκεκριμένες ιδιότητες είναι η δικαιοσύνη , ο τάδε η ψυχή και ο νους, ο άλλος ο καιρός και ούτω καθεξής. Στους αριθμούς ακόμη έβλεπαν τις ιδιότητες και τις σχέσεις των μουσικών

αρμονιών. Αφού, λοιπόν, όλα τα άλλα όντα φαίνο­νταν να είναι τέλεια απεικόνιση των αριθμών και αφού οι αριθμοί είναι πρωταρχικοί σε ολόκληρη τη φύση , θεώρησαν ότι τα στοιχεία όλων των όντων και ότι ολόκληρος ο ουρανός δεν είναι παρά αρμο­νία και αριθμός όσες, λοιπόν, αναλογίες μπορού­σαν να δείξουν ανάμεσα στους αριθμούς και στις αρμονίες αφενός και στις μεταβολές, τα μέρη του ουρανού και τη γενική διάταξη του κόσμου αφετέ­ρου, τις συγκέντρωναν και τις ενσωμάτωναν στο σύστημά τους. Και αλλού : «Πρόσθεσαν, όμως, και ένα νέο στοι­

χείο, που είναι καθαρά δικό τους: το Περατό, το Άπειρο και το Ένα δεν τα θεώρησαν ότι απλά ανή­κουν σε διαφορετικές φύσεις, όπως η φωτιά, η γη ή κάτι παρόμοιο, αλλά το ίδιο το άπειρο και το ίδιο το ένα είναι η ουσία των πραγμάτων στα οποία α­ποδίδονται. Γι ' αυτό θεώρησαν τον αριθμό ως ουσία όλων των πραγμάτων»

" f : Αριστοτέλη «Μετά τα Φυσικά», 985b, 24-986a,5 και Α 987a, 1 4 [στο επόμενο : η γνώμη των Πλατωνικών]

ΕΥΚΛΕΙΔΗΣ Β ' τ.2/24

Page 27: Ευκλειδης Β 62

Μια εξήγηση για κάτι που πολλοί θα θεωρούσαν παράξενη σύμπτωση

«Όλοι μας έχουμε εντυπωσιαστεί από διάφορες συμπτώσεις, άλλες ευχάριστες και άλλες δυσάρε­στες. Συμβάλλουν κι αυτές στο να σημαδευτεί η ζωή μας από κάποια γεγονότα. Αρκετοί άνθρωποι προσδίδουν μεταφυσικές προεκτάσεις σε πολλές συμπτώσεις, παρασυρόμενοι ίσως από τα ωροσκό­πια και τις άλλες μεθόδους «πρόβλεψης» του μέλ­λοντος.

Σκοπός του άρθρου αυτού δεν είναι να εξηγήσει όλες τις συμπτώσεις. Σίγουρα, πολλές εξηγούνται επιστημονικά. Εμείς θα εξετάσουμε μια πολύ συ­γκεκριμένη περίπτωση : το να έχουν δυο τουλάχι­στον άτομα, που παρευρίσκονται σε μια κοινωνική συγκέντρωση, κοινή ημερομηνία γέννησης. Φυσικά θα πρέπει να παρευρίσκονται λιγότεροι από 367 άνθρωποι για να έχει νόημα το υπό εξέταση ζήτη­μα. Αφού ο χρόνος μπορεί να έχει 366 ημέρες (αν συμπεριληφθεί και η 29η Φεβρουαρίου ), αν συγκε­ντρωθούν τουλάχιστον 367 άτομα, σίγουρα θα υ­πάρχουν τουλάχιστον δυο με κοινή ημερομηνία γέννησης.

Σε όλη τη διαπραγμάτευση που θα γίνει θα θεω­ρηθεί ότι η κάθε ημερομηνία έχει την ίδια πιθανό­τητα να εμφανιστεί ως ημερομηνία γέννησης ενός ατόμου.

Α ν μαζευτούν λοιπόν σε μια κοινωνική συγκέ­ντρωση 50 άτομα, ποια είναι η πιθανότητα δυο τουλάχιστον εξ αυτών να έχουν κοινή ημερομηνία γέννησης; Ίσως πολλοί νομίζουν ότι η πιθανότητα αυτή είναι μάλλον μικρή, αφού το 50 είναι μικρό­τερο από το 1 17 του 366 . Η διαισθητική αυτή αντί­ληψη είναι όμως λάθος όπως θα δείξουμε πιο κάτω.

Κατ ' αρχήν θα υπολογίσουμε την πιθανότητα του συμπληρωματικού ενδεχομένου : «και οι 50 να έ­χουν διαφορετικές ημερομηνίες γέννησης».

Χάριν απλότητας θα θεωρούμε ότι το έτος έχει 365 ημέρες. Κάθε ένας από τους 50 ανθρώπους έχει 365 δυνατές ημερομηνίες γενεθλίων. Ο πρώτος μπορεί να έχει τα γενέθλιά του οποιαδήποτε μέρα του χρόνου, άρα για αυτόν υπάρχουν 365 επιλογές. Για τον δεύτερο υπάρχουν 364 επιλογές γιατί δεν πρέπει τα γενέθλιά του να συμπέσουν με αυτά του πρώτου, για τον τρίτο υπάρχουν 363 επιλογές γιατί δεν πρέπει τα γενέθλιά του να συμπέσουν με αυτά των δυο προηγούμενων. Συνεχίζουμε ομοίως και για τους 50 ανθρώπους και βλέπουμε ότι για τον προτελευταίο υπάρχουν 3 1 7 επιλογές και για τον τελευταίο υπάρχουν 3 1 6 επιλογές ημερομηνίας γε­νεθλίων.

Επομένως η πιθανότητα να έχουν και οι 50 διαφορετικές ημερομηνίες γενεθλίων είναι 365 . 364

· . . . · 3 1 7 . 3 1 6 = 365 · 364 · . . . · 3 1 7 · 3 1 6 = 0.03 . 365 365 365 365 36550

Άρα η πιθανότητα που ζητάμε είναι ίση με 1 - 0 .03 = 0.97 = 97% . Τελικά είναι σχεδόν βέ­βαιο ότι θα βρεθούν τουλάχιστον δυο άτομα με κοινή ημερομηνία γέννησης ανάμεσα σε 50 άτομα.

Με παρόμοιους υπολογισμούς διαπιστώνουμε ότι αρκούν μόνο 23 άτομα για να είναι η εν λόγω πιθα­νότητα ίση με 50%. Δηλαδή μια σχολική τάξη . Δεν δοκιμάζετε, αγαπητοί μας μαθητές, να πειραματι-

ΠΛΗΘΟΣ ΑΤΟΜΩΝ ΠΙΘΑΝΟΤΗΤΑ

B I RTHDAYS ΟΝ ΤΗΕ SAt1E DAY

� :q � ;;; it ι;; � Numlιe<r of Birtlιιdags

στείτε πάνω στο θέμα αυτό με τα παιδιά της τάξης σας;

Αν δοκιμάσουμε να δούμε τι γίνεται για 70 και πλέον άτομα θα διαπιστώσουμε ότι η πιθανότητά μας πλησιάζει το 1 . Παρακάτω παραθέτουμε έναν πίνακα και δυο διαγράμματα:

ΕΥΚΛΕΙΔΗΣ Β' τ.2/25

Page 28: Ευκλειδης Β 62

------------------------------ HdArd Af�/H&AfAr/CVf -----------------------------

Οι μαθηματικοί συχνά αρέσκονται να εξηγούν σε άλλους ανθρώπους το παραπάνω φαινόμενο . Παρατίθεται η παρακάτω ιστορία:

πογοητεύτηκε όταν έφτασε και στον τελευταίο χω­ρίς να πετύχει ούτε μια σύμπτωση . Αλλά η κατά­σταση σώθηκε από το 24 ο πρόσωπο που βρέθηκε στην αίθουσα, την υπηρέτρια, που ανήγγειλε ξαφ-

Σ ' ένα δείπνο, ο μεγάλος μαθηματικός Γ . Γουί- νικά ότι είχε γεννηθεί την ίδια ημερομηνία με έναν βερ εξηγούσε στους άλλους 22 συνδαιτυμόνες, α- από τους στρατηγούς» [η συνέχεια στο επόμενο] νώτερους στρατιωτικούς, τις πιθανότητες να συ-μπίπτουν οι ημερομηνίες γεννήσεων, κι άρχισε να ρωτά τον καθένα ποια ημερομηνία γεννήθηκε. Α-

Τηλέμαχος :V1 παλτσαβιάς ( Κεφαλονιά)

i l l . «Η τ ρ ιχοτό μη ση γων ίας απ ' τη σκοπιά των πυθ αγορείων»

Στο υπ ' αριθμ. 61 τεύχος του Β ' ΕΥΚΛΕΙΔΗ Ιουλίου- Σεπτεμβρίου 2006 και στη στήλη Homo Mathematicus παρουσιάστηκε μια ωραία εργασία με τον τίτλο «Μια τριχοτόμος καμπύλη» του Νίκου Βαδιβούλη από την Άρτα. Επίσης στην εισαγωγή της και στον επίλογό της υπάρχει μια ιστορική και συμπερασματική αναφορά ότι η τριχοτόμηση μιας τυχούσας γωνίας με κανόνα και διαβήτη είναι, Μα­θηματικά αποδεδειγμένα, ΑΔ ΥΝΑ ΤΗ, αναφέρο­ντας αορίστως τη σχετική ανακοίνωση του Ρ .L . W antzel του έτους 1 83 7, πράγμα που η παγκόσμια Μαθηματική κοινότητα το έχει μέχρι σήμερα απο­δεχθεί.

Όμως το 1 994 παρουσίασα δημοσίως, ( . . . ), μα­θηματική απόδειξη ότι η άνω ανακοίνωση του Wantzel είναι λανθασμένη (αριθμός πνευματικής ιδιοκτησίας 877/23/5/1 989 Εθνικής Βιβλιοθήκης της Ελλάδος) συνεπώς και τα συμπεράσματά του δέον να είναι λανθασμένα ότι δηλ. «η τριχοτόμηση τυχούσας γωνίας με κανόνα και διαβήτη είναι αδύ­νατη , διότι η τριτοβάθμια εξίσωση χ3-3αχ2-3χ+α=Ο, όπου α=εφ3θ και χ=εφθ που επικαλείται δεν έχει ρίζες γραμμικές και δεν μπορεί να τραπεί σε γινόμενο γραμμικών παραγόντων, πρώτου βαθ­μού», παρ ' όλον που υπάρχουν χιλιάδες περιπτώ­σεις τριχοτομήσεως ατομικών εκάστοτε γωνιών.

Η απόδειξη του λάθους είναι απλούστατη . Η άνω εξίσωση είναι μία Τ Α ΥΤΟΤΗΤ Α και ουχί εξί­σωση, διότι προήλθε από τη γνωστή τριτοβάθμια τριγωνομετρική ταυτότητα

3θ 3εφθ - εφ3θ

εφ = -'----------::..__ 1 - 3εφ2θ

κατόπιν αντικαταστάσεως των τριγωνομετρικών συμβόλων με σύμβολα αλγεβρικά (εφ3θ=α και

εφθ=χ), πράγμα που δεν δύναται να αλλάξει την υφή της ταυτότητος και να την μετατρέψει σε απλή εξίσωση . άρα, η άνω εξίσωση ως ταυτότητα πλέον έχει απείρους λύσεις και γραμμικές και τρέπεται και σε γινόμενο γραμμικών παραγόντων, όπως εί­ναι οι ομάδες των γωνιών των παραδειγμάτων που τριχοτομήθηκαν (υπάρχει σχετική αναφορά στο τεύχος 5 8 στην αυτή στήλη , του Β ' Ευκλείδη της Ε.Μ.Ε.) .

Βέβαια το πρόβλημα εξακολουθεί να υπάρχει αλλά δεν είναι πλέον άλυτο και άρα η προσπάθεια δέον να είναι επαινετή και όχι απαγορευμένη .

Επικουρικά σας αναφέρω, ( . . . ), ότι υπάρχει σχετική μελέτη μου της γενίκευσης της υπ ' αρ . 1 0/IV πρότάσης των στοιχείων του Ευκλείδη ( . . . ) , κατά την οποία ζητείται «να κατασκευασθεί ισο­σκελές τρίγωνο που η γωνία της βάσεώς του να εί­ναι νι-πλάσια της γωνίας της κορυφής», και από την οποία καταφαίνεται για ν=3 ή ν= l /3 η δυνατό­τητα επίλυσης της τριχοτόμησης τυχούσας γωνίας. Αλλά για το θέμα αυτό καλό θα ήταν μια ιδιαιτέρα διερεύνηση από ομάδα μαθηματικών της ΕΜΕ προς την οποία δύναμαι να αναπτύξω την άνω γε­νίκευση, και τον τρόπο εφαρμογής αυτής δια την κατασκευήν τυχούσης γωνίας, με κανόνα και δια­βήτη .

Δεν παραλείπω να αναφέρω ότι από την άνω γενίκευση προέκυψε και η Νομοτέλεια της κατα­σκευής όλης της γκάμας των κανονικών ΠΟΛ Υ­ΓΩΝΩΝ που κατασκευάζονται όταν ο λόγος (ν) πάρει τις τιμές ν=Ο,25λ , ( . . . )

Ι ππ ο κ ρ άτη ς Λά κογλο υ , Α θ tΊνα

I V. Η τ ρ ιχοτό μη ση γωνίας - οφ ειλό μενη αναφο ρ ά τη ς στήλη ς

Από τον εκλεκτό φίλο της στήλης Ν. Βαδιβούλη, σαμε στο τεύχος 61 (σελ. 23-25) . Συγκεκριμένα στη λάβαμε ένα σημείωμα με το οποίο εντοπίζει κάποιο σελ. 24 (αριστερή στήλη), οι έξι πρώτες γραμμές του λάθος στο κείμενο της εργασίας του που δημοσιεύ- κειμένου, θα πρέπει να τοποθετηθούν στη δεξιά στή-

ΕΥΚΛΕΙΔΗΣ Β ' τ.2/26

Page 29: Ευκλειδης Β 62

H��� ��THE�AT/CVf ----------------------------­

λη (της ίδιας σελίδας) αμέσως μετά τις τρεις πρώτες Αυτό, όμως, δεν απαλλάσσει τον υπεύθυνο για το συ­γραμμές του κειμένου και συγκεκριμένα αμέσως με- γκεκριμένο κείμενο (τον Γιάννη Κερασαρίδη), ο ο­τά τη φράση «του θεωρήματος του συνημιτόνου της ποίος ζητά συγγνώμη από το Νίκο Βαδιβούλη και γωνίας». από τους αναγνώστες του. Ελπίζουμε, όσο είναι αν-

θρώπινα δυνατό, να μη επαναληφθεί αυτό το φαινό­Αυτή η μετάθεση του κειμένου (απ ' τη μια στήλη μενο.

στην άλλη), έγινε με υπαιτιότητα του wπογραφείου.

ν . Σύντο μες αναφο ρ ές σε θέματα επ ικαιρ ότητας των Μ αθη ματ ικών

• Η πίκρα τη ς αποτυχίας κρίJ βει τον σπό ρ ο της επ ιτυχίας Ο Άγγελος Σισμάνης είναι ένας μαθητής της Α '

τάξης στο 3° Λύκειο Νίκαιας. Με τον Άγγελο γνωριστήκαμε σε μια τριημερίδα με θέμα τα Μα­θηματικά. Ο καλός μας φίλος, έσπευσε να στείλει στη στήλη ένα γράμμα απ ' το οποίο δημοσιεύουμε μόνο μερικά τμήματα, ( . . . )

( . . . ) δεν ξεχνώ τα λόγια από την παρουσίαση της ταινίας για τον λύτη του προβλήματος του

Fennat, στα πλαίσια του συνεδρίου «ΑΦΗΓΗΜΑ­ΠΚΕΣ ΤΕΧΝΕΣ και ΜΑΘΗΜΑΠΚΑ»: «η πίκρα της αποτυχίας κρύβει τον σπόρο της επιτυχίας» . Η κατάκτηση της γνώσης των μυστηρίων της φύσης με στρέφουν προς τη θετική κατεύθυνση και συνε­πώς η ενασχόλησή μου με τα μαθηματικά και το σκάκι θα συνεχίσουν για πολλά χρόνια ακόμα να με απασχολούν και να αναλώνουν δημιουργικά τον χρόνο μου ( . . . )

<�> \ ιεθνής συνάντη ση σε θέματα Ι στο ρ ίας κ α ι Π α ιδαγωγικών των Μ αθη ματικών

Στις 1 4 και 1 5 Απρίλη του 2006 πραγματοποιή­θηκε στη Θεσσαλονίκη «Ελληνική Συνάντηση της Intemational Study Group ση the Relations Be­tween the History and Pedagogy of Mathematics» (ΗΡΜ Group ), στην οποία παρουσιάστηκαν ενδια­φέρουσες εργασίες από καταξιωμένους συναδέλ­φους μαθηματικούς. Ο φίλος της στήλης Γιάννης ΘωμαΊδης (σημαντικός ερευνητής της Ιστορίας των Μαθηματικών) παρουσίασε μια εργασία με τίτλο «Διδακτικές όψεις της γενίκευσης στα Αριθμητικά

� :\'1 ι α Γεωμετ ρ ι κή Π ρ αγματεία

Από το φίλο της στήλης ΘΗνι'i ση �. ΤζεμπΕ­λίκο, λάβαμε μια 60-σέλιδη πραγματεία με τίτλο «Μια Γεωμετρική Πραγματεία», με την παράκλη­ση να δημοσιευτεί στη στήλη μας.

Διαβάσαμε με πολύ προσοχή το σύνολο της εργασίας του και η γνώμη που αποκομίσαμε είναι πως έχουμε να κάνουμε με πολύ ενδιαφέρουσες ιδέες. Η εργασία αυτή αποτελείται από 3 κεφάλαι-α.

Κεφάλαιο 1 ° : «Γεωμετρικές Κατασκευές» (με χάρακα και

διαβήτη, αλλά με προσεγγιστικές μεθόδους), Κεφάλαιο 2° : «Η Γεωμετρική Κατασκευή του Χωροχρονι­

κού Κώνου» (ανάπτυγμα της σειράς Fibonacci στον τρισδιάστατο χώρο),

του Διόφαντου». Ακόμη, είχε την ευγενή καλοσύνη να μας στείλει το πλήρες αγγλόφωνο κείμενο της εργασίας του με τίτλο «Α Framework for Defining the Generality of Diophantos ' Methods in "Arith­metica"»

Τον ευχαριστούμε θερμά. Η στήλη μας ευελπιστεί πως σύντομα θα έχει την άδεια κάποιων ομιλητών για τη δημοσίευση αρκετών εισηγήσεων από αυτό το συνέδριο.

Κεφάλαιο 3° : «Θεωρία για το πραγματικό σχήμα του Σύ­

μπαντος» .

Δυστυχώς, όμως, δεν μπορούμε να δημοσιεύ­σουμε κανένα από τα τρία κεφάλαια για δύο λό­γους: α) είναι πολυσέλιδα (η στήλη μας μόνο τε­τρασέλιδη και οι προς δημοσίευση εργασίες πολ­λές) , β) κανένα από τα τρία κεφάλαια δεν εμπίπτει στους σκοπούς του περιοδικού (το κεφάλαιο 1 ° , διαπραγματεύεται τις κατασκευές με «προσεγγι­στικές μεθόδους»).

Ευχαριστούμε τον κ. Τζεμπελίκο για την ευ­καιρία που μας έδωσε να δούμε κάποια γνωστά πράγματα από μια άλλη οπτική .

ΕΥΚΛΕΙΔΗΣ Β ' τ.2/27

Page 30: Ευκλειδης Β 62

•••ιιιι•ι••• ,.. ... �� .. �,, .... Αιι••Ι•ιι

Άλγεβρα Α • Τριώνυμο

Στο άρθρο που ακολουθεί υπάρχουν υποδειγμα­

τικές ασκήσεις σε θέματα που αφορούν το τριώνυ­

μο.

i . Δίνεται η εξίσωση χ2+ 10χ+ι!=Ο με λ ε R .(1) i) Υπολογίστε την διακρίνουσα της εξίσω­

σης. ii) Να βρείτε για ποιες τιμές του λ η εξίσω­

ση (I) έχει δύο πραγματικές και άνισες ρίζες.

iii) Για -5<λ<5 να βρεθεί η εξίσωση με ρί­ζες διπλάσιες της (1).

ΛίJση : i) Είναι Δ=β2-4αγ= 1 02-4λ2•

ii) Για να έχει η (Ι) δύο πραγματικές ρίζες πρέπει

να είναι Δ>Ο <=> 1 00-4λ2>0 <=> 4λ2< 1 00 <=>

<=> λ2<25 <=> lλl2<52 <=> lλl<5 <=> -5<λ<5

iii) Αφού -5<λ<5 η (Ι) έχει δύο πραγματικές άνι­

σες ρίζες χ 1 ,χ2 με x 1 +xz= - 1 0 και χ ι χz=λ2 •

Τότε S=2x 1+2x2=2(x 1+2xz)=2( - 1 0)= -20

και Ρ=2χ 12χz=4Χ ιΧzΑλ2 •

Άρα η εξίσωση με ρίζες 2χ ι , 2xz είναι η

χ2 -Sx+P=O δηλαδή η χ2+ 20χ+4λ2=0

2 . Δίνεται η εξίσωση χ2-93χ+2006=0. i) Να βρείτε τις ρίζες της παραπάνω εξίσωσης ii) Αν ρ1 η μικρότερη ρίζα της εξίσωσης να

βρείτε την τιμή του πραγματικού αριθμού λ έτσι ώστε η εξίσωση 25χ2+(ρι+ 16)χ -λ=Ο να έχει δύο ρίζες ίσες.

Τζορντανίδης Αναστάσιος

Λύση : i) Η εξίσωση χ2 -93χ+2006=0 έχει διακρίνουσα

Δ=932 - 4 · 2006=8643-8024=625 .

Άρα έχει ρίζες

= 93 - J625

= 93 - 25

= 68

= 34 ρ1 2 2 2

= 93 + J625

= 93 + 25

= 1 1 8

= 59 � 2 2 2

ii) Αφού ρ 1<ρ2 είναι ρ 1 =34. Τότε η εξίσωση

25χ2+(ρ 1 + 1 6)χ -λ=Ο γίνεται 25χ2+50χ-λ=Ο. Για να

έχει δύο ρίζες ίσες θα πρέπει να είναι

Δ=Ο <=> 502+4 · 25λ=Ο <=> 2500+ 1 ΟΟλ=Ο <=>

<=> 1 00λ= - 2500 <=> λ= -25 .

3 . Δίνεται η εξίσωση χ2-λχ-λ2-5=0, λε R , με ρίζες χι , χ2.

i) Να βρεθούν οι τιμές του λ έτσι ώστε να ισχύει η σχέση (χι-2)(χz-2)=-4.

ii) Για την μικρότερη τιμή του λ που βρή­κατε στο ερώτημα (i) να σχηματίσετε ε­ξίσωση με ρίζες τους αριθμούς 2χι+1 , 2χ2+1 .

λίJση : i) Είναι (χ ι-2)(χ2-2)= -4 <=> χ ι χz-2χ ι-2χz+4

= -4 <=> x 1 xz-2(x ι +Xz)+8=0 ( 1 ) όμως

χ 1 χ2=-λ2 -5 και χ ι+χz=λ άρα η ( 1 )

γίνεται -λ2-5-2λ+8=0 <=> -λ2-2λ+3=0 <=>

<=> λ2+2λ-3=0. Δ=4+ 1 2= 1 6 .

ΕΥΚΛΕΙΔΗΣ Β ' τ.2/28

Page 31: Ευκλειδης Β 62

--------------Μαθη ματικά για την Α ' Λυκείου --------------

Ά λ -2 + 4

1 ' λ -2 - 4

3 ρα - -- - η - -- - --2

- -2

- ο

ii) Για την τιμή λ= - 3 εξίσωση γίνεται

χ2+3χ- 1 4=0. Έχουμε τότε χ ι+χ2= - 3 και

χ ι χ2= - 1 4.

Είναι S=(2x 1+ 1 )+(2χ2+ 1 )=2(χ 1+χ2)+2= -6+2=

=-4 και Ρ=(2χι + 1 )(2χ2+ 1 )=4χ ι χ2+2χ ι+2χ2+ 1 =

=4χ ι χ2+2(χ ι+χ2)+ 1 =4( -1 4)+2( -3)+ 1 = -6 1 .

Άρα έχω την εξίσωση χ2+4χ -6 1 =Ο με ρίζες

2χ 1 + 1 , 2χ2+ 1 .

\ '

Δίνονται οι ευθείες με εξισώσεις ψ=(λ2 -2λ)χ -2007 και ψ=( 4λ -9)χ+ 2006 Να βρεθεί η τιμή του πραγματικού αριθμού λ, ώστε οι ευθείες να είναι παράλληλες.

Για να είναι οι ευθείες παράλληλες πρέπει να είναι λ2 -2λ=4λ-9 <=> λ2 -6λ+9=0

Δ=36 -36=0. Άρα η εξίσωση έχει μία λύση την 6 λ= - =3

2

'' Δίνεται η συνάρτηση f με τύπο f(x)=lx2-x-2 1 i) Να βρείτε τα σημεία τομής της γραφι­

κής παράστασης της f με την ευθεία ψ=4

ii) Να βρείτε τα σημεία τομής των γραφι­κών παραστάσεων των συναρτήσεων f και g με g(x)=lx2 -1 1

- ol

i) Για να βρούμε τα σημεία τομής της γραφικής

παράστασης της f με την ευθεία ψ=4 λύνουμε την

εξίσωση f(x)=4 <=> Ιχ2 -χ-2 1=4 <=> χ2-χ-2=4 (I ) ή

χ2 -χ-2= - 4 (Π) (Ι) <=> χ2-χ-6=0 Δ= 1 +24=25 . Άρα έχει ρίζες

1 + 5 1 - 5 χ = -- = 3 χ = -- = -2 I 2 ' 2 2

(Π) <=> χ2-χ+2=0. Δ= 1-8�7<0. Αδύνατη .

Επομένως τα σημεία τομής είναι τα Α(3 ,4) και

Β(-2,4) .

ii) Θα λύσουμε το σύστημα ψ=f(χ) και ψ=g(χ) δη­

λαδή f(x)=g(x) � Ιχ2-χ-2 Ι= Ιχ2- 1 1 �

� χ2-χ-2=χ2- 1 � -χ= 1 <=> χ=-1 .

ή χ2-χ-2�χ2+ 1 � 2χ2-χ-3=0. Δ= 1 +24=25

1 + 5 3 1 - 5 χ ι = -- = - χ = -- = - 1 .

4 2 , 2 4

Επομένως τέμνονται στα σημεία τους με τε­

τμημένες � και - 1 . Είναι g( � )=Ι � - 1 1= 2_ και g(-2 2 4 4

1 )=0 δηλαδή στα σημεία Μ(-1 , Ο) και Ν α, υ Να λυθεί η εξίσωση αχ2+βχ+γ=Ο, (α :;t: Ο) με 4α-2β+γ=Ο.

Είναι 4α -2β+γ=Ο <=> γ=2β -4α.

Άρα η εξίσωση γίνεται αχ2+βχ+2β -4γ=Ο.

Έχουμε

Δ=β2 -4α(2β-4α)=β2 -8αβ+ 1 6α2=(β-4α)2 .

Άρα η εξίσωση έχει ρίζες

χ _ -β + β - 4α

_ -4α

_ _ 2 1 - 2α -

2α -

'

-β - β + 4α -2β + 4α -γ χ 2 = = = - .

2α 2α 2α

Δίνεται η συνάρτηση f με f(χ) = αχ2 + βχ + γ

με γραφική παράσταση

3

ο 1 3

i) Να βρεθούν οι αριθμοί α, β, γ. ii) Με τη βοήθεια της γραφικής παράστα­

σης να λυθεί η ανίσωση f(x) � Ο.

Αφού η γραφική παράσταση της f τέμνει τον

ψψ ' στο σημείο (0,3) θα είναι

f(0)=3 <=> γ=3 . Η γραφική παράσταση τέμνει

τον χχ 'στα σημεία ( 1 ,0) και (3,0)

Άρα θα είναι

ΕΥΚΛΕΙΔΗΣ Β ' τ.2/29

Page 32: Ευκλειδης Β 62

--------------Μαθη ματικά για την Α' Λυκείου --------------

f(l) = Ο }α + β + γ = Ο }α + β + 3 = 0 } α + β = -3

f(3) = Ο 9α + 3β + 3 = Ο 9α + 3β + 3 = Ο 3α + β = - 1

Είναι α= 1 και β= --4 επομένως η f έχει τύπο

f(x)=x2 - 4χ +3 .

i i) Για να λύσουμε την f(x) :Ξ::: Ο βρίσκουμε το διά­

στημα που η γραφική παράσταση της f είναι κάτω

από τον άξονα χχ ' . Επομένως είναι το ( 1 ,3) .

1'J . Αν χι , χι ο ι ρίζες της εξίσωσης χι - (2α +l)x +α -1=0 (I) να βρεθεί

i) Για ποιες τιμές του α Ε IR. η (I) έχει ρίζες αντίθετες.

ii) Για ποιες τιμές του α Ε IR. η (I) έχει ρίζες aντίστροφες.

iii) Για ποιες τιμές του α Ε IR. είναι Χ ι - Χι = 3

i) Για να έχει η εξίσωση (Ι) ρίζες αντίθετες πρέ-1

πει χ 1 + χ2 =Ο � 2α + Ι =Ο � α= - -2 ii) Για να έχει η (Ι) ρίζες aντίστροφες πρέπει

χ ι χ2= Ι � α - Ι = Ι � α =2

iii) Έχουμε χ ι-χ2=3 και χ ι +χ2 =2 α + Ι άρα 2χ ι =2

α +4 � χ 1 = α + 2 τότε

χ2 = 2α + 1 -χ ι = 2 α + Ι -α - 2 = α -Ι όμως χ ι χ2 =α

- Ι � ( α +2)(α - Ι ) = α - Ι �

� (α +2)(α - 1 ) - (α - Ι )= Ο �

� (α - Ι )( α + 2 - Ι )=Ο � (α - 1 )(α + Ι )= Ο �

� α = Ι ή α = - Ι .

Ο! Δίνεται η εξίσωση αχι+βχ+γ=Ο. Αν οι αριθ­μοί α, γ είναι ετερόσημοι να δείξετε ότι η ε­ξίσωση έχει δύο ρίζες ετερόσημες.

λ(Jση : Είναι Δ=β2--4αγ. Όμως αγ<Ο αφού α, γ ετερό­

σημοι αριθμοί.

Τότε --4αγ>Ο και επομένως Δ>Ο.

Άρα η εξίσωση έχει δύο πραγματικές και άνι-

σες ρίζες x 1 x2= L <0. Άρα Χ ι ,χ2 ετερόσημοι. α

1 11 Να βρείτε τα α,β Ε IR. ώστε η εξίσωση 3χι-(α+β-1)χ+2β-5=0 να έχει διπλή ρίζα τον αριθμό 1 .

. \ {Jση : .::.ερουμε ότι για να έχει η εξίσωση

αχ2+βχ+γ=Ο διπλή ρίζα πρέπει Δ=Ο και τότε η ρίζα

' β

ειναι η ρ= -- . 2α

α + β - Ι Άρα πρέπει 1 και Δ=Ο

6

Είναι 6=α+β-1 � α=7-β.

Τότε η εξίσωση γίνεται

3χ2- (7-β+β-Ι )χ+2β-5=0 � 3χ2-6χ+2β-5=0

Πρέπει Δ=Ο � 36- Ι 2(2β-5)=0 �

� 36-24β+60=0 � 24β=96 � β=4

Άρα α=7--4=3

J '' . Αν για τους πραγματικούς αριθμούς α,β ι­σχύει αι+βι -4α+6β+ 13=0 να λυθεί η εξίσω­ση αχι+(2α+β)χ+β=Ο.

Είναι α2+β2 --4α+6β+ Ι 3=0 �

� α2 --4α+4+β2+6β+9=0 � ( α-2)2+(β+ 3 )2=0

Άρα πρέπει α-2=0 � α=2 και β+ 3=0 � β=-3

Η εξίσωση αχ2+(2 α +β)χ+β=Ο τότε γίνεται

2χ2+χ-3=0. - 1 + 5

Επομένως έχει ρίζες χ 1 = -- = 1 , 4 - 1 - 5 -6 -3

χ2= -- = - = - . 4 4 2 R 2 . Να λυθεί η εξίσωση χι - α Ιχ Ι -2=0 όταν για

τον πραγματικό αριθμό α ισχύει Ι αι + α - 2 Ι + Ι α ιοο6 - 1 Ι = ο.

Λύση : Έχουμε I α2 + α - 2 I + I α 2006 - Ι I = Ο �

και <=>και Λύνουμε την εξίσωση

lα2 +α-21 =0} α2 + α-2 = 0) Ια2006 _ 11 = 0 α2006 _ 1 = 0

- Ι + 3 α 2 + α -2 = Ο Είναι Δ= Ι +8=9 . Άρα α = -- = Ι

2 - Ι - 3

η α = -- = -2 2

Λύνουμε την α 2006 - Ι = Ο � α 2

006 = Ι �

� α = ±2οο.(Ι = ± 1

ΕΥΚΛΕΙΔΗΣ Β ' τ.2/30

Page 33: Ευκλειδης Β 62

--------------Μαθη ματικά για την Α ' Λυκείου --------------

Επομένως η κοινή λύση των δύο εξισώσεων

είναι η α = 1 . Τότε η εξίσωση χ2 - α Ιχ Ι -2=0

γίνεται χ2 - Ιχ Ι -2=0 <=> I χ Ι 2 -1 χΙ -2 = Ο,

θέτω Ι χ Ι =ω � Ο και η εξίσωση γίνεται

ω2 - ω -2 = Ο Δ= 1 +8 = 9 '

ω = 1 + 3

= 2 δεκτη' ω = 1 - 3

= - 1 I 2 ' 2 2

απορρίπτεται, άρα Ι χ Ι = 2 <=> χ = 2 ή χ = - 2 .

1 3 . Αν ρι , ρ2 ο ι ρίζες της εξίσωσης α χ 2 +β χ + γ = Ο και ρ ι = 2 ρ2 να λυθεί η ε­ξίσωση 2αχ2 - 4βχ +9γ = Ο.

\ ίJση ; Είναι ρ 1 ,

άρα θα

ρ2 ρίζες της εξίσωσης αχ2+βχ+γ=Ο β

έχουμε ρ1 +� =- <=> 2ρ2 α

β β β +ρ2 = - - <=> 3ρ2 = - - <=> ρ2 = - -

α α 3α

τ , 2 2β

οτε ρ 1 = ρ2= --3α

Όμως Ρ 1 Ρ2 = 1_ <=> (- 2β J(-l__J = 1_ <=> α 3α 3α α

2β2 2β2 - = l <::::> - = γ <::::> 2β2 = 9αγ (I) 9α2 α 9α

Η εξίσωση 2αχ2- 4βχ +9γ =Ο έχει ( I )

Δ = 1 6β2 - 4 · 2α · 9γ = 1 6β2 - 72αγ =

8(9αγ) - 72αγ = 72αγ - 72αγ = Ο

Άρα η εξίσωση έχει μία διπλή ρίζα την ,ι(β β

χ = - = -

Jα α.

Β . Η έννοια της συνάρτη σης Βρέκας Χάρης

• Αν η συνάρτηση f δίνεται : "f: Α � R", τότε το πεδίο ορισμού της είναι το σύνολο Α. :ιι Αν η συνάρτηση δίνεται μόνο με τον τύπο της f(χ) τότε το πεδίο ορισμού της είναι το ευρύτερο

υποσύνολο του R για το οποίο έχει νόημα πραγματικού αριθμού η παράσταση f(χ).

Ασκήσεις n ) Να βρεθεί το πεδίο ορισμού της συνάρτη- που προκύπτει από ενδεχόμενες aπλοποιήσεις.

χ2 - 4χ + 3 σης: f(χ) 2 και στη συνέχεια να 2 ) Να βρεθεί το πεδίο ορισμού των συναρτή-χ - χ απλοποιηθεί ο τύπος της.

Α.πάντηση : • Η f(χ) έχει νόημα πραγματικού αριθμού όταν:

x 2 - x :;t: O <::::> x (x - 1) :;t: O <::::> (x :;t: O και x :;t: 1)

Άρα το πεδίο ορισμού της συνάρτησης f είναι

το σύνολο

Α= R - {0, 1 } = (-οο, Ο) U (0, 1) U ( l , +oo) .

e Για τα χ Ε Α είναι:

f(χ)= �(χ - 3) <::::> f(x) =

χ - 3

χ� χ

l:zόλιο Πρέπει να προσέξουμε ότι η εύρεση του Α, αφορά τον αρχικό δοσμένο τύπο και όχι αυτόν

σεων:

(i)

(ii)

(iii)

f(x) = χ + �2 - .J3 + χ2

f(x) = � χ2 + 1

f(x) = �2 - l x + 31

Απάντηση : (i) Πρέπει 3 + χ 2 � Ο , που ισχύει για κάθε

χ Ε R και ακόμη :

2 - .J3 + x2 � 0 <::::> 2 � .J3 + x2 <::::> 4 � 3 + χ2 <::::> 1 � χ2

<=> lx l 2 ::;; 12 <=> lxl:� 1 <::::> - 1 ::;; χ ::;; 1

Άρα το πεδίο ορισμού της f είναι το σύνολο :

Α=[-1 , 1 ]

ΕΥΚΛΕ ΙΔΗΣ Β ' τ.2/3 1

Page 34: Ευκλειδης Β 62

-------------Μ αθηματικά για την Α ' Λυκείου-------------

(ii) Πρέπει: l x l -χ � Ο και χ2 + 1 :;t: Ο Ξέρουμε ότι: l xl � χ <=>lxl -χ � Ο, για κάθε χ ε R . Ακόμη χ 2 + 1 :;t: Ο, για κάθε χ Ε R . Έτσι το πεδίο ορισμού της f είναι το σύνολο A= R

(iii) Πρέπει: 2 - Ι χ + 3 1 � Ο <::::> l x + 3 1 ::; 2 <::::> -2 ::; χ + 3 ::; 2 <::::> -5 ::; χ ::; - 1 Άρα το πεδίο ορισμού της f είναι το σύνολο Α=[-5 ,- 1 ] Για να βρούμε την τιμή της συνάρτησης f στο α, δηλαδή το f(α) , θέτουμε στον τύπο της όπου χ το α και κάνουμε πράξεις. Δίνεται η συνάρτηση f(x) = χ2 + 1 Να βρεθούν τα f(-2), f(2), f( J3 ), f(-J3 ). Τι

παρατηρείτε;

Είναι φανερό ότι το πεδίο ορισμού της f είναι το Α=R.

Έχουμε ότι f(-2)=(-2)2+ 1=4+ 1 =5 f(2)= 22+ 1 =4+ 1 =5 f( J3 )=( J3 )2+ 1 =3+ 1 =4 f(-J3 )=(-J3 )+ 1 =3+ 1 =4.

Παρατηρούμε ότι: f( -2)=f(2) και f( J3 )=f(-J3 ) . Δηλαδή τα διαφορετικά στοιχεία: -2, 2 του πεδίου ορισμού της f έχουν την ίδια τιμή : 5 .

Επίσης τα διαφορετικά στοιχεία -J3 , J3 έ­χουν την ίδια τιμή 4.

Δίνεται η συνάρτηση :

f(x) = J,...--9 --4-J;.=x (i) Να βρεθεί το πεδίο ορισμού της (ii) Να βρεθούν τα: f(O), f(4), f(5).

,. ·;, ,• l • i

(i) Πρέπει (χ � Ο και 9 - 4-Fx � Ο) <=>

<=> (χ � Ο και 9 � 4Fx) <=>

<=> (χ � Ο και 8 1 � 1 6χ) <=>

8 1 <=> (χ � Ο και χ ::;-) 1 6

Άρα το πεδίο ορισμού της f είναι το : Α=[Ο, � ] 1 6

(ii) f(O) = J9 - 4.J0 = .J9 = 3 f(4) = J9 - 4J4 = � =JΪ = 1 f(5) = J9 - 4.J5 = J5 + 4 - 2 . 2-JS = = �22 + <J5>2 - 2 - 2JS = �<2 -# = 1 2 - JSI = JS - 2 Δίνεται η συνάρτηση f(χ)= �

4 - χ2 (i) Να βρεθεί το πεδίο ορισμού Α της f. (ii) Α ν α ε Α , να δείξετε ότι:

-α ε Α και f(-α) = -f(α)

(i) : Πρέπει: 4 - χ2 � Ο <::::> -χ2 � -4 <::::> χ2 ::; 4 <::::>

<::::> l x l2 ::; 22 <::::> l x l ::; 2 <::::> -2 ::; χ ::; 2

άρα Α=[-2,2] (ii) : Αφού α Ε Α , θα έχουμε ότι:

( - 1 ) ( 0 -2 ::; α ::; 2 <::::> 2 � -α � -2 <::::> -2 ::; -α ::; 2 <::::> -α ε Α ακόμη :

f(-α) = (-α) Ι-αl �4 - (-α)2

(-α) iα l = - α iαl = -f(α) .J4 - α2 .J4 - α2

Μια τέτοια συνάρτηση , όπως η παραπάνω, λέγεται Περιττή Εύκολα μπορούμε να διαπιστώσουμε ότι η γραφική παράστασή της είναι συμμετρική ως προς την αρχή των αξόνων. Να βρεθεί το πεδίο ορισμού και το σύνολο

τιμών της συνάρτησης:

f(x) = .Jx2 - 4 + .j4 - χ2

Πρέπει: (χ2--4�0 και 4-χ2�0)<::>(χ2�4 και χ2:::;4) <::::> χ2=4<::>(χ=2 ή χ=--2) επομένως το πεδίο ορισμού είναι το : Α={-2,2} .

Είναι f(-2)=0 και f(2)=0 Άρα το σύνολο τιμών της f είναι το : f(A)={O} .

Δίνεται η συνάρτηση

f( χ) = 5 + ..jr-�-2 --4χ_+_4 . χ - 2

ΕΥΚΛΕΙΔΗΣ Β ' τ.2/32

Page 35: Ευκλειδης Β 62

--------------Μαθηματικά για την Α ' Λυκείου --------------

(ί) Βρείτε το πεδίο ορισμού της.

(ίί) Απλοποιείστε τον τύπο της.

(ί) Πρέπει (x:;t:2 και x2-4x+4�0)<=>(x:;t:2) και

(χ-2)2�0) όμως η σχέση (χ-2)2 �0 ισχύει για κάθε

πραγματικό αριθμό άρα το πεδίο ορισμού Α της f

είναι:

A=R- {2 } =(-oo,2)u(2, οο)

(ίί) Για χ ε Α έχουμε :

f(x) = 5 + �(χ - 2)2 = 5 )χ - 2 1

( 1 ) χ - 2 χ - 2

Π_ρ

όσ:ιΊ}:tΟ �l]� _παράστασ% Χ:-::� Ιι<Χ !;:!"�: _

χ -00

χ-2

Ιχ-2 1 -χ+2

2 +οο

ο +

ο χ-2

έτσι από τη σχέση ( 1 ) έχουμε : -χ + 2

f(x)=5+ -- = 5 - 1 = 4 , για χ(2 χ - 2

χ - 2 και f(x)=5+ -- = 5 + 1 = 6 , για χ>2

χ - 2 {4, χ < 2 δηλαδή : f(x) =

6, χ > 2

" i Δίνεται η συνάρτηση : f(x)=x2-4. (ί) Να βρεθούν τα σημεία τομης της Cr με

τους άξονες χ'χ , ψ ' ψ.

(ίί) Ν α εξετάσετε αν τα σημεία Α( -3, 5), Β(1 ,-2) ανήκουν στην Cr.

Είναι φανερό ότι το πεδίο ορισμού της f είναι

το Ar =R.

(ί) Επειδή Ο ε Ar , η Cr τέμνει τον ψ 'ψ. Επίσης

είναι f(0)=02-4 = -4.

Άρα το σημείο τομής της Cr με τον ψ ' ψ είναι

το Μ(Ο,-4) Για να βρούμε τα κοινά σημεία της Cr με τον

χ'χ , λύνουμε την εξίσωση :

f(x)=O. f(x)=O <=> χ2-4=0 <=> χ2 =4 <::::>χ=--2 ή χ=2 .

ι.: 5of[-3J

χ

Άρα η Cr τέμνει τον άξονα χ' χ στα σημεία:

Ν(-2,0) και Κ(2,0).

(ίί) Είναι f(-3)=(-3)2-4=9-4=5, οπότε το ση­

μείο A ε Cr.

Είναι f( l )= 1 2-4= 1-4 = -3:;t:-2, οπότε το ση­

μείο Β � Cr (σχήμα) .

Δίνεται η συνάρτηση {2χ - 1, αν - 2 � χ < 1 I 2 f(x)= χ2 + 1, αν 1 Ι 2 � χ < 3

Να βρεθούν (αν υπάρχουν) τα κοινά σημεία της Cr με τους άξονες χ 'χ , ψ' ψ.

'I' ! 1

I 1 11 i μ.1 : 1 )

Πεδίο ορισμού της συνάρτησης είναι το σύνο­

λο A=[-2, 1 12)u [ l /2, 3 ]=[-2,3 ] .

Το Ο ε [-2,3] = Α και f(0)=2 ·0 - 1 =-- 1 . Άρα η

CΓτέμνει τον ψ 'ψ στο σημείο Μ(Ο,- 1 ) .

"' Για χε [-2, 1 12), f(x)=O <=> 2χ- 1 =0 <=> χ= 1 12

άτοπο.

Για χ ε [ 1 12 , 3] , f(x)=O<::::>x2+ 1 =0<::::>x2=- 1 , άτοπο .

Δηλαδή για κάθε χε Α, f(x):;t:O. Άρα η Cr δεν

τέμνει τον χ 'χ . (σχήμα)

ΕΥΚΛΕΙΔΗΣ Β' τ.2/33

Page 36: Ευκλειδης Β 62

--------------Μ αθηματικά για την Α ' Λυκείου --------------

10 ) Δίνεται η συνάρτηση f(x)=3 l x l -6

Να βρεθούν τα διαστήματα στα οποία η γραφική παράσταση της f βρίσκεται πάνω από τον χ 'χ. Απά\•τη ση : Το πεδίο ορισμού είναι όλο το R.

Θα βρούμε εκείνα τα χεR, ώστε f(χ)>Ο.

f(x)>O � 3 lx l -6>0�3 lx l >6� lx l >2�(χ>2 ή χ(-2)

Έτσι η Cr βρίσκεται πάνω από τον άξονα χ'χ

όταν χε (--οο,-2)u(2 , +οο) (σχήμα)

ψ

-2 ο

-6

Γεωμετρία

Α . Παραλληλόγραμμα Πλιάτσιος Ανδρέας - Ντίνας Βασίλειος,

Κοζάνη «Μια περιήγηση στις παράλληλες ευθείες και τα παραλληλόγραμμα για τον μαθητή της Α ' Λυκείου μέσα από μια σειρά θεμάτων που χαρακτηρίζονται για την απλότητά τους, αλλά και για την πρωτοwπία τους. »

Θ E l\I A 1 Τρεις ευθείες Οχ ,Οψ ,Oz σχηματίζουν με­

ταξύ τους γωνίες χ0ψ=ψ0z=60° . Από τυχαίο σημείο Μ που βρίσκεται στο εσωτερικό της χΟψ φέρουμε τις κάθετες ΜΑ ,ΜΒ ,ΜΓ προς τις Οχ ,Οψ ,Oz αντίστοιχα. Κατόπιν προεκτεί­νουμε την ΜΑ η οποία τέμνει την Οψ στο Δ. Από το Δ φέρνω την ΔΑ ' 1.. Oz και η προέκταση της ΜΒ τέμνει την ΔΑ ' στο ΜΌ

Να δειχθεί ότι: i) ΔΑ '=ΔΑ ii) Το τρίγωνο ΔΜ'Μ είναι ισόπλευρο iii) ΜΑ+ΜΒ=ΜΓ

z

Μ '

Α '

Γ

ο Λί1 ση

Ν Δ

Μ

Α

ψ

χ

i) Αφού χΟψ = ψΟz = 60° τότε η Οψ είναι διχο-

τόμος της χΟz και ΔΑ, ΔΑ ' οι αποστάσεις του Λ

σημείου Δ από τις πλευρές της χΟz άρα ΔΑ=ΔΑ ' ii) Είναι ΑΊiΟ=Μ0=30° ως συμπληρωματικές

των 60° στα ορθογώνια τρίγωνα ΟΑ ' Δ και ΟΔΑ. Στο τρίγωνο ΔΜΜ ' η ΔΒ είναι ύψος και διχο­

τόμος, άρα το τρίγωνο είναι ισοσκελές και επειδή

μία του γωνία η ΜΊ�Μ=60° άρα θα είναι ισό­πλευρο . iii) Φέρνω το ύψος ΜΝ του τριγώνου ΜΜ 'Δ τότε το τετράπλευρο Γ Α 'ΝΜ είναι ορθογώνιο παραλ­ληλόγραμμο (τρεις ορθές) άρα ΜΓ=ΝΜ '+ΜΆ' αλλά ΝΜ'=ΒΜ σαν μισά ίσων πλευρών ΔΜ ' και ΜΜ' του ισοπλεύρου τριγώνου ΔΜ 'Μ.

ΜΆ'=ΜΑ ως υπόλοιπα των ίσων πλευρών ΔΑ'=ΔΑ.

Οπότε ΜΒ+ΜΑ=ΝΜ '+ΜΆ'=ΝΑ '=ΜΓ. Θ F: !\/Ι Λ 2

Δίνεται γωνία χΟψ πάνω στην πλευρά της Οχ παίρνουμε δύο τυχαία σημεία Α και Α ' και πάνω στην πλευρά της Οψ δύο άλλα σημεία Β και Β ' έτσι ώστε ΑΑ'=ΒΒ ' . Από το Α' φέρουμε την παράλληλη προς την ΑΒ και από το Β την παράλληλη προς την Οχ , οι οποίες τέμνονται στο Γ. Έστω ακόμη Δ, Ε και Ε ' τα μέσα των τμημάτων ΑΒ, ΓΒ ' και Α'Β ' αντίστοιχα. Να δειχθεί ότι : i) Το τετράπλευρο ΑΑΤΒ είναι παραλληλό-

γραμμο. ii) Το τρίγωνο ΒΒ Τ είναι ισοσκελές iii) Το ΕΒΔΕ ' είναι παραλληλόγραμμο. ίν) Η ευθεία ΕΕ ' είναι παράλληλη της διχοτό-

Λ

μου Οδ της γωνίας χΟψ ΕΥΚΛ ΕΙΔΗΣ Β' τ.2/34

Page 37: Ευκλειδης Β 62

--------------Μαθη ματικά για την Α ' Λυκείου --------------

ψ

ο δ '

Α δ Β Ε

Α' Ε ' Β ' Δ

Χ Γ ψ i) Είναι ΑΑ ΤΒ παραλληλόγραμμο γιατί οι απένα­ντί πλευρές του είναι παράλληλες ii) Είναι ΒΓ=ΑΑ ' (επειδή το ΑΑ ΤΒ είναι παραλ­ληλόγραμμο) και ΒΒ '=ΑΑ ' (από υπόθεση) άρα ΒΓ=ΒΒ ' δηλαδή ΒΓΒ ' ισοσκελές. iii) Αφού τα Ε ' και Δ είναι μέσα των πλευρών Α'Β ' και ΓΒ ' αντίστοιχα, τότε στο τρίγωνο Α 'Β Τ

έχουμε: Ε 'Δ= 1 1 ΑΤ =1 1 ΑΒ =1 1 ΕΒ άρα Ε 'Δ= 1 1 ΕΒ 2 2 δηλαδή το τετράπλευρο ΕΒΔΕ ' είναι παραλληλό­γραμμο .

iv) Είναι χΟψ = ΓΙ3ψ ως εντός εκτός και επί τα

αυτά μέρη των παραλλήλων Οχ , ΒΓ που τέμνονται από την Οψ οπότε

' ' , , χΟψ ΓΒψ , , χΟψ = ΓΒψ ή -- = -- ή Οι = Β ι 2 2

Επομένως ΒΔ//Οδ. Αλλά ΒΔ//ΕΕ ' άρα ΕΕ Ί/Οδ όπου Οδ η διχοτόμος της γωνίας

χΟψ .

Από τα άκρα Α, Β του ευθυγράμμου τμήμα­τος ΑΒ φέρνω προς το ίδιο μέρος ημιευθείες Αχ 1 1 Βψ και παίρνω Γ τυχαίο σημείο της ΑΒ . Πάνω στις Αχ και Βψ παίρνω ΑΕ=ΑΓ και ΒΖ=ΒΓ αντίστοιχα . Αν Μ μέσο της ΕΖ και οι προεκτάσεις των ΑΜ και ΒΜ τέμνουν τις Βψ και Αχ στο Η και Θ αντίστοιχα , να δειχθεί ότι : i) Τα τρίγωνα ΑΕΜ και ΗΜΖ είναι ίσα. ii) Το τετράπλευρο ΑΒΗΘ είναι ρόμβος.

Γ

λ ιJση

χ; ' Ψ I I I

- - � I

Β

i) Είναι Α ι = Η ι (ως εντός εναλλάξ) και

Μ ι =Μ2 (ως κατακορυφή) επομένως ΑΕΜ = 1 80° - λ - Μ ι και

MZH = l 80° - Μ2 - Μι άρα ΑΕΜ = ΜΖΗ . Για τα τρίγωνα ΑΕΜ και ΗΜΖ έχουμε:

Μι = Μ2 , ΑΕΜ = ΜΖΗ , Μ μέσο της ΕΖ επο­μένως είναι ίσα.

Δ Δ i i) Όμοια είναι ΕΜΘ = ΜΖΒ , οπότε από τις ισότητες των τριγώνων, έχουμε ότι οι διαγώνι­ες του τετραπλεύρου ΑΒΗΘ διχοτομούνται, άρα είναι παραλληλόγραμμο . Επίσης είναι ΒΗ=ΒΖ+ΖΗ=ΒΓ+ΑΕ=ΒΓ+ΓΑ=ΒΑ, δηλαδή το ΑΒΗΘ είναι ρόμβος. Θ f:<: \ 1 .\ 4

Δίνεται ορθογώνιο τρίγωνο ΑΒΓ (Α = 90° ) . Φέρουμε το ύψος ΑΗ και παίρνουμε τα Δ και Ε συμμετρικά του Η ως προς τις πλευρές ΑΒ και ΑΓ. Να δειχθεί ότι : i) Τα σημεία Δ , Α , Ε είναι συνευθειακά . ii) ΒΔ 1 1 ΕΓ iii) ΑΔ=ΑΗ=ΑΕ

Δ

Α

Γ Η

Β

.\ {Jση

Ε

i) Αφού τα Δ και Ε είναι σημεία συμμετρικά του Η ως προς τις ΑΒ και ΑΓ αντίστοιχα, από τις ισότητες των τριγώνων ΑΔΚ και ΚΑΗ έχουμε ότι

Αι = Α2 και από τις ισότητες των τριγώνων ΗΑΜ ' ' ' ' ο και ΑΜΕ έχουμε Α3 = Α4 όμως Α2 + Α3 = 90

' ' ' ' ' ο οποτε Αι + Α2 + Α3 + Α4 = 1 80 άρα τα σημεία Δ,

Α και Ε είναι συνευθειακά. Δ Δ

ii) ΒΔΑ = ΒΗΑ λόγω συμμετρίας άρα ' ' ΒΔΑ = ΑΗΒ ομοίως ΑΗΓ = ΑΕΓ άρα

ΒΔΑ + ΓΕΑ = 90° + 90° = 1 80° άρα ΒΔ 1 1 ΕΓ iii) Είναι ΑΔ=ΑΗ και ΑΗ=ΑΕ από τις ισότητες των παραπάνω τριγώνων άρα ΑΔ=ΑΗ=ΑΕ

tH: �, i .\ 5 Σε παραλληλόγραμμο ΑΒΓΔ είναι ΑΒ=2ΒΓ.

Φέρνουμε την ΑΕ κάθετη στην ΒΓ και παίρ­νουμε τα μέσα Μ, Ν των ΑΒ , Γ Δ αντίστοιχα. Να δειχθεί ότι : i) Το τρίγωνο ΜΕΝ είναι ισοσκελές . ii) Η ΝΕ είναι διχοτόμος της Μ ΕΓ

� � iii) ΔΝΕ = 3ΝΕΓ ΕΥΚΛΕΙΔΗΣ Β ' τ.2/35

Page 38: Ευκλειδης Β 62

--------------Μαθηματικά για την Α ' Λυκείου --------------

Δ Ν /' Γ

Α Μ 1 Β

i) Είναι ΑΒ=I IΓΔ ή ΑΒ

=1 1 ΓΔ

ή MB=I INΓ συνε-2 2

πώς το τετράπλευρο ΜΒΓΝ είναι παραλληλό­γραμμο, επομένως ΒΓ=ΜΝ Στο ορθογώνιο τρίγω­νο ΑΕΒ η ΕΜ είναι διάμεσος άρα

ΑΒ 2ΒΓ , ΕΜ = - = -- = ΒΓ = ΜΝ δηλαδη το ΜΕΝ

2 2 είναι ισοσκελές

ii) Είναι ΜΝ 1 1 ΓΕ άρα Ν ι = Ε2 (ως εντός εναλ­

λάξ) και Ν ι = Ε ι (ΜΕΝ ισοσκελές τρίγωνο) άρα

Ει = Ε2 δηλαδή η ΝΕ είναι διχοτόμος της γωνίας

ΜΕΓ .

Επειδή ΜΕ = ΑΒ

= ΜΒ τότε το τρίγωνο 2

ΕΜΒ είναι ισοσκελές άρα ΜΕΒ = Β , ή 2Ε2 = Β ι

ή 2Ε2 = Γ . Αλλά

ΔΝΕ = ΔΝΜ + Νι = f + E2 = 2Ε2 +

Ε2 = 3Ε

2 = 3ΝΕΓ

Δίνεται τετράπλευρο ΑΒΓ Δ με κέντρο Ο. Α ν Ε είναι το μέσο της ΟΓ, Μ το μέσο της ΟΔ και

Ζ σημείο της ΟΔ τέτοιο ώστε ΟΖ = .!.οΔ . Να 4

δειχθεί ότι Δ Δ

ί) Τα τρίγωνα ΟΜΓ και ΟΕΒ είναι ίσα. Δ

ίί) Το τρίγωνο Ε Β Ζ είναι ορθογώνιο.

Δ Δ

Δ Γ

Α

i) ΟΜΓ = ΟΕΒ γιατί: α) είναι ορθογώνια β) ΟΓ=ΟΒ

, ΟΔ ΟΓ γ) ΟΜ=ΟΕ αφου ΟΜ = - , ΟΕ = - και ΟΔ = ΟΓ

2 2

Δ Λ Λ Λ

ii) Στο ΕΒΖ ει ναι ΖΕΒ = Ε, + Ε2 ( Ι ) Δ

Στο ΟΜΓ είναι Ζ, Ε μέσα , άρα ΖΕ 1 1 ΓΜ οπότε

Μ ι = Ζ 1 (2) Λ Λ Δ Δ

Όμως Μ ι = Ει (3) αφού ΟΜΓ = ΟΕΒ ,

από την (2), (3) προκύπτει Ζ ι = Ε ι όμως

Ζι + Ε2 = 90° ή Ε ι + Ε2 = 90° άρα ΖΕΒ = 90°

συνεπώς το τρίγωνο ΕΖΒ είναι ορθογώνιο .

Δίνεται ορθογώνιο τρίγωνο ΑΒΓ στο οποίο η γωνία Β = 30° . Στο μέσο Ο της ΒΓ φέρνουμε κάθετη στη ΒΓ που τέμνει την ΑΒ στο Ζ και την ΑΓ στο Ε . Επίσης φέρνουμε στο σημείο Α κά­θετη στην ΑΟ που τέμνει την ΕΖ στο Η . Να δειχθεί ότι : i) Τα τρίγωνα ΑΟΓ και ΑΗΖ είναι ισόπλευρα . ίί) ΕΗ=ΗΖ.

Β

. · , i l ' ! .

Ε

Η

2 z .

ο

2

Γ

i) Το τρίγωνο ΑΒΓ είναι ορθογώνιο και Β = 30°

άρα ΑΓ = ΒΓ

( 1 ) 2

Η Αο δ ' ' ΑΟ --

ΒΓ (2) ' ιαμεσος τοτε απο τις

2 Δ

( 1 ), (2) έχουμε ΑΓ=ΑΟ άρα ΑΟΓ ισοσκελές όμως Δ

Γ = 60° άρα το ΑΟΓ είναι ισόπλευρο.

Είναι Α2 = 60° οπότε Α ι = 30° και Α3 = 60°

Δ αφού ΑΗ .l ΑΟ . Στο ορθογώνιο τρίγωνο ΖΟΒ

Λ Λ Λ ο είναι Β = 30° και Ζ ι = 60° άρα Ζ2 = 60 ( αφού

Λ Λ Δ Ζι = Ζ2 ως κατακορυφήν) άρα το ΑΗΖ είναι ι-

σόπλευρο. ii) Επειδή το τρίγωνο ΗΑΖ είναι ισόπλευρο έχουμε

ότι Α3 = 60° άρα Α4 = 30° . Στο ορθογώνιο τρί-

γωνο ΕΟΓ είναι Γ = 60° επομένως Ε = 30° . Τελι­

κά Α4 = Ε = 30° δηλαδή το τρίγωνο ΕΗΑ είναι

ισοσκελές Επομένως ΕΗ=ΗΑ=ΗΖ.

ΕΥΚΛΕΙΔΗΣ Β ' τ.2/36

Page 39: Ευκλειδης Β 62

--------------Μαθη ματικά για την Α ' Λυκείου --------------

l. ,· · . ' Α

Δίνεται γωνία χΟψ . Στην Οχ παίρνουμε δι-αδοχικά τα τμήματα ΟΑ και ΑΑ ' και στην Οψ τα τμήματα ΟΒ και ΒΒ ', ώστε ΒΒ '=ΑΑ', (ΟΑ ;;�; ΟΒ) . Να δειχθεί ότι η ευθεία που συνδέ-ει τα μέσα των ΑΒ και Α 'Β ' είναι παράλληλη στην διχοτόμο της γωνίας χΟψ .

Έστω Οδ η διχοτόμος της γωνίας χΟψ . Φέρ­

νουμε Al .l Οδ που τέμνει την Οψ στο Γ, οπότε το τρίγωνο ΑΟΓ είναι ισοσκελές γιατί η ΟΙ είναι δι­χοτόμος και ύψος. Στο τρίγωνο ΑΓΒ τα I , Ε, είναι

μέσα πλευρών τότε ΙΕ = 1 1 ΒΓ ( 1 ) .

2

Α

Α '

χ

ο

ι Γ Ε

Ε ' δ

, ,

Β Δ

Β ' ψ

Όμοια αν Α Τ l_ Οδ, το τρίγωνο ΟΑ ' Δ είναι

ισοσκελές και Ι 'Ε · = 1 1 ΔΒ . (2) .

2 Αλλά ΓΔ=ΟΔ-ΟΓ=ΟΑ '-ΟΑ=ΑΑ '=ΒΒ ' τότε

τα τμήματα ΒΓ και ΔΒ ' είναι ίσα ως διαφορές ί­σων ευθ . Τμημάτων και από τις σχέσεις ( 1 ) και (2) έχουμε ΙΕ=Ι Ι Ι Έ 'δηλαδή το τετράπλευρο ΙΕΕ Τ εί­ναι παραλληλόγραμμων επομένως ΕΕ ' //Οδ.

Δίνεται τρίγωνο ΑΒΓ . Πάνω στις πλευρές ΑΒ και Γ Δ παίρνουμε σημεία Δ και Ε έτσι ώστε ΒΔ=ΓΕ. Αν Μ είναι το μέσο της ΒΓ και Ν το μέσο της ΔΕ , να δειχθεί ότι : ί) Το τετράπλευρο ΝΚΜΛ είναι ρόμβος , όπου

Κ μέσο της ΒΕ και Λ μέσο της ΔΓ ii) Η ΜΝ είναι παράλληλη με τη διχοτόμο της

Λ

γωνίας Α . Α

Δ

κ

Β Μ Γ i) Στο τρίγωνο ΔΓΒ τα Λ, Μ είναι μέσα των ΔΓ,

ΒΓ άρα ΛΜ = 1 1 ΒΓ ( 1 )

2

Στο τρίγωνο ΔΕΒ τα Κ, Ν είναι μέσα των ΒΕ,

ΔΕ άρα ΚΝ = 1 1 ΒΔ (2)

2 Από την ( 1 ), (2) έχουμε ότι ΛΜ =1 1 ΚΝ οπότε

το ΝΚΜΛ είναι παραλληλόγραμμο. ΕΓ

Ακόμη στο τρίγωνο ΔΕΓ είναι ΝΑ = - και 2

στο ΔΓΒ είναι ΛΜ = ΒΔ

, αφού ΕΓ=ΒΔ τότε 2

ΝΛ=ΛΜ δηλαδή το παραλληλόγραμμο ΝΚΜΛ εί­ναι ρόμβος. ii) Επειδή ΝΚΜΛ είναι ρόμβος η διαγώνιος ΝΜ

είναι και διχοτόμος της ΚΝΛ , όμως ΚΝΛ = Α γιατί είναι οξείες και έχουν τις πλευρές τους πα­ράλληλες. Άρα και οι διχοτόμοι τους θα είναι πα­ράλληλες.

.. .,. �-, ' -. ,, , · ί ·

Δίνεται το ισοσκελές τρίγωνο ΑΒΓ με ΑΒ=ΑΓ και με ύψος ΑΔ. Το Μ είναι μέσο της

ΑΓ και είναι ΜΕ=ΜΖ, ME.l ΑΓ και ΖΛ 1 1 ΑΒ . Να δειχθεί ότι ί) Το ΑΖΓΕ είναι ρόμβος. ii) ίίί) ΖΝ=ΝΛ

Α

,. .... \' , ; 1 2 \ ' ,

Β Δ

' ' '

ΖΓ.lΒΓ

' z

Γ

i) Το τετράπλευρο ΑΖΓΕ είναι παραλληλόγραμ­μο γιατί οι διαγώνιες διχοτομούνται επειδή

ΑΓ .lEZ, τότε το τετράπλευρο είναι ρόμβος.

ii) Αφού το ΑΖΓΕ είναι ρόμβος τότε ΖΓ//ΑΕ και ΑΔ .l ΒΓ, συνεπώς θα είναι ΖΓ .lΒΓ.

iii) Είναι ΑΒ//ΖΛ τότε Λ, = Β (ως εντός εκτός και

επί τα αυτά μέρη γωνίες ίσες) . Είναι Β = f οπότε

Λ, = f δηλαδή το τρίγωνο ΛΝΓ είναι ισοσκελές

συνεπώς ΝΛ=ΝΓ ( 1 ) Είναι f , = Α 2 (ως εντός εναλλάξ) Ζ, = Α , (ο-

ξείες γωνίες με πλευρές παράλληλες) και Α1 = Α 2 (η ΑΔ είναι και διχοτόμος του τριγώνου ΑΒΓ) άρα

z, = f , συνεπώς το τρίγωνο ΖΝΓ είναι ισοσκελές,

τότε ΝΓ=ΖΝ (2).

Από τις ( 1 ) και (2) έχουμε ΖΝ=ΝΓ.

ΕΥΚΛ ΕΙΔΗΣ Β' τ.2/37

Page 40: Ευκλειδης Β 62

•••ιιιι••••• ,., ., • ., ιr 8'ιl�φ 8'•ιι Αιι••Ι•ιι

Άλγεβρα Πολυώνυμα

Υπολογισμός αθ ρ ο ισμάτων

Παρακάτω θα αναφερθούμε στη συμβολή των πολυωνύμων για τον υπολογισμό των αθροισμά­των των οποίων οι όροι σχηματίζονται με την ίδια λογική αλλά αναφέρονται σε διαδοχικές τιμές της μεταβλητής που είναι φυσικός αριθμός.

_ _ ν (ν + 1 ) * Π.χ. S - 1 + 2 + 3 + . . . + ν - , ν ε Ν 2

Π ρ ό βλη μ α I α) Να προσδιορίσετε το πολυώνυμο Ρ(χ) τρίτου βαθμού όταν ισχύουν: Ρ(Ο)=Ο και

P(x+l)P(x)= χ2• β) Να υπολογίσετε το S=12+22+ . . . +ν2, νε Ν* συναρτήσει του ν. ΛίJση

α) Έστω Ρ( χ)= αχ3+βχ2+γχ+δ, α:ji:O και α,β,γ,δ ε � Τότε από την Ρ(Ο)=Ο<::>δ=Ο άρα

Ρ(χ)= αχ3+βχ2+γχ και από την Ρ(χ+ 1 ) -Ρ(χ)= χ2, χ εR<::>α(χ+ 1 )3+β(χ+ 1 )2+γ(χ+ 1 ) -[αχ3+βχ2+γχ]=χ2 <::>3αχ2 +(3α+2β)χ+(α+β+γ)= χ2+0χ+Ο<::>

1 1 1 <::> α = 3 , β = -2 , γ = 6 .

, χ3 x z χ χ (2χ2 - 3χ + 1 ) Άρα το Ρ ( χ ) =- --+- =�---� 3 2 6 6

' ( ) χ ( χ - 1 ) ( 2χ - 1 ) η Ρ χ = . 6

β) Χρησιμοποιώντας το (I) για x= l ,2, . . . ,ν. Βρίσκουμε :

Ρ ( 2 ) - Ρ ( 1 ) = 1 2 Ρ (3 ) - Ρ ( 2) = 22 Ρ (4) - Ρ (3 ) = 32

Ρ ( ν + 1 ) - Ρ (ν) = ν2

Λ. Θαρραλίδης, Κ. Παρλαπάνης, Μ. Ρουσούλης

Προσθέτοντας κατά μέλη τις παραπάνω ισότητες. Έχουμε: Ρ ( ν + 1 ) - Ρ ( 1 ) = 12 + 22 + . . . + ν2 = S

( ν + 1 ) · ν · ( 2ν + 1 ) Έτσι S = - Ο τελικά

Π ρ ό βλη μα 2

6

S = ν (ν + 1 ) { 2ν + 1 ) 6

α) Να προσδιορίσετε το πολυώνυμο τετάρτου βαθμού όταν ισχύουν: Ρ(Ο)=Ο και

P(x+l) -Ρ(χ)= χ3, χεR β) Να υπολογίσετε το άθροισμα S3 = 13 + 23 + . • • + ν3 , ν ε Ν * συναρτήσει του ν

Λύση

Α ν εργαστούμε με τον ίδιο τρόπο θα βρούμε , ( ) χ 2 ( χ - 1γ ν2 · ( ν + 1 )2 οτι το Ρ χ = και το S = -------'---'-

4 4

Π ρ ό βλη μα 3 α) Να προσδιορίσετε το πολυώνυμο τρίτου βαθ­μού όταν ισχύουν Ρ(Ο)=Ο και

P(x+l)-P(x)=x(x+l), χεR β) Να υπολογίσετε το άθροισμα

S=1 ·2+2·3+3·4+ . . . +ν(ν+1) Λύση

Αν εργαστούμε με τον ίδιο τρόπο θα βρούμε ότι το Ρ ( χ ) = ( χ - 1 ) χ ( χ + 1 ) και το 3 S = ν ( ν + 1 ) {ν + 2) . 3 Π ρ ό βλη μ α 4 α) Να προσδιορίσετε το πολυώνυμο τρίτου βαθ­μού όταν ισχύουν Ρ(Ο)=Ο και

ΕΥΚΛΕΙΔΗΣ Β' τ.2/38

Page 41: Ευκλειδης Β 62

Μαθη ματικά για την Β ' Λυκείου

P(x+l )-P(x)= χ(χ+2), χεR β) Να υπολογίσετε το άθροισμα

S=1 ·3+2·4+3·5+ . . . +ν(ν+21 ), νε Ν*

. \ ίiση Αν εργαστούμε με τον ίδιο τρόπο θα βρούμε

ότι το Ρ (χ) = χ (χ - 1 ) (2χ + 5) και το S = ν (ν + 1 ) (2ν + 7 )

_ 6 6

Ιl ιφ1ι.τηρψη:ις Στα παραπάνω προβλήματα παρατηρούμε ότι για το πολυώνυμο Ρ(χ) ισχύουν : i) Ρ(Ο)=Ο i i) Η Διαφορά Ρ( χ+ 1 )-Ρ( χ) είναι πολυώνυμο ίσο

με το πολυώνυμο που αντιστοιχεί στον τελευ­ταίο όρο του αθροίσματος που θέλουμε να υ­πολογίσουμε βάζοντας τη μεταβλητή χ στη θέση του ν.

iii) Ο βαθμός του Ρ(χ) είναι μεγαλύτερος κατά 1 από το βαθμό του [Ρ(χ+ 1 )-Ρ(χ)] . Δημιουργείστε δικά σας αθροίσματα και υπο­λογίστε τα.

Διαιρετότητα μi: δυο παράγοντες Π ρ όταση- Λσκηση

Να δείξετε ότι αν το πολυώνυμο Ρ(χ) έχει παράγοντες x-k και χ-λ με k;t:λ θα έχει και ως παράγοντα το γινόμενο (x-k). (χ-λ)

(Δηλαδή το Ρ(χ) θα διαιρείται δια του (x-k) · (χ-λ)).

Ισχύει και το αντίστροφο.

Επειδή το Ρ( χ) έχει παράγοντες του x-k και χ­λ θα ισχύουν P(k)=O και Ρ(λ)=Ο

Αν η διαίρεση Ρ(χ) : (x-k) έχει πηλίκο π(χ) θα ισχύει P(x)=(x-k) . π(χ) ( 1 )

Από την ( 1 ) προκύπτει Ρ(λ)=(λ-k) ·π(λ) <::>Ο=(λ-k) - π(λ) <=> π(λ)=Ο άρα το λ είναι ρίζα του π( χ) τότε π( χ)= (χ-λ)Q(χ) (2)

Έτσι από τις ( 1 ) και (2) Ρ(χ)=(χ-k) · (χ-λ) Q(x). Δηλαδή το (x-k) · (χ-λ) είναι παράγοντας του Ρ(χ) . Το αντίστροφο είναι προφανές.

l:z6λιο : Η πρόταση- άσκηση γενικεύεται: Αν το Ρ(χ) έχει παράγοντες (χ-ρ 1 ) , (χ-ρ2) ,

(χ-ρ3) , . . . , (χ-ρv) με ρ , , ρ2, ρ3 , . . . , ρν είναι διαφορε­τικοί αριθμοί ανά δυο τότε το Ρ(χ) διαιρείται με το

γινόμενό τους (χ-ρ , ) - (χ-ρ2) , . (χ-ρ3) . . . (χ-ρv)

Άσκη ση α) Αν το πολυώνυμο Ρ(χ), διαιρείται με το πο­λυώνυμο Q(x) και έχει ρίζα το ρ που δεν είναι ρίζα του Q(x) δείξτε ότι το Ρ(χ) διαιρείται με το γινόμενο Q(x) · (χ-ρ) β) Αν ο μεγιστοβάθμιος όρος ενός πολυωνύμου Ρ(χ) είναι το χ3 και το Ρ(χ) έχει ρίζα το α ε R και διαιρείται με το χ2-αχ+1 . i) Να δείξετε ότι Ρ(χ)= (χ2-αχ+1) (χ-α) ii) Αν το Ρ(χ) έχει θετική διπλή ρίζα υπολογί­

στε το α και κατόπιν να λυθεί την εξίσωση Ρ(χ)=Ο

.\ίJση α) Από την υπόθεση θα ισχύουν

Ρ(χ)= Q(χ).Π(χ) ( 1 ) Ρ(ρ)=Ο (2) Q(ρ);t: Ο (3)

Τότε από τις 1 ,2 έχουμε Q(ρ) Π(ρ)=Ο<::> Π(ρ)=Ο Έτσι το Π(χ) θα έχει παράγοντα χ-ρ, επο­μένως θα ισχύει Π(χ)= (χ-ρ). φ(χ). (4)

τελικά από τις 1 ' 4 προκύπτει Ρ(χ)= Q(χ) · (χ-ρ) φ(χ) .

β) i) Παρατηρούμε ότι πολυώνυμο Q(x)= χ2 -αχ+ 1

δεν έχει ρίζα το α αφού Q(α)= 1:;t0. Τότε σύμ­φωνα με το α) ερώτημα το Ρ(χ) θα διαιρείται με το Q(x). (χ-α)= χ3 -2αχ2+( α2+ 1 ) χ-α. Έτσι η διαίρεση του Ρ( χ)= χ3 -α2χ2+α 1 χ +α0 με το Q(x). (χ-α) έχει υπόλοιπο U(x)=O και προφα­νώς το πηλίκο είναι Π(χ)= 1 , άρα

Ρ(χ)=(χ2 -αχ+ 1 ) (χ-α) ii) Οι ρίζες του Ρ(χ) είναι το α και οι (ενδεχόμε­

νες) ρίζες του Q(χ)=χ2-αχ+ 1 . Αφού το α δεν είναι ρίζα του Q(x) συμπεραίνουμε ότι θετική διπλή ρίζα δεν είναι το α. Δηλαδή θα πρέπει το Q(x) να έχει θετική διπλή ρίζα αρκεί: Δ=Ο και S>Ο<::>α2--4=0 και α>Ο�α=2 . Με α=2 το Ρ(χ)=(χ2-αχ+ l ) (χ-α)= (χ- 1 )2(χ-2) . Έτσι το Ρ(χ)=Ο<::> χ=2 ή χ= 1 (Διπλή) !\σκηση Το πολυώνυμο Ρ(χ)= χ3+2χ2+αχ+β, α,βε R

έχει ρίζες τους αριθμούς 1 , ρ 1 , Ρ2 αν Ρ 1 "Ρ2=2 να υπολογίσετε το Ρ(χ) .\ίJση

Αφού το Ρ(χ) έχει ρίζα το 1 θα ισχύει ΕΥΚΛΕΙΔΗΣ Β' τ.2/39

Page 42: Ευκλειδης Β 62

Μαθηματικά για την Β ' Λυκείου

Ρ( 1 )=0�α+β+3=0�Β= -α-3 . Έτσι το Ρ(χ)= χ3+2χ2+αχ-α-3 το οποίο έχει

παράγοντα το (χ- 1 ) Κάνουμε με το σχήμα του Horner την

Ρ(χ) : (χ+ 1 ) 2 α 1

3 α+3 3 α+3 ο

Οπότε Ρ(χ)= (χ- 1 ) [ χ2+3χ+(α+3)] Επειδή το Ρ(χ) έχει ρίζες και τους p 1 , p2 αυτές

θα είναι ρίζες του Π(χ)= χ2+3χ+(α+3) . Έτσι Ρ ιΡ2=α+ 3=2�α=-1 και β=-2 άρα το

Ρ(χ)=χ3+2χ2-χ-2 ;\ Ο' [Ι( :ΗΠ� Να βρείτε τα α, β ώστε το Ρ(χ)= χ3+αχ-β να

διαιρείται με το πολυώνυμο Q(x)=x2 -5χ+4

Για να διαιρείται το Ρ(χ) με το Q(x) αρκεί να ισχύει: Ρ(χ)=Q(χ) .Π(χ) Vx Ε IR.

� χ3+αχ-β=(χ2 -5χ+4) . Π(χ) � χ3+αχ-β=(χ--4)( χ-1 ) . Π(χ) ( 1 )

Η ισότητα ( 1 ) ισχύει Vx Ε IR. άρα και για χ=4 και χ= 1 . {1 + α - β = 0 {1 + α - β = Ο {β = -20 Ετσι: � � 64 + 4α- β = Ο 63 + 3α = Ο α = -2 1

Δίνεται το Ρ(χ)=(α2-α)χ3+(α2-1)χ2+3αχ-α-1 , αΕR

α) Να δείξετε ότι τ ο -2 δεν είναι ρίζα του Ρ( χ) β) Να βρείτε το αΕR ώστε το Ρ(χ) να είναι 2ου βαθμού

α) Θα δείξουμε ότι για κάθε αΕR το Ρ(-2):1:0 . Έχουμε Ρ( -2)=( α2 -α) . (-2)3+( α2 -1 )(-2)+ 3α(-2)-α-1 =

=4α2+α-5 . Όμως το τριώνυμο 4α2+α-5 έχει Δ=-79<0 άρα για κάθε αΕR ισχύει 4α2+α-5>0 δηλαδή Ρ(-2)>0 άρα πράγματι το (-2) δεν είναι ρίζα του Ρ(χ)

{α (α - 1 ) = 0 � και � α = Ο

α :;t ± 1

Δίνεται το πολυώνυμο P(x+l)=x2-3x α) Να βρείτε το Ρ(χ) β) Ν α δείξετε ότι έχει ρίζα το 1

α) Με την μέθοδο της αντικατάστασης κάνουμε το μετασχηματισμό χ+ 1 =y � x=y-1 Έτσι P(y)=(y-1 )2-3 (y- 1 ) ή P(y)= y2-2y+ l-3y+3 ή P(y)= /-5y+4 β) Παρατηρώ ότι Ρ( 1 )= 1 2-5 . 1 +4=0 άρα το 1 είναι ρίζα του Ρ( χ)

Να βρείτε πολυώνυμο Ρ(χ) 2ου βαθμού αν η πολυωνυμική συνάρτηση είναι άρτια, το άθροι­σμα των συντελεστών του Ρ(χ) είναι (-1) και ι­σχύει Ρ(-2)=-7

Το ζητούμενο Ρ(χ) θα έχει μορφή Ρ(χ)= αχ2+βχ+γ. Επειδή η συνάρτηση Ρ(χ) είναι άρτια για κάθε xER το -xER και Ρ(χ)= Ρ(-χ). Έτσι αχ2+βχ+γ=α(-χ2) +β(-χ)+γ� 2βχ=Ο�β=Ο άρα Ρ(χ)= αχ2+γ ( 1 ) Επίσης από τα δεδομένα έχουμε α+γ=- 1 και Ρ(-2)=7� α+γ=-1 και 4α+γ=-7 �α=-2 και γ= 1 Έτσι Ρ(χ)=-2χ2+ 1

Αν η εξίσωση χ3-χ2ημω+(α+2)χ+l=Ο (Ι) έχει θετικούς ακεραίους συντελεστές και αρνητική ακέραια ρίζα α) Να βρείτε τα α και ω αν ω Ε [0,2π] και να λύ­σετε την εξίσωση (I)

Για να έχει η δοθείσα εξίσωση θετικούς ακέ­ραιους συντελεστές θα πρέπει α+2>0 και ημω=-1 (αφού ισχύει -1 �ημω� 1 ) . Επομένως θα είναι α>-2

3π και ω =- . 2 β) Για να είναι το Ρ(χ) 2ou βαθμού θα πρέπει α2-α=Ο και α2- 1;t:Ο .

Όμως από την υπόθεση η εξίσωση έχει αρνη­τική ακέραια ρίζα. Επομένως αυτή θα είναι υπο­χρεωτικά διαιρέτης του σταθερού όρου α0= 1 δη­

ΕΥΚΛΕΙΔΗΣ Β ' τ.2/40

Page 43: Ευκλειδης Β 62

Μ αθηματικά για την Β ' Λυκείου

λαδή ο αριθμός -1 οπότε από την εξίσωση προκύ­πτει (-1 )3 -(-1 )2(-1 )+(α+2)(- 1 )+ 1 =Ο <:::>--- 1 + 1 -α-

2+ 1 =0�α=-1 .

Με α=-1 και ω = 3π η εξίσωση (I) γράφεται: 2

χ3+χ2+χ+ 1 =Ο � χ2(χ+ 1 )+χ+ 1 =Ο�( χ+ 1 ) (χ2+ 1 )=Ο {χ + 1 = ο {χ = - 1 � ή � � χ = - 1 .

2 χ 2 = - 1 αδύνατη χ + 1 = 0

;\ σ κ ι1ση Δίνεται το Ρ(χ)= 2χ3-7χ2+ 7x+l

α) Να βρείτε το πολυώνυμο Q(x) ώστε Ρ(χ)= (χ-1) Q(x)+3

β) Να λύσετε τη εξίσωση Ρ(2χ-1)=3 γ) Να λύσετε την aνίσωση Ρ(χ2+ χ+2)<3

Η Q = (2χ - 1 ) = 0 � 2χ - 1 = 2 ή 2χ - 1 = .!.. � χ = � ή

2 2

3 χ = - .

4

Έτσι οι ζητούμενες ρίζες είναι 1

' 3 ' 3 Χ = η χ = - η Χ = - .

2 4 γ) Η ανίσωση Ρ(χ2+χ+2)<3�(χ2+χ+ 1 )

Q(x2+ χ+2)+ 3<3�(χ2+χ+ 1 )Q(x2+x+ 2)<0

Q(x2+ χ+2)<0 (Π) (το χ2+ χ+ 1 έχει Δ=-3<0 άρα Vx Ε IR. ισχύει χ2+ χ+ 1 >0)

Θα μελετήσουμε το πρόσημο του

� ��F2��2)�-1 Π:

+οο

+

Επομένως y=x2+x+2 από την (Π) προκύπτει

α) Από Ρ(χ)= (χ- 1 ) Q(x)+3�2x3-7x2+ 7χ-2=(χ-1 ) .Q(x) άρα το Q(x) θα είναι το πηλίκο της (2χ3-

7χ2+ 7χ-2) : (χ- 1 ) από το σχήμα του Homer βρί­σκουμε Q(x)=2x

2-5x+2 ή Q(x)=2(x-2) · (x-1 /2)

xz + χ + 2 >-!. xz + χ +� > Ο ( 1) 2 2

β) Η εξίσωση P(2x-1 )=3�(2x-2)Q(2x-

1 )+3=3�2(x-1 )Q(2x-1 )=0�x= 1 ή Q(2x-1 )=0

(Π) Επειδή το Q(x) έχει ρίζες ρ1 = 2 και ρ2 = .!_ . 2

Η ( 1 ) ισχύει για κάθε χ Ε R αφού Δ<Ο

Η (2) � x(x+ l )<O <:::>-- 1 <χ<Ο άρα Χ Ε (- 1 ,0) .

Γεωμετρία Εμβαδά ευθυγράμμων σχη μάτων

(2)

Δημάδης Χρήστος Μάγκος Αθανάσιος

Οι σχέσεις για τα τρίγωνα που θα χρησιμοποιηθούν παρακάτω είναι:

Ι) E= lhαυa= lhβυp= lhγυγ

2) Ε = �τ( τ - α)(τ - β)( τ - γ) όπου τ η ημιπερίμετρος του τριγώνου

3) Ε= lhβγημΑ = lhαγημΒ= lhαβημΓ

4) Ε = αβγ όπου R η ακτίνα του περιγεγραμμένου κύκλου του τριγώνου. 4R

5) Ε=τρ όπου ρ η ακτίνα του εγγεγραμμένου κύκλου του τριγώνου.

6) Ε=(τ-α)ρa =(τ-β)ρp =(τ-γ)ργ όπου Ρα , PfJ , pγ οι ακτίνες των παρεγεγραμμένων στο τρίγωνο κύκλων.

Για να λύσουμε μία άσκηση δεν ξεχνάμε ότι: 7) Αν δύο τρίγωνα έχουν ίσες βάσεις , τότε ο λόγος των εμβαδών είναι ίσος με τον λόγο των αντίστοιχων

υψών.

8) Αν δύο τρίγωνα έχουν ίσα ύψη , τότε ο λόγος των εμβαδών είναι ίσος με το λόγο των αντίστοιχων βά­

σεων.

ΕΥΚΛΕΙΔΗΣ Β ' τ.2/4 1

Page 44: Ευκλειδης Β 62

Μ αθη ματικά για την Β ' Λυκείου

9) Αν δύο τρίγωνα ΑΒΓ και Α 'Β 'Γ έχουν Α = λ ή Α +λ = 1 80° τότε (ΑΒΓ) βγ (ΑΉ 'Γ) βγ

1 Ο) Αν τα τρίγωνα ΑΒΓ και Α Β 'Γ είναι όμοια με λόγο ομοιότητας λ τότε (��::;:.) = λ 2 . Η σχέση ισχύει

για οποιαδήποτε όμοια πολύγωνα.

()f:μο η " Έστω κύκλος κέντρου Ο και ακτίνας ρ και

σημεία Μ, Κ αυτού. Στην επίκεντρη γωνία ΜΟΚ εγγράφουμε κύκλο κέντρου Π και ακτί­νας R. Να υπολογίσετε το εμβαδόν του τετρα­πλεύρου ΜΟΚΠ.

Μ R ρ

π ο

κ

Ως γνωστόν, η διάκεντρος ΟΠ διχοτομεί τη γωνία ΜΟΚ και άρα τα τριγωνα ΜΟΠ και ΚΟΠ είναι ίσα (αφού ΟΠ κοινή και ΟΜ=ΟΚ=ρ) . Επο­μένως (ΜΟΚΠ)=2(ΜΟΠ).

1 1 Είναι όμως (ΜΟΠ) = 20Μ · υ0Μ = 2ρ · R , άρα (ΜΟΚΠ)= ρ · R

Θi:μα 2 " Δίνεται τρίγωνο ΑΒΓ με α > β . Αποδείξτε

ότι ισχύει α + υ α � β + υ β και να εξεταστεί πότε

ισχύει η ισότητα.

. \ ί;ση Για το εμβαδόν Ε του τριγώνου ΑΒΓ ισχύουν 1 2Ε 1 2Ε Ε =2α · υα <::::> υα =-;: και Ε = '2β · υp <::::> υβ =τ · Έχουμε επομένως α + υ α � β + υβ

2Ε 2Ε 1 1 <::::> α +- � β +- <=> α - β + 2Ε · (- - -) � Ο <::> α β α β αβ(α - β) - 2Ε(α - β) � Ο <::::> (α - β)(αβ - 2Ε) � Ο . Η σχέση αυτή όμως ισχύει, αφού α > β και

1 1 Ε = -αβη μ Γ ::::; -αβ . 2 2 Για να ισχύει α + υ α = β + υβ , όπως φαίνεται

από την απόδειξη , θα πρέπει να έχουμε

Ε = _!_αβ <::::> η μ Γ = 1 <=> Γ = 90° , δηλαδή το τρί-2 γωνο να είναι ορθογώνιο, με ορθή τη γωνία Γ.

Θ{:μα 3" Έστω τρίγωνο ΑΒΓ εμβαδού Ε. Ας είναι Μ

το μέσο της πλευράς ΒΓ, Ν το μέσο της διαμέ­σου ΑΜ, και Δ το σημείο τομής της ΒΝ με την πλευρά ΑΓ. Να υπολογίσετε το εμβαδό του τρι­γώνου ΑΔΝ.

Α

Δ

Ν z

Β Μ Γ

ΑίJση Η ΒΝ είναι διάμεσος του τριγώνου ΑΒΜ, άρα

(ΑΒΝ)=_!_ (ΑΒΜ) , και ομοίως ανάλογα 2 (ΑΒΜ)=_!_ (ΑΒΓ) , αφού η ΑΜ είναι διάμεσος του 2 τριγώνου ΑΒΓ. Επομένως έχουμε (ΑΒΝ)=_!_ (ΑΒΓ) = _!_Ε . Φέρουμε τώρα από το Μ 4 4 παράλληλη στην ΒΔ, η οποία τέμνει την ΑΓ στο σημείο Ζ. Επειδή Ν μέσο της ΑΜ και ΝΔ//ΕΖ, ι­σχύει ΑΔ=ΔΖ. Ομοίως, Ε μέσο της ΒΓ και ΕΖ//ΒΔ, άρα ΔΖ=ΖΓ, δηλαδή ΑΔ=_!_ΑΓ . 3

Παρατηρούμε τώρα ότι τα τρίγωνα ΑΒΔ και ΑΒΓ έχουν ίσα ύψη και επομένως για το λόγο των β δ . θ . (ΑΒΔ) ΑΔ 1 . εμ α ων τους α ισχυει =- =- , αρα (ΑΒΓ) ΑΓ 3

1 (ΑΒΔ)= -Ε . Τελικά επομένως λαμβάνουμε 3 1 1 1 (ΑΔΝ)=(ΑΒΔ)-(ΑΒΝ)= -Ε --Ε =-Ε 3 4 1 2

Θέμα 4" Έστω ορθογώνιο τρίγωνο ΑΒΓ με Α=90°.

ΕΥΚΛΕΙΔΗΣ Β . τ.2/42

Page 45: Ευκλειδης Β 62

Μαθη ματικά για την Β ' Λυκείου

Εξωτερικά του τριγώνου κατασκευάζουμε το τετράγωνο ΓΒΕΔ. Να δειχθεί ότι (Γ ΔΑ)+(ΑΒΕ)=Υ2ΒΓ2

/ I """-----"'ω'-':::_ - - - - - - - - .l - - -Λ Β Τ

ΡΔ _l ΑΓ, ΕΤ _l ΑΒ Τα ορθογώνια τρίγωνα ΑΒΓ, ΓΡΔ και ΒΕΤ εί­

ναι ίσα μεταξύ τους άρα ΡΓ=ΑΒ=ΤΕ και ΡΔ=ΑΓ=ΒΤ

Αν ΑΗ//ΔΕ τότε ( ΑΓΔ) = (ΗΓΔ) = _!_(ΗΘΓΔ) 2 (ΑΒΕ) = (ΗΒΕ ) = _!_(ΗΘΒΕ) 2

Οπότε ( ΑΓΔ) + ( ΑΒΕ) =_!_ [(ΗΘΓΔ) + (ΗΘΒΕ) J = 2 = _!_(ΒΓ ΔΕ ) = _!_ (ΒΓ)1 2 2 Οπότε : (ΑΔΓ)+(ΑΒΕ)=ΥzΑΓ ΡΔ+ΥzΑΒ ΤΕ=

=ΥzΑΓ ΑΓ+ΥzΑΒ ΑΒ=Υz(ΑΓ2+ΑΒ2)=ΥzΒΓ2

Δύο κύκλοι εφάπτονται εσωτερικά στο ση­μείο Λ , και τυχαία χορδή ΑΒ του μεγάλου κύ­κλου τέμνει τον μικρό στην χορδή ΒΓ. Η σειρά των σημείων είναι Α ,Γ ,Δ και Β. Να δειχθεί ότι:

Α φ

ΑΓ ΚΑ · ΚΓ = ---ΔΒ ΚΔ · ΚΒ

Δ

Γ ω

θ

ω=φ+θ ' στο ΑΓΚ τρίγωνο

Β

θ Ψ

κ

ω=φ+θ (από χορδή ΚΔ και εφαπτομένη) Άρα θ=θ ' δηλαδή ΜΤ = ΔΚΒ (ΑΓΚ) ΚΑ · ΚΓ ( 1 ) (ΔΚΒ) ΚΔ · ΚΒ

όμως τα τρίγωνα ΑΓΚ και ΔΒΚ έχουν και κοινό ύψος στην ΑΓ και ΔΒ άρα

(ΑΓΚ) ΑΓ (ΔΒΚ) ΔΒ (2)

Από την ( 1 ) και (2) προκύπτει το ζητούμενο.

Τριγώνου ΑΒΓ, ΑΔ η διχοτόμος και ΑΜ η διάμεσος. Ο περιγεγραμμένος κύκλος του τρι­γώνου ΑΒΓ τέμνει τις ΑΒ και ΑΓ στα σημεία Ε, Ζ. Να δειχθεί

α) ΒΕ=ΓΖ β) (ΒΕΔ)=(ΔΖΓ) γ) ΕΔ=ΔΖ

Α

Ε

Β

, \; ση Δ Μ

z

Γ

α) Η δύναμη του Β και του Δ αντίστοιχα ως προς τον περιγεγραμμένο στο ΑΔΜ τρίγωνο είναι

ΒΔ · ΒΜ = ΒΕ · ΒΑ ( 1 )

ΓΜ · ΓΔ = ΓΖ - ΓΑ (2) Με διαίρεση κατά μέλη τις ( 1 ) , (2)

ΒΔ ΒΑ έχουμε ΒΕ=ΓΖ αφού ΒΜ=ΜΓ και - - ­ΓΔ ΓΑ Θ.Διχοτόμου . β) Τα τρίγωνα ΒΕΔ και ΔΖΓ τις βάσεις ΒΕ=ΓΖ και ίσα τα αντίστοιχα σε αυτές ύψη αφου το Δ α­νήκει στην διχοτόμο της γωνίας ΒΑΓ . γ) ΒΕΔ = ΑΖΔ και ΑΖΔ + ΔΖΓ = 1 80°

άρα (ΒΕΔ) = ΒΕ . ΔΕ = 1 ά α ΔΕ=ΔΖ (ΔΖΓ) ΔΖ · ΖJ7' ρ

(Προκύπτει και άμεσα αφού ΔΑΕ = ΔΑz => ΔΕ = ΔΖ=> ΔΕ = ΔΖ )

ΕΥΚΛΕΙΔΗΣ Β ' τ.2/43

Page 46: Ευκλειδης Β 62

Μαθη ματικά για την Β ' Λυκείου

Θ I; μ α Έστω τρίγωνο ΑΒΓ και ΓΖ διάμεσος του.

Ας είναι Π τυχαίο σημείο της ΓΖ και Ε, Δ τα σημεία στα οποία οι ΒΠ, ΑΠ, τέμνουν τις ΑΓ και ΒΓ αντίστοιχα. Ν α αποδείξετε ότι (ΠΕΓ)=(ΠΓ Δ).

Α

Ε

z π

Β Δ Γ

Τα τρίγωνα ΠΖΑ και ΠΓΔ έχουν τις γωνίες τους ΑΠΖ και ΠΓΔ ίσες ως κατακορυφήν. Άρα ι­σ ' ει (ΠΖΑ) = ΠΖ · ΠΑ ( 1 ) .

χυ (ΠΓΔ) ΠΔ · ΠΓ . (ΠΕΓ) ΠΕ · ΠΓ Ομοίως προκυπτει = (2). Ε-(ΠΒΖ) ΠΖ · ΠΒ

ξάλλου, η ΠΖ είναι διάμεσος του τριγώνου ΠΑΒ και άρα ισχύει (ΠΑΖ)=(ΠΒΖ) (3) .

Πολλαπλασιάζοντας τις ( 1 ) , (2) κατά μέλη . (ΠΖΑ) · (ΠΕΓ) ΠΖ · ΠΑ · ΠΕ · ΠΓ λαμβανουμε = ------(ΠΓΔ) · (ΠΒΖ) ΠΔ · ΠΓ · ΠΖ · ΠΒ

και κάνοντας χρήση της (3) προκύπτει (ΠΕΓ) ΠΑ · ΠΕ δ ξ . 'λ . -'--------'- = . Το ε ιο με ος της σχεσης (ΠΓΔ) ΠΔ · ΠΒ

, , . , (ΠΑΕ) , , αυτης ειναι ομως ισο με , αφου τα τριγωνα (ΠΒΔ) ΠΑΕ και ΠΒΔ έχουν ένα ζεύγος γωνιών ίσες ως

(ΠΕΓ) (ΠΑΕ) , . δ . . Ισχύει λοιπόν = , οποτε απο ι ιο-(ΠΓΔ) (ΠΒΔ) τητα των αναλογιών θα ισχύει και (ΠΕΓ) (ΠΑΕ) _ (ΠΕΓ) + (ΠΑΕ) (ΠΓΔ) = (ΠΒΔ) - (ΠΓΔ) + (ΠΒΔ) ' δηλαδή (ΠΕΓ) - (Γ ΑΖ) - (ΠΑΖ) 1 , γιατί (Γ ΑΖ)=(ΓΒΖ) (ΠΓ Δ) (ΓΒΖ) - (ΠΒΖ) και (ΠΑΖ)=(ΠΒΖ) αφού Ζ μέσο της ΑΒ.

Προέκυψε δηλαδή (ΠΕΓ)=(ΠΓ Δ).

αφού fi 1 = fi2 αρκεί ΠΑ · ΠΕ = ΠΒ · ΠΔ , ή ΠΑ = ΠΒ , ή τελικά ΔΕ//ΒΓ. ΠΔ ΠΕ

Πράγματι, από το Θ. Ceνa έχουμε : ΖΑ . ΔΒ . ΕΓ = Ι => ΔΒ = ΕΑ => ΔΕ //ΒΓ ΖΒ ΔΓ ΕΑ ΔΓ ΕΓ

ΘΕΜΑ 7 Τρίγωνο ΑΒΓ εγγεγραμμένο σε κύκλο. Οι

διάμετροι ΑΛ,ΒΜ και ΓΝ τέμνουν το τρίγωνο στα σημεία Δ,Ε και Ζ. Να δειχθεί ότι

Ν Ζ

Β

1 1 1 2 - + - + - = -ΑΔ ΒΕ ΓΖ R

Α

Ε κ

Δ

Λ

Μ

Γ

Τα τρίγωνα ΑΒΚ και ΑΒΔ έχουν κοινό ύψος από την κορυφή Β και τα ΑΓΚ και ΑΓ Δ από την κορυφή Γ. Επομένως (ΑΒΚ) (ΑΒΔ)

ΑΚ R (ΑΓΚ) ΑΚ R -=- και =-=-ΑΔ ΑΔ (ΑΓΔ) ΑΔ ΑΔ Άρα

(ΑΒΚ) (ΑΓΚ) (ΑΒΚ) + (ΑΓΚ) = = (ΑΒΔ) (ΑΓΔ) (ΑΒΔ) + (ΑΓΔ) (ΑΒΚ) + (ΑΓΚ)

(ΑΒΓ) Με τον ίδιο τρόπο (ΑΒΚ) + (ΒΚΓ) = � (2) (ΑΒΓ) ΒΕ (ΑΚΓ)+(ΓΚΒ) = � (3) (ΑΒΓ) ΓΖ

και

= R ( 1 )

= ΑΔ

με πρόσθεση κατά μέλη των σχέσεων ( 1 ) , (2), (3) έχουμε

� + � + � = 2(ΑΒΓ) = 2 ΑΔ ΒΕ ΓΖ (ΑΒΓ) δηλαδή -1- + -1-+- = 2

ΑΔ ΒΕ ΓΖ R Αφού (ΠΑΓ)=(ΠΒΓ), αρκεί (ΠΑΕ)=(ΠΒΔ) και

ΕΥΚΛΕΙΔΗΣ Β' τ.2/44

Page 47: Ευκλειδης Β 62

Μ αθηματικά για την Β ' Λυκείου

Θέμα 8" Έστω τρίγωνο ΜΛΚ εγγεγραμμένο στο

τρίγωνο ΑΒΓ και περιγεγραμμένο στο ΤΠΝ. Οι πλευρές του ΤΠΝ είναι παράλληλες μία προς μία με τις πλευρές του ΑΒΓ. Να δειχθεί ότι (ΜΛΚ)2=(τΠΝ)(ΑΒΓ). (τα εμβαδά των τριών τριγώνων αποτελούν γεωμετρική πρόοδο )

Α

Μ

Ρ

Β Σ

π

τ Λ

Ν

κ Γ

.\{)ση

ΑΣ = υ α και ΑΡ=υ α

ΒΓ = α και ΠΝ=α '

(ΜΑΠ)=(ΠΑ τ) και (τ ΑΝ)=( Α ΤΝ) επειδή η κορυ­φή μεταφέρεται παράλληλα προς την βάση . (ΜΛΚ)=(ΜΠτ)+(τ ΛΝ)+(τΠΝ)+(ΠΝΚ)=

=(ΠΑ Τ)+(τ ΑΝ)+(τΠΝ)+(ΠΝΚ)= =(ΑΠΝ)+(ΠΝΚ)= 1 1 1 =-ΠΝ · ΑΡ + -ΠΝ · ΡΣ = -ΠΝ · ΑΣ ( 1 ) 2 2 2

Επειδή ΑΒΓ όμοιο με ΤΠΝ α. = � (2)

α υ . α

Από την ( 1 ) και (2) 2 1 2 2 1 ·2 2 1 . 1 (ΜΛΚ) =-ΠΝ ΑΣ = - α υ = - α υ - αυ . =

4 4 α 2 α 2 α

1 1 . = - α · υ -α · υ . = (ΑΒΓ) (τΠΝ) 2 α 2 α

Μ αθη ματικά Κατεύθυνσης Απολλώνιες κατασ κευές f,1ε Αναλυτική Γεωμετρί�z

f. ισυγωγtΊ Ένα περίφημο γεωμετρικό θέμα που εμπίπτει

στην ύλη μας είναι τα Δέκα Προβλήματα του Α­πολλωνίου, (Πέργαμος Μικράς Ασίας) . Ας θεωρή­σουμε τα ακόλουθα τρία είδη συνθηκών που είναι σε θέση να ικανοποιεί ένας κύκλος C.

(Σ) Να διέρχεται από ένα δεδομένο σημείο (Ε) Να εφάπτεται σε μια δεδομένη ευθεία (Κ) Να εφάπτεται σε ένα δεδομένο κύκλο

Αν λάβουμε ταυτοχρόνως τρεις συνθήκες από τα παραπάνω τρία είδη συνθηκών, δημιουργούνται δέκα είδη συστημάτων συνθηκών που συμβολίζο­νται ως εξής:

(1) ΣΣΣ (Βασικό) (2) ΣΣΕ (Βασικό) (3) ΣΣΚ (Βασικό) (4) ΣΕΕ (5) ΕΕΕ (6) ΣΕΚ (7) ΣΚΚ (8) ΕΕΚ (9) ΕΚΚ ( 10) ΚΚΚ

Τα δέκα προβλήματα του Απολλωνίου συνίστα­νται στον προσδιορισμό κύκλου C που πληροί ένα

Μιντεκίδης Παντελής,

από τα παραπάνω δέκα συστήματα συνθηκών. π.χ.1 . Το σύστημα ΣΕΕ σημαίνει: «Να προσδιοριστεί κύκλος που διέρχεται από

δεδομένο σημείο και εφάπτεται δύο δεδομένων ευθειών».

π.χ.2 . Το σύστημα ΣΕΚ σημαίνει: Να προσδιοριστεί κύκλος που διέρχεται από

δεδομένο σημείο, εφάπτεται δεδομένης ευθείας και εφάπτεται δεδομένου κύκλου.

Π ιψ ιΡ.η� ρ�σης : 1 . Τα τρία πρώτα προβλήματα τα ονομάζουμε

βασικά, διότι η επίλυση όλων των άλλων ανά­γεται εν τέλει στην επίλυση ενός εκ των τριών βασικών.

2. Το πλήθος λύσεων καθενός εκ των δέκα προ­βλημάτων ποικίλει αναλόγως με τη σχετική θέση των τριών δεδομένων στοιχείων. Έτσι,

ΕΥΚΛΕΙΔΗΣ Β ' τ.2/45

Page 48: Ευκλειδης Β 62

Μαθη ματικά για την ο · Λυκείου

ένα σημαντικό μέρος της επίλυσης του αφορά στη διερεύνησή τους.

π.χ. l . Σχετικώς με το πρόβλημα ΣΣΣ: � Αν τα τρία σημεία είναι συνευθειακά,

τότε το πρόβλημα έχει Ο λύσεις ( αδύνα­το).

� Αν τα τρία σημεία είναι μη συνευθειακά, τότε το πρόβλημα έχει 1 λύση .

π.χ.2 . Σχετικώς με το πρόβλημα ΕΕΕ: � Α ν οι τρεις ευθείες τέμνονται ανά δύο,

τότε το πρόβλημα έχει 4 λύσεις. � Α ν οι δύο μόνον ευθείες είναι παράλλη­

λες, τότε το πρόβλημα έχει 2 λύσεις. � Αν και οι τρεις ευθείες είναι παράλλη­

λες, τότε το πρόβλημα έχει Ο λύσεις. 3 . Η επιστήμη της Ευκλείδειας Γεωμετρίας έχει

ως αντικείμενο μελέτης ένα μέρος του Φυ­σικού Κόσμου : τις σχέσεις των σχημάτων των υλικών σωμάτων. Υπάρχουν δύο τρόποι εφόρασης των γεωμετρικών φαινομένων οι οποίοι συνεπάγονται και δύο αντίστοιχους τρόπους θεμελίωσης, μελέτης και έρευνας της Ευκλείδειας Γεωμετρίας.

Β ΙΗ;ηος τρ6πος : Συνθετικός Εδώ η Ευκλείδεια Γεωμετρία συμπεριφέρεται

ως κλάδος της Φυσικής και σε αυτό τον τρόπο α­ντιστοιχεί η θεμελίωση του Ευκλείδη , του Χίλ­μπερτ, του Τάρσκι, του Μπιρκοφ κ.ά. Ο τρόπος αυτός εφόρασης ονομάζεται Συνθετική Γεωμετρία και είναι αυτός που ακολουθείται από το σχολικό βιβλίο με τίτλο Ευκλείδεια Γεωμετρία(Α ' και Β ' Λυκείου)

�υ :ίηψος τρόπο ς : Αναλυτικός Εδώ η Ευκλείδεια Γεωμετρία συμπεριφέρεται

ως κλάδος των Μαθηματικών και σε αυτό τον τρόπο αντιστοιχεί η θεμελίωση του Ντεκάρτ.

Ο τρόπος αυτός εφόρασης ονομάζεται Αναλυ­τική Γεωμετρία και είναι αυτός που ακολουθείται

τεύθυνσης(Β · Λυκείου) .

τη Συνθετική Γεωμετρία όταν λέμε «να προσ­διοριστεί κύκλος» εννοούμε «να κατασκευαστεί κύκλος», δηλαδή πρόκειται για ένα πρόβλημα γε­ωμετρικής κατασκευής(με κανόνα και διαβήτη) . Έτσι, τα Δέκα Προβλήματα του Απολλωνίου είναι δέκα γεωμετρικές κατασκευές. Εδώ τα δεδομένα δεν είναι απολύτως προσδιοριστά (λόγω έλλειψης συστήματος αναφοράς) αλλά σχετικώς προσδιορι­στά. Στο γεγονός αυτό οφείλεται και η εκτενής δι­ερεύνηση κατά την επίλυση αυτών των προβλημά­των.

Στην Αναλυτική Γεωμετρία όταν λέμε «να προσδιοριστεί κύκλος» εννοούμε «να προσδιορι­στεί η εξίσωση του κύκλου», δηλαδή πρόκειται για ένα πρόβλημα αλγεβρικού λογισμού. Έτσι τα Δέκα Προβλήματα του Απολλωνίου είναι δέκα προβλή­ματα προσδιορισμού εξίσωσης κύκλου . Εδώ τα δεδομένα είναι απολύτως προσδιοριστά (λόγω ύ­παρξης συστήματος αναφοράς) , δηλαδή πρόκειται για δεδομένα «θέσει και μεγέθει». Στο γεγονός αυ­τό οφείλεται και η απουσία διερεύνησης κατά την επίλυση αυτών των προβλημάτων.

5 . Στο πλαίσιο της παρουσίασής μας των Δέ­κα Προβλημάτων του Απολλωνίου θα δώσουμε από ένα παράδειγμα για τα τέσσερα βασικά προ­βλήματα ( 1 ,2,3 ,4) και προτείνονται προς λύση τα υπόλοιπα. Επειδή κάποιοι τύποι δεν ανήκουν στη διδακτέα ύλη του Λυκείου, ενδέχεται μερικοί υπο­λογισμοί να είναι κοπιώδεις. Τέλος, ας σημειωθεί ότι οι λύσεις που δίνονται στο πλαίσιο της Αναλυ­τικής Γεωμετρίας είναι διαφορετικές από αυτές που δίνονται στη Συνθετική Γεωμετρία.

Π ρίψλημα ι ( l:Ll:, βασικό ) Να προσδιοριστεί κύκλος που να διέρχεται

από τα τρία σημεία: A(l ,O), Β(Ο,2) και Γ(2,3).

., Έστω ο ζητούμενος κύκλος C : x 2 + ψ2 + αχ + βψ + γ = Ο

από το σχολικό βιβλίο με τίτλο Μαθηματικά Κα­ΕΥΚΛΕΙΔΗΣ ο · τ.2/46

Page 49: Ευκλειδης Β 62

Μαθηματικά για την Β ' Λυκείου

Θα υπολογίσουμε τις παραμέτρους α, β, γ.

• Είναι έγκυρες οι ισοδυναμίες: Α( 1 ,0) Ε C <::::> 1 2+02+α · 1 +β·Ο+γ=Ο

<::::> α+γ=- 1 . Β(Ο,2) Ε C<::::> 02+22+α·Ο+β ·2+γ=Ο

<::::> 2β+γ=-4. Γ(2 ,3) Ε C<::::> 22+3 2+α ·2+β ·3+γ=Ο

<::::> 2α+ 3 β+γ=- 1 3 • Επίλυση συστήματος: {

;;:=ν-� -4 <::::>

2α + �β + γ = - 1 3

α = -γ - 1 <::::> β = - γ + 4

2 2 (-γ - 1 ) + 3 (- γ ; 4 ) + γ = - 1 3 !α = -γ - 1 α = -3 γ + 4 <::::> β = --2-<::::> {β : ;3 γ = 2 γ

Η καμπύλη (C) ως γνωστόν είναι κύκλος ή μο­νοσύνολο ή κενό σύνολο. Αφού λοιπόν περιέχει 3-σημεία θα είναι σίγουρα κύκλος. Δηλαδή σίγουρα θα ισχύει α2 + β2 - 4γ > Ο , χωρίς έλεγχο .

• Συμπέρασμα: Το πρόβλημα έχει ως λύση : C : χ2+ψ2-3χ-3ψ+2=0

Π ρόβλη μα 2 (ΣΣ Ε , βασικό) Να προσδιοριστεί ο κύκλος που διέρχεται

από τα σημεία Α(3,1), Β(2,-2) και εφάπτεται στην ευθεία ε: 2χ-ψ+ 3=0.

Λί1ση : • Έστω

(Κ, ρ ) : { ( χ - χσ )2 + (Ψ - Ψο γ = ρ2 } ρ > Ο

Ο ζητούμενος κύκλος:

Α(3 , 1 ) (μ) I I I I " I / / I / Β(2,-2)

/� Κ(χο,ψο) / I

/ I " " I

I I I ε :2χ-y+3=0

Θα υπολογίσουμε τις παραμέτρους Χ0, ψ0, ρ2 . • Σκεπτικό :

� Έστω (μ) η μεσοκάθετος της χορδής ΑΒ. Προφανώς ισχύει:

Κ(χο,ψο) ε (μ) ( 1 ) � Εφόσον Α(3 , 1 ) σημείο του κύκλου (Κ,ρ)

ισχύει: (3-ΧΧο)2+( 1 -ψο)2=ρ2 (2)

� Εφόσον (ε) εφαπτομένη του κύκλου (Κ,ρ) ισχύει:

D(Κ,ε)2=ρ2 (3 ) � Λύνοντας το σύστημα των ( 1 ) , (2) , (3)

προσδιορίζουμε τις ζητούμενες παραμέ-2 τρους Χο, ψ0, ρ .

Ακολουθεί η εκπλήρωση των βημάτων του σκεπτικού . • Εξίσωση της μεσοκαθέτου μ:

Μ(χ,ψ) ε (μ)<::::>ΜΑ=ΜΒ <::::>ΜΑ2=ΜΒ2 <=>(χ-3)2+(ψ-1 )2=(χ-2)2+(ψ+ 2)2 <::::>χ+3ψ- 1=0

επομένως η συνθήκη ( 1 ) γίνεται: Χa+3ψ0= 1 (4)

• Ακόμη η (3) γράφεται:

D(Κ,ε J""p' <c> ( l 2x o Js" + 3 1 )' �ρ' <c>

(2χο - ψο + 3)2 = p2 (5) 5 • Επιλύουμε τώρα το σύστημα των (4), (2) και

(5) : ΕΥ ΚΛΕΙΔΗΣ Β ' τ.2/47

Page 50: Ευκλειδης Β 62

Μαθηματικά για την Β ' Λυκείου

χ0 + 3ψ 0 = Ι (3 - χσ γ + ( Ι - ψο )2 = ρ2 <=> ( 2χσ - Ψο + 3)2 2 = ρ 5 χ0 + 3ψ 0 = Ι

( )2 ( )2 ( 2χ0 - ψ0 + 3)2 {::> 3 - χ + ψ - ψ =--'--------� ο ο 5

ρ2 = (3 - χο )2 + ( Ι - ψο )2

{( Χ0 , ψ 0 ) = ( Ι , Ο) ή ( Χ0 , Ψ ο ) = ( 36 Ι , - Ι 20) {::> ? 2 ρ2 = (3 - χ0 γ + ( Ι - ψ0 )

<=> { ( χ0 , ψ 0 ) = ( Ι , Ο ) , {( χ0 , ψ0 ) = ( 36 Ι , - Ι 20) ? η 2 ρ- = 5 ρ = Ι 69

Συμπέρασμα: Το πρόβλημα έχει δύο λύσεις :

(χ- Ι )2+ψ2=5, (χ-36 Ι )2+(ψ+ Ι 20)2

J f ; ψ � Ι ' " " �� '< ' Να προσδιοριστεί ο κύκλος που διέρχεται

από τα σημεία A(l ,O), B(O,l) και εφάπτεται στον κύκλο χ2+(ψ-3)2=4.

Έστω (Κ, ρ ) :

{( χ - χο )2 + (Ψ - Ψο )2 = ρ2 ρ > Ο

ο ζητούμενος κύκλος. A( l ,O) / (μ)

' " " " )'.

" \ " \ / \ 8(0, 1 )

_, Κ(Χο,ψο) i " I " ! / /

Λ(0,3)

Θα υπολογίσουμε τις παραμέτρους Χ0, ψ0, ρ2 . Ο δεδομένος κύκλος έχει κέντρο Λ(Ο,3) και ακτίνα 2.

� Έστω μ η μεσοκάθετος της χορδής ΑΒ . Προφανώς ισχύει:

Κ(χσ,Ψσ) ε μ ( Ι ) � Εφόσον A( l ,O) σημείο του κύκλου (Κ,ρ)

ισχύει: ( 1 -Χο)2+ψο 2=ρ2 (2)

� Εφόσον οι κύκλοι (Κ, ρ), (Λ,2) εφάπτονται, ισχύει:

d(Κ,Λ)2=(ρ+2)2 ή d(Κ,Λ)2=(ρ-2)2 (3) � Λύνοντας το σύστημα των ( Ι ) , (2),(3)

προσδιορίζουμε τις παραμέτρους Χ0, ψ0, ρ2 Ακολουθεί η εκπλήρωση των βημάτων του

σκεπτικού . ,, Εξίσωση της μεσοκαθέτου μ:

Μ(χ,ψ) ε μ<=>ΜΑ=ΜΒ<=>ΜΑ2=ΜΒ2 <=>(χ-Ι )2+(ψ-0)2=(χ-0)2+(ψ-Ι )2<=>ψ=χ

Επομένως η συνθήκη ( Ι ) γίνεται: ψ0=Χ0, (4)

,.. Ακόμη η (3) γράφεται: Χ02+(ψ0-3)2=(ρ+2)2 ή Χ02+(ψ0-3)2=(ρ-2)2

"' Το πρόβλημά μας λοιπόν ανάγεται στην επί­λυση των ακόλουθων δύο συστημάτων:

( Ι ) ι�ο-=χ:;2 + ψ� = ρ2 χ � + (Ψο - 3)2 = ( ρ + 2)2

( Ι Ι ) {� ο-=χ:;2 + ψ� = ρ2 χ � + (Ψο - 3 )2 = ( ρ - 2 )2

ι,1 Επίλυση του συστήματος (Ι) {Ψο = Χο <=> ρ2 = 2χΞ -2χο + Ι

χ0 = Ι -ρ {Ψ ο = Χο <=> ρ2 = 2 ( Ι - ρ)2 - 2 ( Ι - ρ ) + Ι

χ0 = Ι - ρ ΕΥΚΛΕΙΔΗΣ Β ' τ.2/48

Page 51: Ευκλειδης Β 62

Μαθη ματικά για την Β ' Λυκείου {Ψο = Χο {(χ0 , ψ 0 ) = (0, 0 ) <=> ρ = 1 <=> ρ = 1 χ0 = 1 - ρ

Επίλυση του συστήματος (Π) {Ψο = Χο {Ψο = Χο ( 11 ) ( 1 - χο )2 + ψ� = ρ2 <=> ρ2 = 2χ� - 2χο + 1

χ� + (Ψο - 3)- = (ρ - 2)2 Χο = 1 + ρ {Ψ ο = Χ ο {Ψ 0 = Χ0 <=> ρ2 = 2 ( 1 + ρ)2 - 2 ( 1 + ρ ) + 1 <=> ρ = - 1

χ0 = 1 + ρ χ0 = 1 + ρ

Η λύση απορρίπτεται διότι η ρ=- 1 (Ο είναι α­δύνατη .

Συμπέρασμα: Το πρόβλημα έχει μοναδική λύ­ση τον κύκλο : χ2+ψ2= 1 .

Να προσδιοριστεί ο κύκλος που διέρχεται από

Έστω μ η μεσοπαράλληλος των ε ι , ε2 . Προφανώς ισχύουν:

.. � Εφόσον A( l ,2) σημείο του κύκλου (Κ, ρ) ισχύει : ( 1 -χο)2+(2-y0)2=ρ2 (3) Από τις (1 ) , (2), (3 ) προσδιορίζουμε τις

' 2 παραμετρους Χο. Υ ο, ρ . Ακολουθεί η εκπλήρωση των βημάτων του

σκεπτικού .

Εξίσωση της μεσοπαραλλήλου μ: Λαμβάνουμε δυο συγκεκριμένα σημεία επί

των (ε ι ), (ε2) αντιστοίχως Μ(Ο,5) ε ε 1 , Ν(Ο, -5) ε ε2

Το μέσον του τμήματος ΜΝ είναι 0=(0,0) και ισχύει 0=0,0) ε μ

Ακόμη επειδή μ!/ε 1 ισχύει λμ=λε δηλαδή λμ=2 Άρα η εξίσωση της μ είναι

y-0=2(x-O) το σημείο A(l ,2) και εφάπτεται στις δυο ευθείες δηλαδή ε1 :2χ-y+5=0 και εz : 2x-y-5=0

μ: y=2χ

Παρατηρώ ότι ε 1 //ε2 'Ε (Κ ) · {( χ - χο )2 + ( Υ - Υο )2 = ρ2 στω , ρ . ρ > Ο ο ζητούμενος κύκλος.

ε ι :2χ-y+5=0

A( l ,2)

-/

/// '

.-/,/

κ

//

//

μ

ε2:2x-y-S=O

Θ λ ' ' 2 α υπο ογισουμε τις παραμετρους χσ, Υο, ρ

Τώρα η εξίσωση ( 1 ) γίνεται Υο=2χο (4)

Ακόμη η (2) γίνεται 1 1 ρ = Ξd ( ε , , ε2 ) = 2d (Μ, ε2 ) =

= _!_ 12 . ο - 5 - 5 1 = J5 2 J22 + (- 1 )2

Δηλαδή ρ2 = 5

Λύση του συστήματος των (3) και (4) {( 1 - χο )2 + (2 - yo )2 = ρ2 Υ ο = 2χο

<=> {( -χ0 )2 + ( 2 - 2χ0 ) 2 = 5 <=> {Χ0 : Ο ή Χ0 = 2 <=> Υ = 2χ Υ ο - 2χο ο ο

<=> (x0 , y0 ) = (Ο, Ο ) ή ( X0 , y0 ) = (2, 4 ) Συμπέρασμα: Το πρόβλημα έχει δυο λύσεις, τους κύκλους χ2+/=5 , (x-2)2+(y-4)2=5

ΕΥΚΛΕΙΔΗΣ Β' τ.2/49

Page 52: Ευκλειδης Β 62

Μαθηματικά για την Β ' Λυκείου

Να προσδιοριστεί ο κύκλος που διέρχεται από το σημείο Α(6,0) και εφάπτεται στις ευθείες ε1 :4χ-3y+8=0 και εz :3x+4y-1 9=0

\1Jση Παρατηρώ ότι ε ι ){ ε2

(Κ, ρ ) : { ( χ - χσ )z + ( Y - Yo )z = ρz ρ > Ο

ο ζητούμενος κύκλος.

\.-) ""

\, \ \ / K('O,yo) )

ε ι :4x-3y+8=0

\� εz : 3x+4y- 1 9=0

Θα υπολογίσουμε τις παραμέτρους χο, Υο, ρ2 ιι l:ια:πηκu. r Εφόσον (Κ,ρ) εφάπτεται στις ε ι . ε2 . ισχύει:

d(Κ ι ε ι )2=d(Κ2 ε2)2= ρ2 ( 1 ) ,- Εφόσον Α(6,0) σημείο του κύκλου (Κ,ρ), ι­

σχύει: (6-Χ0)2+/= ρ2 "" Από τις ( 1 ) και (2) προσδιορίζουμε τις παρα-, 2 μετρους Χο, yo, ρ .

Ακολουθεί η εκπλήρωση των βημάτων του σκεπτικού . ο!! Η ( 1 ) γράφεται:

(4x0 - 3y0 + 8)2 25

• Λύση του συστήματος των (2) και (3 ) ( 6 - Χ0 γ + Υ� = ρ2 (4χ0 - 3y0 + 8 )2 = ( 3χ0 + 4y0 - 1 9)2 (4χ0 - 3y0 + 8 · 1 )2 = 25 [( 6 - χ0 )2 + y� J 2 ( )2 2 ρ = 6 - χσ + Υο

<=> 4x0 - 3y0 + 8 = 3x0 + 4y0 - 1 9 ( 4χ0 - 3y0 + 8)2 = 25 [ ( 6 - χ0 )2 + y� J

2 ( 6 )2 2 ρ = - Χο + Υ ο ή 4χ0 - 3y0 + 8 = - (3χ0 + 4y0 - 1 9)

(4χ0 - 3y0 + 8)2 = 25 [( 6 - χ0 )2 + y� J 2 ( )2 2 ρ = 6 - χο + yo

<=> X0 - 7y0 + 27 = 0 (4χ0 - 3y0 + 8)2 = 25 [( 6 - χ0 )2 + y� J ρ2 = ( 6 - Χ ο )2 + Υ�

ή 7x0 + y0 - 1 1 = 0 (4χ0 - 3y0 + 8)2 = 25 [( 6 - χ0 )2 + y� J 2 ( )2 2 ρ = 6 - χο + Υο

<=> x0 = 7y0 -27 [ 4(7y0 - 27) -3y0 + 8 τ = 25 {[ 6- (7y0 - 27)2 ] + y� }

ή Yo = -7 yo + 1 1

!Ρ2 = ( 6 - Χ0 )2 + Υ�

[ 4χ 0 - 3 ( -7 Χ 0 + 1 1 ) + 8 ]2 = 25 [ ( 6 - Χ 0 )2 + ( -7 χ 0 + 1 1 )2 J ρ = 6- χο + Υσ

<=> X0 = 7y0 -27 j 2 ( )2 2

25y� -262y0 + 689 =Ο( Δ < Ο, αδύνατη) ρ = 6 -χο + yo

ή Χ0 = 7χ0 + 1 1 j 2 ( )2 2

25χ� - 1 1 6χ0 - 132 = 0 66 Χ =-0 25 1 87

2 ( )2 2 ρ = 6 - χο + yo {Χ0 = 2 <=> Υο = -3

ρ2 = 25 ή Υ ο = - 25

<=>

2 1 68 1 ρ =25

Υο =-7χ0 + 1 1 2 ' 66 Χο = η Χο =-25

Συμπέρασμα το πρόβλημα έχει δυο λύσεις, τους κύκλους

( 66 )2 ( 1 87 )2 - ( 4 1 ) 2 χ +- + y +- - -25 25 5

ΕΥΚΛΕ ΙΔΗΣ Β ' τ.2/50

Page 53: Ευκλειδης Β 62

........... ,_ .. ,. r Ι'�lι ... ., Αιι••Ι•ιι

Μαθη ματικά Γενικής Παιδείας Α . Ασκήσεις Στατιστικής

Ν. Ζανταρίδης, Π. Μυταρέλλης, Κ. Παπαδόπουλος

[ ι σαγι•η ι κ ιί. Η Στατιστική αποτελεί ένα από τα τρία κεφάλαια που διδάσκονται και εξετάζονται οι μαθητές της Γ

τάξης του Ενιαίου Λυκείου στα Μαθηματικά Γενικής Παιδείας. Οι έννοιές της ουσιαστικά διδάσκονται για πρώτη φορά και είναι ανεξάρτητες από τις αποκτηθείσες

μαθηματικές γνώσεις των μαθητών. Μέσα από το άρθρο αυτό θέλουμε να πετύχουμε δύο πράγματα. Την κατανόηση και εμπέδωση αυ­

τών των νέων εννοιών με την παράθεση ασκήσεων και τη σύνδεσή τους με μαθηματικές γνώσεις που αποκτήθηκαν στο πρώτο κεφάλαιο του σχολικού βιβλίου .

Πιστεύουμε ότι η καλή γνώση των εννοιών της Στατιστικής αποτελεί ικανό και αναγκαίο εφόδιο για να ανταποκριθεί χωρίς ιδιαίτερη δυσκολία ο μαθητής στις σχολικές και εξεταστικές ανάγκες.

Ά σκη ση l η Σε μία άσκηση που δόθηκε σε ένα διαγώνισμα οι χρόνοι (σε min), που χρειάστηκαν οι μαθητές

για να λύσουν την άσκηση, έχουν ομαδοποιηθεί σε 4 κλάσεις ίσου πλάτους με αντίστοιχες συχνότη­τες 6, 10, 7, 7.

Αν η συνάρτηση r ( χ) = 6( Χ ι - χ) 2 + 10 ( χ2 - χ)2

+ 7 ( χ3 - χ) 2 + 7 ( χ4 - χ ) 2

παρουσιάζει ακρότατο

για χ = 7 με τιμή ακροτάτου 134, όπου χι , χ2 , χ3 , χ4 τα κέντρα των αντίστοιχων κλάσεων, α) να βρεθούν τα κέντρα των κλάσεων και να γίνει ο πίνακας συχνοτήτων β) να βρεθεί η τυπική απόκλιση και ο CV%.

, \ ίJση : α) Είναι γνωστό (ας δούμε την Εφαρμογή 2 του σχολικού βιβλίου σελίδα 98) ότι η συνάρτηση παρου­σιάζει ελάχιστο για:

6χ ι + 1 0χ2 + 7χ3 + 7χ4 7 6χ ι + 1 0χ 2 + 7χ3 + 7χ4 6 1 0 7 7 2 1 0 χ = <::::> = <=> Χ ι + χ2 + Χ3 + χ4 =

Άρα: Επίσης :

6 + 1 0 + 7 + 7 30 <::::> 6χ ι + 1 0 (χ ι + c ) + 7 (χ ι + 2c ) + 7 (χ ι + 3c ) = 2 1 0 <::::> 30χ ι + 45c = 2 1 0 , όπου c το εύρος των κλάσεων.

3 3 Χ ι + -c = 7 <=> χι = 7 - -c . 2 2 f(7)= 1 34<::::> 6 ( χ ι - 7 )2 + 1 0 (χ2 - 7 )2 + 7 (χ3 - 7 )2 + 7 (χ4 - 7 )2 = 1 34

<::::> 6 (χ ι - 7/ + 1 0 (χ ι + c - 7 )2 + 7 (χ ι + 2c - 7/ + 7 ( χ ι + 3c - 7 )2 = 1 34

<::::> 6 (% c)2 + 1 0 (Ξ)2 + 7 (Ξ) 2 + 7 ( 3; ) 2 = 1 34 <::::> c2 = 4 <::::> c = 2 . Άρα χ ι + 3 = 7 <::::> χ ι = 4 , οπότε τα κέντρα των κλάσεων και ο πίνακας συχνοτήτων είναι:

ΕΥΚΛΕΙΔΗΣ Β' τ.2/5 1

Page 54: Ευκλειδης Β 62

Κλάσεις Xi [ - ) 3 - 5 4 5 - 7 6 7 - 9 8

9 - 1 1 1 0 Σύνολο

β) Ε' - Σ Χ; · ν ; 2 1 0 7 ιναι χ = -- =-= ν 30

Μαθηματικά για την Γ Λυκείου

νί Xi ·νi χ 2 I

2 Χ ; ·νί 6 24 1 6 96 1 0 60 36 360 7 56 64 448 7 70 1 00 700

ν = 30 2 1 0 1 604

1 ( 2 102 ) 1 ( 44100) 1 1 =- 1604 -- = - 1604--- =- ( 1604-1470) =- · 134=4 46�s=2, 1 1 30 30 30 30 30 30 '

s Άρα CV = = = 0, 30 ή CV = 30% χ

Σε ένα διαγώνισμα δώθηκαν δύο προβλήματα. Για το πρώτο πρόβλημα οι χρόνοι λύσεις από τους μαθητές έχουν ομαδοποιηθεί σε τέσσερις κλάσεις, όπου το εύρος είναι 1 6 λεπτά και το κέντρο μιας κλάσης είναι 3. Γνωρίζουμε ότι το 50% των μαθητών χρειάστηκε τουλάχιστον 1 Ο min για να λύσει το πρόβλημα, ενώ το 40% χρειάστηκε λιγότερο από 9 min. Επίσης η κεντρική γωνία του κυ­κλικού διαγράμματος που αντιστοιχεί στην πρώτη κλάση είναι 36°. Αν θεωρηθεί ότι οι χρόνοι είναι ομοιόμορφα κατανεμημένοι στις κλάσεις τότε:

α) Να βρεθούν οι κλάσεις της κατανομής και να γίνει ο πίνακας συχνοτήτων. β) Να βρεθεί η μέση τιμή και η διάμεσος του δείγματος. γ) Αν για το δεύτερο πρόβλημα χρειάστηκε 12 min παραπάνω ο κάθε μαθητής, να βρεθεί ο μέ­

σος χρόνος λύσης και των δύο προβλημάτων μαζί.

Ε ' λ ' ' ' ' 1 6 λ ' ' θ λ ' ' 1 6 4 φοσον οι κ ασεις ειναι τεσσερις με ευρος , το π ατος κα ε κ ασης ειναι - = . 4 Αν το 3 ήταν το μέσο της δεύτερης κλάσης, τότε τα άκρα της δεύτερης κλάσης θα ήταν [ 1 , 5) , οπότε

τα άκρα της πρώτης κλάσης θα ήταν [-3 , 1 ), πράγμα που έρχεται σε αντίφαση με τη φύση του προβλή­ματος, αφού δεν νοείται κάποιος να χρειαστεί . . . αρνητικό χρόνο λύσης ! Έτσι το 3 δεν μπορεί να είναι το κέντρο της δεύτερης κλάσης, πόσο μάλλον της 3ης ή 4ης κλάσης, οπότε έχουμε τον ακόλουθο ελλιπή πίνακα:

Χρόνος Κέντρο fi% Fi% [ - ) Xi 1 - 5 3 5 - 9 7 40

9 - 1 3 1 1 1 3 - 1 7 1 5

Αν 36° είναι η κεντρική γωνία του κυκλικού διαγράμματος που αντιστοιχεί στην πρώτη κλάση , τότε : 360 ·fι =36 � fι = 0, 1

Επίσης F2% = 40 � fι% + f2% = 40 � f2% = 40 - 1 0 = 30 . Το 50% χρειάστηκε τουλάχιστον 1 Ο min οπότε το άλλο 50% χρειάστηκε λιγότερο από 1 Ο min. 'Ετσι (F2 +± f3 )% = 50 � 40 + ± f3% = 50 � f3% = 40 . Επίσης, γνωρίζουμε ότι: ( f, + f2 + f3 + f4 )% = 1 00 � 1 0 + 30 + 40 + f4% = 1 00 � f4% = 20 Οπότε, συμπληρωμένος ο πίνακας σχετικών συχνοτήτων είναι:

ΕΥΚΛΕΙΔΗΣ Β' τ.2/52

Page 55: Ευκλειδης Β 62

Μαθηματικά για την Γ Λυκείου

Χρόνος Κέντρο fj% fj Xϊ ·fi [ - ) Xi 1 - 5 3 1 0 0, 1 0,3 5 - 9 7 30 0,3 2 , 1

9 - 1 3 1 1 40 0,4 4,4 1 3 - 1 7 1 5 20 0,2 3 ,0

Σύνολο 9,8 4

β) Η μέση τιμή χ είναι χ = Σχi · fi = 9, 8 i = l Εφόσον το 50% χρειάστηκε χρόνο 1 Ο λεπτά, σύμφωνα με τον ορισμό της διαμέσου το 1 Ο είναι η διάμε­σος.

γ) Έστω Yi οι χρόνοι λύσης του δεύτερου προβλήματος. Τότε ισχύει: Yi = xi + 12 , όπου xi οι χρόνοι λύσης του πρώτου προβλήματος.

Όμως χ = 9, 8 , οπότε γνωρίζουμε ότι y = χ + 1 2 . Έτσι, ο μέσος χρόνος λύσης και των δύο προβλημάτων μαζί είναι:

- ( χ ι + yι ) + (χ2 + y2 ) + . . . + (xv + yv ) Χι + Χ2 + . . . + Xv Υι + y2 + . . . + yv - - 9 8 9 8 12 3 1 6 · t = = + = x + y = , + , + = , Inln ν ν ν

Οι χρόνοι (σε min) που χρειάστηκαν για να προσέλθουν στο σχολείο τους οι μαθητές έχουν ομο­αδοποιηθεί σε κλάσεις ίσου πλάτους. Το πολύγωνο σχετικών συχνοτήτων f;% της κατανομής (Χ) των χρόνων αυτών, έχει κορυφές τα σημεία Α(2, 0), Β(6, 5), Γ(10, ω, Δ(14, ω, Ε(18, f4), Ζ(22, 5), Η(26, 0). Η κατακόρυφη γραμμή με εξίσωση χ = 1 1 διαιρεί το χωρίο που ορίζεται από το πολύγωνο σχετικών συχνοτήτων και τον οριζόντιο άξονα σε δύο ισεμβαδικά χωρία. Ο μέσος χρόνος προσέλευ­σης είναι χ = 12 min . Θεωρώντας τα δεδομένα ομοιόμορφα κατανεμημένα: α) Να δειχτεί ότι: f2% = 60, f3% = 20, f4% = 10. β) Η κατανομή είναι κανονική ; γ) Να γίνει ο πίνακας σχετικών συχνοτήτων και σχετικών αθροιστικών συχνοτήτων.

Από τα δεδομένα οι κλάσεις είναι: [9,8) , [8 , 1 2) , [ 1 2 , 1 6), [ 1 6 ,20), [20,24] . α) Εφόσον η κατακόρυφη γραμμή χ = 1 1 χωρίζει το πολύγωνο σε δύο ισεμβαδικά χωρία, τότε το 50% των παρατηρήσεων είναι μικρότερες από 1 1 και το άλλο 50% είναι μεγαλύτερες από το 1 1 . Συνεπώς η δ = 1 1 είναι η διάμεσος.

, 3 3 4 · 45 Ετσι: 5 + -f2% = 50 � -f2% = 50 - 5 � f2% = -- = 60 4 4 3

{��·��·--�����-----·�· 2 6 1 0 1 4 1 8 22 26 Ο μέσος χρόνος προσέλευσης είναι χ = 1 2 min άρα:

Επίσης, ισχύει:

5 Σχ/ί = 1 2 � 6 ·ο,ο5 + 1 0 ·0,6 + 14 ·f3 + 1 8 ·f4 + 22.ο,ο5 = 1 2 � i = l

�0,3 + 6 + 1 4 ·f3 + 1 8 ·f4 + 1 , 1 = 12 � 14 ·f3 + 1 8 ·f4 = 4,6 ( 1 ) fι + f2 + f3 + f4 + fs = 1 � 0,05 + 0,6 + f3 + f4 + 0,05 = 1 � f3 + f4 = 0,3 (2)

Λύνοντας το σύστημα των εξισώσεων ( 1 ) και (2), καταλήγουμε ότι: f3 = 0,2 και f4 = 0, 1 οπότε: f3% = 20 και f4% = 1 0 . β) Η κατανομή δεν μπορεί να είναι κανονική , γιατί η διάμεσος (δ = 1 1 ) και η μέση τιμή ( χ = 1 2 ) είναι διαφορετικές. γ) Ο πίνακας αθροιστικών σχετικών συχνοτήτων είναι:

ΕΥΚΛΕΙΔΗΣ Β' τ.2/53

Page 56: Ευκλειδης Β 62

Μαθηματικά για την Γ Λυκείου

Κλάσεις fj% F;% [ - ) 4 - 8 5 5 8 - 1 2 60 65 1 2 - 1 6 20 85 1 6 - 20 1 0 95 20 - 24 5 1 00

λσικηση 4η Στον παρακάτω φαίνονται οι χρόνοι (σε min) που χρειάστηκαν για να λύσουν μια άσκηση οι

μαθητές ενός σχολείου. Γνωρίζουμε ότι το 85% των μαθητών χρειάστηκαν λιγότερο από 16 min για να λύσουν την άσκηση, ενώ το 35% των μαθητών χρειάστηκε τουλάχιστον 1 2 min για να λύσουν την άσκηση.

Χρόνος Σχετική συχνότητα fj%

4 - 8 8 - 1 2 5 1 2 - 1 6 1 6 - 20 1 0 20 - 24

α) Να συμπληρωθεί ο πίνακας β) Να βρεθεί η μέση τιμή και η διάμεσος .

. \ίJση : α) Εφόσον το 85% των μαθητών χρειάστηκε λιγότερο από 1 6 min για ναλύσει την άσκηση , τότε το 1 5% χρειάστηκε τουλάχιστον 1 6 min.

Έτσι, έχουμε: f4% + fs% = 1 5 <::::> fs% = 1 5 - 10 = 5 => fs% = 5 . Επίσης, το 3 5% χρειάστηκε τουλάχιστον 1 2 min για να λύσει την άσκηση , οπότε το 65% χρειάστηκε

λιγότερο από 1 2 min. Έτσι: f1% + f2% = 65 => f1% = 65 - 5 = 60. Ακόμη : f1% + f2% + f3% + f4% + fs% = 1 00 => f3% = 20 Έτσι, ο πίνακας συμπληρωμένος είναι:

Χρόνος Σχετική συχνότητα f;%

4 - 8 60 8 - 1 2 5 1 2 - 1 6 20 1 6 - 20 1 0 20 - 24 5 Σύνολο : 1 00

5 β) Η μέση τιμή ξέρουμε ότι δίνεται από τη σχέση : χ = Σ χ ; · f; , όπου χ; είναι τα κέντρα των κλάσεων,

i = l οπότε : χ = 6 ·0,6 + 1 0 ·0,05 + 1 4 ·0,2 + 1 8 ·0 , 1 + 22·0,05 = 3 ,6 + 0,5 + 2 ,8 + 1 ,8 + 1 , 1 = 9,8 min.

Ο πίνακας αθροιστικών σχετικών συχνοτήτων είναι: Κλάσεις fj% F;%

Γ - ) 4 - 8 60 60 8 - 1 2 5 65 1 2 - 1 6 20 85 1 6 - 20 1 0 95 20 - 24 5 1 00

ΕΥΚΛΕΙΔΗΣ Β ' τ.2/54

Page 57: Ευκλειδης Β 62

Μαθηματικά για την Γ Λυκείου

Το πολύγωνο αθροιστικών σχετικών συχνοτήτων είναι: (F,%)

80

60 50 40 20

y8 δ

1 2 1 6 20 24 min Από το διάγραμμα βλέπουμε ότι τα τρίγωνα ΑΒΓ και ΑΔΕ είναι όμοια (κοινή γωνία και ορθογώνια),

, , ΒΓ ΑΒ ΒΓ 50 1 Ο οποτε εχουμε : - = - <:::::> - = - <:::::> ΒΓ =- . ΔΕ ΑΔ 4 60 3

ο ' δ ' ' δ 4 1 Ο 22 Ε ' ' θ ' ' ' ποτε η ιαμεσος ειναι: = + - = - . πισης και με το «ματι» α μπορουσε απο το σχημα να ε-3 3

κτιμηθεί ότι δ = 7 .

Ασκηση 5η Κάποιος μαθητής παρουσίασε το παρακάτω κυκλικό διάγραμμα

μιας μεταβλητής Χ. Όμως ένας άλλος μαθητής, αφού το μελέτησε είπε ότι υπάρχει κάποιο λάθος στο κυκλικό διάγραμμα. Θα συμφω­νούσατε με τη γνώμη του δεύτερου μαθητή ; Δικαιολογείστε την α­πάντησή σας.

�--�-=-�� ' \ I '

/ \ \

', 1 20°

30'!

Έστω ν το μέγεθος του δείγματος στο όπιο αναφέρεται το πρόβλημα. Α ν ήταν σωστό το κυκλικό διάγραμμα, τότε θα ίσχυε ότι η κεντρική

γωνία που αντιστοιχεί στις τιμές χ2 και χ5 αθροιστικά είναι: 360 - 30 - 1 20 - 30 = 1 80 . ν ν 43 + 86 Τότε : ω2 + ω5 = 1 80 <:::::> 360-5 + 360-5 = 1 80 <:::::> 360 · = 1 80 <=:>ν = 258 . ν ν ν

Τότε, για τη συχνότητα της τιμής χ , θα είναι: ω1 = 360� <:::::> ν

i

30°

30 360 ν1 30 · 258 258 lM , , , δ ' δ ' θ ' = · -- <:::::> ν 1 = = -- !i>: 1 -..:ι ατοπο αρα εχει ικιο ο ευτερος μα ητηc . Άσκη ση 6η

258 360 1 2 -

-

ν,=86

Τα διπλανά σχήματα είναι τα πολύγωνα συχνοτήτων και αθροιστικών σχετικών συχνοτήτων και παρουσιάζουν το χρόνο που χρειάστηκαν μαθητές για να λύσουν μια άσκηση.

(ν,) (F,%) β ---- \ 00 1------------- - - - - - - - - - - - - - - - - - - - - - - - - - - - - - - -------- - - - · · · · · · ·- - - - - - - - - - - - - - - - - - - - - - - - - - - - - - - - - --ιι

1 5 ---

1 0

α ----- - - - - - - - - - - - - - - - - - - - - - - - -- - - -

2.5 min

70 ....

30 '''

γ

α) Να βρεθούν οι αριθμοί α, β, γ και να γίνει ο πίνακας συχνοτήτων. β) Να βρεθεί το εμβαδό του πολυγώνου συχνοτήτων.

ΕΥΚΛΕΙΔΗΣ Β' τ.2/55

25 min

Page 58: Ευκλειδης Β 62

Μαθηματικά για την Γ Λυκείου

γ) Πόσοι μαθητές θα χρειαστούν τουλάχιστον 1 7 λεπτά για να λύσουν την άσκηση ; Θεωρούμε τα δεδομένα ομοιόμορφα κατανενημένα.

Έστω ν το μέγεθος του δείγματος στο οποίο αναφέρεται το πρόβλημα. Τα δεδομένα έχουν ομαδοποιηθεί σε τέσσερις κλάσεις. Αν c το εύρος κάθε κλάσης, τότε από τα

διαγράμματα προκύπτει ότι : 25 - 2,5 = 4c + � � 22,5 = 9c � 45 = c � c = 5 . 2 2 9 Έτσι, έχουμε τον ελλιπή πίνακα συχνοτήτων και αθροιστικών σχετικών συχνοτήτων:

Κλάσεις Συχνότητα Σχετική Αθροιστική Σχετική αθρ. [ - ) ν; συχνότητα συχνότητα συχνότητα

ti Ν F;% 5 - 1 0 α γ 1 0 - 1 5 1 0 30 1 5 - 20 β 70 20 - 25 1 5 1 00

Όμως, Επίσης,

F3% = (F2% + f3%) � 70 = 30 + f3% � f3% = 40.

Τότε, από την ισότητα:

Επίσης,

0 ν4 1 5 ω�σ = - · 1 00 � 30 = - · 1 00�ν = 50.

4

ν ν f3% = 40 � 1._ · 1 00 = 40 � β = 20 50

Σ ν; = 50 � α + 1 0 + 20 + 1 5 = 50�α = 5 i = l

ν 5 τότε : F 1% = f1% = _!_ · 1 00 � γ =- · 1 00 � γ = 1 0 ν 50 Ο πίνακας συχνοτήτων συμπληρωμένος είναι:

Κλάσεις Γ - ) ν; ti% 5 - 1 0 5 1 0 1 0 - 1 5 1 0 20 1 5 - 20 20 40 20 - 25 1 5 30 Σύνολο 50 1 00

Το εμβαδό του πολυγώνου συχνοτήτων είναι γνωστό ότι ισούται με το μέγεθος του δείγματος, οπότε Ε = ν = 50 .

Επειδή οι τιμές κατανέμονται ομοιόμορφα σε κάθε κλάση , τότε στο διάστημα 1 7-20 αντιστοιχεί στα � της κλάσης [ 1 5 , 20) , επομένως το πλήθος των μαθητών στο διάστημα 1 7-20 θα είναι � ·20 = 1 2 . 5 5

Άρα τουλάχιστον 1 7 λεπτά θα χρειαστούν 1 5 + 1 2 = 27 μαθητές.

Το βάρος του κάθε μαθητή μιας τάξης Λυκείου είναι τουλάχιστον 45 κιλά, αλλά μικρότερο από 85 κιλά. Το 90% των μαθητών έχει βάρος τουλάχιστον 55 κιλά, 1 5 μαθητές έχουν βάρος μικρότερο από 65 κιλά, 35 μαθητές έχουν βάρος τουλάχιστον 65 κιλά και το 60% των μαθητών έχει βάρος λιγότερο από 75 κιλά.

α) Να παρασταθούν τα δεδομένα σε ένα πίνακα συχνοτήτων, σχετικών συχνοτήτων, αθροιστικών συχνοτήτων, και σχετικών αθροιστικών συχνοτήτων.

β) Ν α βρείτε το ποσοστό των μαθητών που έχουν βάρος τουλάχιστον 60 κιλά, αλλά λιγότερο από 80 κιλά.

γ) Να βρεθούν οι γωνίες των αντίστοιχων κυκλικών τομέων του κυκλικού διαγράμματος σχετικών

ΕΥΚΛΕΙΔΗΣ Β' τ.2/56

Page 59: Ευκλειδης Β 62

Μαθηματικά για την Γ Λυκείου

συχνοτήτων για τα δεδομένα του προβλήματος.

Από τα δεδομένα θα μπορούσαμε ότι μπορούμε να ομαδοποιήσουμε τα στοιχεία σε τέσσερις κλάσεις που θ ' θ ' ' 85 - 45 1 0 η κα εμια α εχει ευρος: 4 = . Εφόσον λιγότερο από 65 κιλά είναι 1 5 μαθητές και 35 έχουν βάρους τουλάχιστον 65 κιλά, προκύπτει

ότι το σύνολο των μαθητών είναι 1 5 + 35 = 50 Επίσης το 90% έχει βάρος τουλάχιστον 55 κιλά, οπότε το 1 0% έχει βάρος λιγότερο από 55 κιλά.

Άρα f1 % = � · 1 00 <::> 1 0 = � · 1 00 <::> ν 1 = 5 . ν 50 1 0 Οπότε, από Ν2 = 1 5 <::> 5 + ν2 = 1 5 <::> ν2 = 1 0 και f2 % =- · 1 00 = 20 . 50

Από F3% = 60 <::> 10 + 20 + f3% = 60 <::> f3% =30<::> � · 1 00 = 30 <::>ν3 = 1 5 . 50 Έτσι έχουμε συμπληρωμένο τον πίνακα ως εξής:

Βάρος Συχνότητα Σχετική Αθροιστική Σχετική αθρ. [ - ) νί Συχνότητα συχνότητα συχνότητα

fi% Ν Fi% 45 - 55 5 1 0 5 1 0 55 - 65 1 0 20 1 5 30 65 - 75 1 5 30 30 60 75 - 85 20 40 50 1 00 Σύνολο : 50 1 00

Θεωρώντας τα βάρη των μαθητών ομοιόμορφα κατανεμημένα και επειδή το 60 είναι το κέντρο της 2ης κλάσης και το 80 της 4ης κλάσης, δεχόμαστε ότι οι μισοί μαθητές της 2ης κλάσης έχουν βάρος άνω των 60 κιλών και οι μισοί της 4ης έχουν βάρος κάτω των 80 κιλών, οπότε το ποσοστό των μαθητών που έχουν βάρος τουλάχιστον 60 κιλά, αλλά λιγότερο από 80 κιλά είναι: 1 0% + 30% + 20% = 60%.

Οι γωνίες των αντίστοιχων κυκλικών τομέων του κυκλικού διαγράμματος σχετικών συχνοτήτων για τα δεδομένα του προβλήματος είναι: α; = 360 f; % = 3, 6° · f; % οπότε: α 1=3 ,6° · 1 0 = 36° , α2=3 ,6° ·20=72° , α3=3 ,6° ·30= 1 08°, α4 =3 ,6° -40= 1 44° . 1 00

Έστω χ1 < χ2 < χ3 < χ4 οι τιμές μιας μεταβλητής Χ και F; οι αθροιστικές σχετικές συχνότητες · 2 4

της μεταβλητής. Αν ισχύει: F; = � όπου ί = ι , 2, 3, 4. κ

α) Να βρεθεί η τιμή του κ. β) Να βρεθούν οι σχετικές συχνότητες f; όπου ί = ι, 2, 3, 4. γ) Να βρεθεί το ποσοστό των παρατηρήσεων που έχουν τιμή τουλάχιστον χ3•

α)Εφόσον Fi είναι οι αθροιστικές σχετικές συχνότητες και: χ 1 < χ2 < χ3 < χ4 οι τιμές της μεταβλητής Χ, 42 + 4 τότε : F4 = 1<::> -- = 1 <::>k = 20.

1 + 4 5 β) Για i = 1 έχουμε : fι = F ι = -- =- = Ο 25 20 20 ,

κ

Για i = 2 έχουμε: F = 4 + 4 _ _!_ οπότε: f2 = F2 - F 1 = 8 - 5 = _2_ = 0 1 5 2 20 - 20 20 20 ,

9 + 4 1 3 ' 1 3 - 8 5 Για i = 3 έ1ουμε: F = ---- οποτε : f3 = F3 - F2 = --=-= 0 25 3 20 - 20 20 20 ,

ΕΥΚΛΕΙΔΗΣ Β ' τ.2/57

Page 60: Ευκλειδης Β 62

Μαθηματικά για την Γ Λυκείου

' 20 1 3 7 Για ί = 4 έχουμε: F4 = 1 συνεπως: f4 = F4 - F3 = 20 - 20 = 20 = 0, 3 5 γ) Το ποσοστό των παρατηρήσεων που έχουν τιμή τουλάχιστον χ 3 είναι :

f3% + f4% = 25% + 35% = 60%

Π. Χριστόπουλος

Αν t ι , t2, . . . tν οι παρατηρήσεις μιας μεταβλητής Χ που ακολουθεί την κανονική κατα-

, - . f { ) s . ( ) , ( ) ( ) 2Χ2 - 3χ + 1 νομη με x = lιm χ , = lιm g χ , οπου f χ = 4ημχ - 3συνχ , g χ = ----=-

2 --

, .... � χ --. 1 χ - χ 2

Εξετάστε αν οι παρατηρήσεις είναι ομοιογενείς και να ευρεθεί το ποσοστό των παρατη­ρήσεων που βρίσκονται στο διάστημα [2g(2), 3g (3)]

Έχουμε ;: = lim f ( 4ημχ - 3συνχ ) = 4 π χ -->-2

2 2 ( χ - 1 ) χ - - 2 x - -( 1 ) ( 1 ) S = lim 2χ - 3χ + 1 = lim 2 = lim 2 = Ι

χ --> 1 χ2 - χ χ --> 1 χ ( χ - 1 ) χ --> 1 χ - -

Το δοθέν διάστημα είναι [3 , 5]=[ χ - 5, χ + 5 ] Άρα έχουμε 3 4 5

x:-s :χ x+S Οπότε το 68% των παρατηρήσεων είναι στο διάστημα [3 ,5 ] Ο συντελεστής μεταβλητότητας είναι: CV = � = _!_ = 25% < 1 0%, οπότε οι παρατηρήσεις δεν χ 4

είναι ομοιογενείς

Αν f(x) = α · s · χ2 - 3� · χ + s2 , χ ε IR , όπου �' s η μέση τιμή και η τυπική απόκλιση μια με­

ταβλητής Χ. Αν η εφαπτόμενη ευθεία στη γραφική παράσταση της f στο σημείο της A(l , f(l )) είναι

παράλληλη προς τον άξονα χ 'χ. Να βρεθεί η τιμή του α ώστε οι παρατηρήσεις της Χ να έ­

χουν ομοιογένεια

Έχουμε : f'(x) = 2α · S · x - 3x Ισχύει: ε // χ 'χ άρα : f' ( 1 ) = Ο <=> 2α · s · χ - 3� = Ο <=> 2α · s = 3;: <=> !. = � άρα CV = � . Για να χ 2α 2α

έχουν οι παρατηρήσεις ομοιογένεια πρέπει: CV :-ς Ο, 1 <=>� :-ς Ο, 1 <=>� :-ς 1 <=> 30 :-ς 2α <=> I α � 1 5 1 . 2α 2α � � (<'1; [01 :'\'η j Δίνονται οι αριθμοί χ, y, 8, 5, 7 που έχουν μέση τιμή 6 και διακύμανση 2. Να βρεθούν τα

χ, y, αν x>y.

ΕΥΚΛΕΙΔΗΣ Β' τ.2/58

Page 61: Ευκλειδης Β 62

Μαθη ματικά για την Γ Λυκείου

'Ε χουμε � = Σti <:::> Σti = 5 · 6 ή χ + y + 8 + 5 + 7 = 30 <=> χ + y = 1 0 ( 1 ) ν

κ 2 ( χ - 6)2 + ( y - 6)2 + (8 - 6)2 + ( 5 - 6)2 + ( 7 - 6γ α ι s = -'---..:....._---'----'----''-------'---'------''-----'---'--

2 --( χ - 6)2 + ( y - 6)2 + 6 ή 5

5

ή ( χ - 6 )2 + ( y - 6)2 = 4 (2)

Λύνοντας το σύστημα των ( 1 ) και (2) προκύπτει: χ2 - 1 0χ+24=0<:::::>χ=6 ή χ=4

Αν χ=6 τότε y=4 ενώ αν χ=4 τότε y=6 . Τότε δεύτερο ζεύγος τιμών απορρίπτεται.

Η μέση βαθμολογία στην κλίμακα από Ο έως 100 των μαθητών της Α ' περιοχής στις Πανελλήνιες ήταν 80 και η τυπική απόκλιση 5 ενώ των μαθητών της Β' περιοχής ήταν 85 και η τυπικά απόκλιση 2. Σε ποια περιοχή είχαμε το μεγαλύτερο ποσοστό αριστούχων; (Α­ριστείο παίρνει ο μαθητής με βαθμό � 90) Δίνεται ότι βαθμολογία ακολουθεί την κανονική κατανομή

Σύμφωνα με τα γνωστά ποσοστά που ισχύουν στην κανονική , παρατηρώντας την παραπάνω σχηματική παράσταση στην Α ' περιοχής 1 00 ή περισσότερα του από το έχει γράψει το 2 ,5% των μαθητών, Στης Β' περιοχή, ίσο ή περισσότερο του 90 έχει γράψει λιγότερο του 2 ,5% των μαθητών .

Άρα μεγαλύτερο ποσοστό αριστούχων έχουμε στην Α ' περιοχή .

Α'

Β '

80 85 90 +- --

x+S . - ι-·rso;,--·-; χ+25

85 87 89 90 : -- : · · - - · .. �ι--�j-� z:so/;'-� -�-� χ x+S χ+25

Στην Α' περιοχή οι άριστοι είναι 2 ,5% ενώ στην Β' περιοχή λιγότεροι.

Ποια είναι η μικρότερη πεντάδα διαδοχικών ακεραίων που έχει ομοιογένεια.

Έστω ν, ν+ 1 , ν+2, ν+ 3 , ν+4 πέντε διαδοχικοί ακέραιοι αριθμοί, τότε χ = ν + 2 2 [ν - ( ν + 2 )] 2 + . . . + [(ν + 4) - (ν + 2 )τ 1 0 και S = = - = 2 5 5

και J2 ν � - - 2 <:::> ν � 1 2, 1 0, 1

( 1 ) Οπότε ο μικρότερος ακέραιός ν που ικανοποιεί την ( 1 ) είναι ο ν= 1 3 . Επομένως η πεντάδα είναι: 1 3 , 1 4, 1 5 , 1 6 , 1 7

ΕΥΚΛΕΙΔΗΣ Β ' τ.2/59

Page 62: Ευκλειδης Β 62

Μαθηματικά για την Γ Λυκείου

Μ αθη ματικά Κατ·εύθυνση ς λ . �Όρια συνάρτη σης στα :±: οο, μια «άλλη» άποψη . � .

Μαρία Αγιοπούλου - Τάκης Δρούτσας Νίκος - Πανουσάκης

Η εύρεση του ορίου μιας συνάρτησης με ριζικά, στα + οο και - οο, βασίζεται κυρίως στις εξής δύο τεχνικές:

1 . την εξαγωγή του μεγιστοβάθμιου χ, ως κοινού παράγοντα και 2. τον πολλαπλασιασμό με κατάλληλη συζυγή παράσταση, σε περίπτωση aπροσδιοριστίας. Ο συνδυασμός των δύο παραπάνω τεχνικών αν και είναι πολύ αποτελεσματικός, μειονεκτεί

στο γεγονός, ότι κατά την εφαρμογή του απαιτούνται πολλές πράξεις, με αποτέλεσμα, η διαδι­κασία να γίνεται εξαιρετικά χρονοβόρα.

Σκεφτείτε, για παράδειγμα, να ζητάμε το όριο : }i.� ( .J9x2 + χ + 1 + �8χ3 + 2 - .J25x2 + 3 ) !

Η διαδικασία, μπορεί να απλοποιηθεί, αν εφαρμόσουμε την τεχνική του μεγιστοβάθμιου χ σε συνδυασμό με τον κανόνα του De L' Hospital .

Π αράιiε ιγμα 1 . Να βρείτε το όριο : lim (.J9x2 + χ + ι + �8χ3 + 2 - .J25x2 + 3 )

Χ-Η«>

Απιί.ντη ση • Εφαρμόζουμε τη «μέθοδο του μεγιστοβάθμι­ου όρου χ». Για κάθε χ ε (0, + οο) , έχουμε : f(x) = ( .J9x 2 + χ + 1 + �8χ3 + 2 - .J25x 2 + 3 ) ή

f(x) = x [�9 +.!_ + 1 + �8 + 2 · 1 - �25 + 3 · 1 J ( 1 χ χ2 χ3 χ2 • Θέτουμε _!_ = h <::::> χ = _!_ (h > Ο) χ h

Τότε η ( 1 ) γράφεται: r ( � ) = � (.J9 + h + h2 + �8 + 2h3 - .J25 + 3h2 )

• Υπολογίζουμε το όριο εφαρμόζοντας τον κα-νόνα του De L' Hospital .

1im f ( χ ) = lim f (!) = χ ---++οο h--+0+ h

. .J9 + h + h2 + �8 + 2h3 - .J25 + 3h2 (%) = 1ιm = h ->0+ h . (.J9 + h + h2 + �8 + 2h3 - .J25 + 3h2 ) '

= 1ιm . = h ->0+ ( h ) '

1 + 2h 6h2 6h -ι====�+ - -�====

. 2.J1 + h + h2 3�(8 + 2h3 / 2.J25 + 3h2 = 1ιm --------'---'-----=------

,, .... ο · 1 1 1 = - + 0 - 0 = -2 2

Π αρ(ιδειγμα 2 . Να βρείτε το όριο :

I . 2x + l - .J4x2 + x + l ιm ----r=====->->+oo 3x + l - �27x3 + 1

Λπ{ιντηση • Για κάθε χ ε (Ο, + οο) , έχουμε:

1 �

r ( x ) = 2 + ;z - ν 4 + ;z +7

3 + .!.._ - �27 + 1

e Θέτουμε χ χ3

1 1 - = h <=> χ = -(h > 0) χ h τότε η ( 1 ) γράφεται

f (_!_) = 2 + h - .J 4 + h + h 2 h 3 + h - �27 + h3

• Υπολογίζουμε το όριο :

( 1 )

1im f (!) = lim 2 + h - .J 4 + h + h 2 (�

h ->O+ h h ->O+ 3 + h - �27 + h3 . ( 2 + h - .J4 + h + h2 )'

= 1ιm , = h ->O+ ( 3 + h - �27 + h3 )

ΕΥΚΛΕΙΔΗΣ Β ' τ.2/60

Page 63: Ευκλειδης Β 62

Μαθηματικά για την Γ Λυκείου

Επομένως: lim f ( χ ) = lim f (!) = � Χ --++Χ )1 --+0+ h 4

i ! ιφ ι:αιηγ�ω �( Να βρείτε τις τιμές των παραμέτρων

α, β Ε R , για τις οποίες ισχύει:

,ιl� {\Ι ι - χ3 + vf χ2 - 2χ - αχ + β) = ι . \ π ιlντηση

• Για κάθε χ Ε ( -οο, Ο) , έχουμε: f(x) = \11 - x3 + vfx2 - 2χ - αχ + β <=>

.;; f(x) = -x ( v:, + l + Jl - 2 · � + α - β · � J ( l ) Είναι lim (-χ ) = +οο και

χ � -χ

lim ( �- \ + 1 + J 1 - 2 · _!_ + α - β · _!_J = 2 + α χ --+--:ι: χ χ χ

;ι Διακρίνουμε τις εξής περιπτώσεις : i) α:;t:-2 και i i) α = - 2 . {+οο, α > -2 i) Αν α :;t:-2, τότε : lim f ( χ ) =

χ --+--οο -οο, α < -2 ii) Αν α = -2, τότε θέτοντας

1 1 - = h <=> x = -, (h < O) χ h η ( 1 ) γίνεται:

r (.!_) = � +.J1=2h + α - βh h -h

Οπότε: ( 1 ) \ft - h3 + νi1 - 2h + α - βh (%) lim f ( χ ) = lim f - = lim =

χ --+--οο h--+0-- h h--+0-- -h ο ( \11 - h3 + νl1 - 2h + α - βh ) '

= lιm �----,---:-------'-h --+ o -- ( -h) ι

-3h2 -2 -------;=== + - β

. 3Ψ 1 - h3 )2 2νt1 - 2h = l ιm = Ο + 1 + β = 1 + β

h --+ � - 1 Επομένως

i)

i i)

i i i)

lim f ( χ ) = 1 <=> {α = -2 }<=> {α = -2 χ--+--οο 1 + β = 1 β = Ο

Ασκήσεις π ο υ :ιη::ι uτεί·Ι ' Οlψ <' Να υπολογίσετε τα παρακάτω όρια:

lim (\18χ3 + χ2 - \18χ3 - χ2 ) Χ 4+00

lim {vfx2 + χ + 1 + vfx2 - χ - 2χ ) x ---t+oo

1 Απ. : -6 Απ. : Ο

lim {vf9x2 + χ + vfx2 + 2χ - vf1 6x2 + 3 ) χ --++οο

1 Απ. : -6

Β .. Παράγωγος Συνά ρτηση ς

1 . Παράγωγο συνάρτησης βρίσκουμε μόνο στα σημεία του πεδίου ορισμού της ( σε εκείνα που είναι συνεχής ) και μάλιστα :

i ) . Σε εσωτερικά σημεία αυτού χρησιμοποιούμε τους τύπους παραγώγισης (η συνάρτηση είναι συνεχής σε αυτά), ενώ

i i ) . Σε κλειστά άκρα (που μετά από εξέταση η συ­νάρτηση είναι συνεχής), η παράγωγος βρίσκε­ται μόνο με τον ορισμό.) (Στην περίπτωση μερικών συναρτήσεων , ενώ έχουμε πεδίο ορισμού το R θα μπορούσαν να θεωρηθούν όλα τα σημεία εσωτερικά , στην διαδικασία όμως παραγώγισης είμαστε ανα­γκασμένοι να χωρίσουμε το R σε διαστήματα

Δαφνής Δημήτρης - Δαφνής Σπύρος

π. χ f(x)= \[;1 κ άρτιος φυσικός ) il ρ οσοzή : Όταν έχουμε παραγώγιση σύνθετων συναρτήσεων καλύτερα να χρησιμοποιούμε : ι . Τον τύπο : [(fog)(x)] '=f(g(x)] ·g ' (x) ( Σύνθεση

δύο συναρτήσεων ) ( Πρέπει η g να είναι παραγωγίσιμη στο χο και η f παραγωγίσιμη στο g(xo) )

1 1 . Τον κανόνα της αλυσίδας, για πάνω από δύο συναρτήσεις

dψ dψ du dk dz -=-X-X-X-dX du dk dz dx

2 ο Η συνάρτηση f(χ)=χα παραγωγίζεται : ι Στο R με f' (χ)=α·χα- ι αν α ε Ν-{ 0, 1 }

ΕΥΚΛΕΙΔΗΣ Β ' τ.2/6 1

Page 64: Ευκλειδης Β 62

Μαθη ματικά για την Γ Λυκείου

Στο R * με f'(χ)=α·χα- 1 αν α Ε Ζ *

1 ι Στο (Ο,+ οο ) με f' (χ)=α·χα- 1 αν α Ε R-Ζ � ιδ ι ΙJοΊ περn:πτωση στην i i i .. : Αν α> 1 η συνάρτηση f : f(x)= χα με α Ε R-Z

παραγωγίζεται στο [0,+ οο )

3 " Ισχύει : (ex) '=ex Π ροrωzή : Για να παραγωγίσουμε εκθετική

συνάρτηση που δεν έχει βάση το e μετατρέπουμε αυτή με την βοήθεια του τύπου : αχ=eχιηα σε ισο­δύναμη σχέση με βάση e , και ισχύει (αχ) '=αχlηα.

"" Ο τύπος αυτός ισχύει μόνο αν α σταθερός πραγματικός αριθμός

Θ Σε κάθε άλλη περίπτωση η παραγώγιση πρέπει να γίνεται αναλυτικά. 1 1

"ι Ισχύουν : ( lnx ) '=-; χ>Ο ( lnixi ) ' = -; χ Ε R * f'(x)

και γενικά : ( ln[f(x)] ) '= f(x) f(x) :;t: Ο Π ροσοχΈ1 : Για να παραγωγίσουμε λογαριθμι­

κή συνάρτηση που δεν έχει βάση το e μετατρέπου­με αυτή με την βοήθεια του τύπου αλλαγής βάσης: I lnx δ ' β οgαχ= - σε ισο υναμη σχέση με άση e και Ιηα ισχύει : (logαx) '= -

1 -xlnα Α 1 ο Να βρεθεί ί. η πρώτη και ii. η δεύτερή

παράγωγος της συνάρτησης f :

f(x)=

. HΙ C:!j

χ3 + 3χ 2 + 2 αν χ ε (-2 , Ι ) χ4 + 5χ χ2 + 22 2χ 2 + 3χ

αν χ ε [ Ι ,2) αν χ ε [2 ,3 ) αν χ ::::: 3

ί. Εύρεση της f' Πεδίο ορισμού της f : Ar=(-2 , +οο )

� Αν χ Ε (-2, Ι ) τότε : f'(x)=3x2+6x � Αν χ ε ( 1 ,2) τότε : f'(x)=4x3+5 � Αν χ Ε (2,3) τότε : f'(x)=2x � Αν χ>3 τότε : f'(x)=4x+3 � Θα βρούμε αν υπάρχει η παράγωγος της

συνάρτησης στα κλειστά άκρα : χ 1 = Ι ,χ2=2 και χ3=3

lim f(x)-f( I ) = lim f(x)-f( l ) =9 τότε : f' ( l )=9 χ�ι+ χ- Ι χ�1 - χ- Ι ο Στο χ2=2 η f είναι συνεχής και ισχύει :

lim f(x)-f(2) :;t: lim f(x)-f(2) x�z · χ-2 x�z- χ-2 Η f δεν παραγωγίζεται στο χ2=2 ο Στο χ3=3 η f δεν είναι συνεχής ( η f δεν παρα­

γωγίζεται στο χ3=3 ) 3χ 2 + 6χ αν χ ε (-2 , 1 )

Άρα f'(x)= 4χ3 + 5 αν χ ε [ Ι ,2) 2χ αν χ ε (2,3)

4χ + 3 αν χ>3 ίί. Εύρεση της f' ' . Πεδίο ορισμού της f' : Ar =(-2 , Ι ) υ [ 1 , 2) υ (2,3 ) υ (3 ,+ οο )

� Αν Χ Ε (-2 , Ι ) τότε : f"(x)=6x+6 � Αν χ Ε ( 1 ,2) τότε : f"(x)= 1 2x2 � Αν χ Ε (2,3) τότε : f"(x)=2 � Αν χ>3 τότε : f" (χ )=4 � Θα βρούμε αν υπάρχει η παράγωγος της

συνάρτησης στο κλειστό άκρο : xa= Ι ο Στο χ 1 = Ι η Γ είναι συνεχής και ισχύει:

lim Γ(χ)-Γ( Ι ) :;t: lim Γ(χ)-Γ( Ι ) χ�ι- χ- Ι χ�ι - χ- Ι δεν υπάρχει η δεύτερη παράγωγος της f στο xa= I

6χ + 6 αν χ ε (-2, Ι ) Άρα f"(x)= αν χ ε (1 ,2)

αν χ ε (2,3) αν χ>3

/�"- Ί " Να βρεθεί η παράγωγος της συνάρτη­σης [ : { 3 Ι

f(x)= χ ημ-; + 3χ αν χ :;t: Ο Ο αν χ=Ο

Πεδίο ορισμού της f : Ar=R � Αν x :;t: Ο Γ(χ)=3χ2ημ � +χ3συν � (-:2 ) =

Ι 1 2 - -=3χ ημ χ - χσυν χ ο Στο χ 1= Ι η f είναι συνεχής και ισχύει : � Α ν χ=Ο η f είναι συνεχής και

ΕΥΚΛΕΙΔΗΣ Β' τ.2/62

Page 65: Ευκλειδης Β 62

Μαθηματικά για την Γ Λυκείου

3 1 χ ημ- + 3χ Γ(Ο)= lim f(x)-f(O) = 1im χ

χ --->0 χ-0 χ--->0 χ 1 = 1im (χ2ημ � +3)=3

χ _,ο-, , 3χ-ημ- - χσυν- αν χ :;ι: Ο { , I 1 Άρα f (χ)= χ χ

3 αν χ=Ο

Λi . Να βρεθεί η παράγωγος των συναρτή­σεων f , ιη , σ , k . η , και g με τύπους :

i. f(x)=x200� . χ ε R ii. m(x)=x-5 , χ ε JR. * 3 -

iii. σ(χ)= χ � . χ ε :i- ίν. k(x)= χ 2 x ε JR. * ν. η( χ)= � χ ε �- και νί. g(χ)=ρ

\ ι .; η • -h( ) · σο� Π � ,

, R ι. 1 1 χ = χ- εοιο ορισμου το

� Μορφή χα με α Ε Ν- { Ο, I }

Άρα : f' ( x )=2004x2003 για κάθε χ Ε R

ii. ιη( χ )=χ -5 Πεδίο ορισμού το R * � Μορφή χ-{! με α Ε Ν*

Άρα : ιη ' ( χ )=-Sχ-6=---;- για κάθε χ Ε R* χ -

iii. σ( χ)= χ � Πεδίο ορισμού το R +

� Η συνάρτηση σ παραγωγίζεται στο 3 �- 1 3 _ _!_ 3 (0 .- χ ) με σ ' (χ)=- χ 4 = - χ 4 =--4 4 4 -�

(Μορφή χα με α=i Ε R-Z και α< 1 ) 4

3

ίν. k(x)= χ 2 Πεδίο ορισμού το R+

� Η συνάρτηση σ παραγωγίζεται στο 3 �-1 _!_ 3 [Ο,+ οο ) με κ ' (χ)=2 χ 2 = χ 2 =2-f;.

3 �-1 3 _ _!_ 3 με κ '(χ)= - χ 4 = - χ 4 =-- . 4 4 4� (Μορφή χα με α=i Ε R-Z και α> 1 ) 2 ν. κ( χ)= rx Πεδίο ορισμού το Α=[Ο,+ οο ) 1 oc . τρ{ιπος

� Βρίσκουμε με τον ορισμό της παράγωγο της συνάρτησης σε τυχαίο χ0 Ε (0,+ οο ) και στη συνέ­χεια εξετάζουμε αν υπάρχει η παράγωγος στο χ0=0

ο 1 0 η(χ)-η(χσ ) 1 " rx-rx: � ιm = ιm 1im χ-->χο Χ-Χ ο χ-->χο Χ-Χ ο χ --> χο

crx-ψ;xif;J +rχψ; +rxr) _

(x-x0 )(if;J �ψ;+ F:[) -lim 1 =-1-χ---> χο ({(;!�ψ;+ F7) 3F:f Άρα η ' (χ)= ,� χ Ε (Ο,+ οο ) 3{/ χ2

� Στο χ0=0 η συνάρτηση η δεν παραγωγίζεται , 1 . η(χ)-η(Ο) R γιατι ιm � Χ--->0 Χ-0 2"c . τρ1iπος

k

� Από την ιδιότητα ι,;J;:! = χ-;; χ � Ο έχουμε : ι

rx = χ 3 για κάθε χ � ο _!_ 1 _!__ ,

� η ' (χ)=( rx ) '=( χ 3 ) '= - χ 3 = 3 1 2 1 1 = - χ 3 =-2 =-- χΕ (Ο,+ οο ) 3 3if;J 3χ 3

(Μορφή χα με α Ε R-Z και α< 1 )

νί. g(χ)=ρ Πεδίο ορισμού το R

I οc .τρόπος.

Βρίσκουμε με τον ορισμό της παράγωγο της συνάρτησης σε τυχαίο χ0 Ε (Ο,+οο ) και στη συνέ­χεια εξετάζουμε αν υπάρχει η παράγωγος στο χο=Ο . (Διαδικασία πολύπλοκη παράδειγμα (ν))

2 "c . τρ{)πος k

� Από την ιδιότητα ι,;J;:! = χ-;; χ � Ο έχουμε : 6

ο ρ = χ7 για κάθε χ � Ο και 6

ο ρ =�(-χ)6 =(- χ)Ί για κάθε χ ::; ο και τότε :

Ί Π � 6 �-1 6 .::.!_ • g . (χ)=( ν χ - ) . =(χ 7 ) • = _ χ 7 = _ χ 7 = 7 7

6 =-- χΕ (Ο,+ οο ) 7zΓχ. 6 g ' (x)=( ρ ) '=( �(-χ)6 ) '=[(-χ) 7 ] '=

6 �-1 6 .::.!_ 6 =- (-χ) 7 (-χ) '= - (-χ) 7 (- 1 )=---7 7 7Γχ χ Ε (-οο ,Ο)

ΕΥΚΛΕΙΔΗΣ Β ' τ.2/63

Page 66: Ευκλειδης Β 62

Μαθηματικά για την Γ Λυκείου

(Μορφή χα με α=� Ε R-Z και α< 1 ) 7

τ , ' ( ) 7zΓχ. ! -6- αν χ Ε (Ο ,+οο)

οτε : g χ = _ _ 6_ αν χ Ε ( -οο,Ο) 7"J.Γχ

I

� 'Ε χουμε J,[;1 = (χ 6 ) 7 για κάθε χ Ε R και τότε :

I

• Η ( χ6) 7 είναι σύνθεση των συναρτήσεων I

κ(χ)=χ6 και g(x)= χ 7 και • χ6 Ε (0, + οο ) ( _.!_ Ε R-Z) . Η συνάρτηση

7 παραγωγίζεται στα χ Ε R *

• Η παράσταση v-;. ορίζεται μόνο αν χ :2: Ο για κάθε φυσικό ν.

• αν χ Ε (Ο,+ οο ) έχουμε : 6χ 5 6χ5 6χ 5

7if;J6- 7Vx3s . χ 7ψχs )7 . χ 6χ 5 6χ5 6

7 1 x s ιrx 7xs-rx. 7zΓχ. • αν χ Ε (- οο ,Ο ) έχουμε :

6χ5 6χ 5 6χ5 7ψ-χγ6

-7Ψ-χ 35 ) · (-χ) 7 1 -x s ιrx 6χ5 -6

------;== = 7( -χ5 )"J.Γχ 7"J.Γχ

I I§_ { (-χ)% αν χ<Ο f(x)= χ Ί = 6 χ 7 αν χ :2: 0

κ .λ. π

Να βρεθεί η παράγωγος των συναρτή­σεων f και g με τύπους :

ί. f(χ)=1η(χ2ν) ν Ε Ν* ίί. k(x)=1n jx3 + x j και ίίί. g(x)=log .Jx2 + 2005

Λ'Q)ση ί . f(x)=ln(x2v) ν Ε Ν* Πεδίο ορισμού το R * � Είναι λάθος να γράψουμε :

f(x)=ln(x2v)=2νlnx ( γιατί ; )

Η f είναι σύνθεση των συναρτήσεων : � k : k(x)=lnx� k'(x)=_!_ και χ � g : g(x)=x2v � g ' (x)= 2ν·χ2ν- Ι Τότε : f' (x)=[k(g(x))] '=κ' (g(x)) · ' ( ) 1 2 2ν- Ι 2ν g χ = - · ν·χ =-χ2ν Χ

1 ο .; " τρ �'�Λ:� � ,��;

f(x)=lnx2v= { lnx 2v = 2ν\ηχ· ln(-x) 2v = 2ν\η(-χ)

Τότε :

αν χ>Ο αν χ<Ο { 2ν αν χ>Ο

= { 2

χν αν χ>Ο

f(x)= χ 2ν (-χ)' αν χ<Ο 2ν αν χ<Ο -χ χ

ίί. k(x)=ln l x3 + χ Ι Πεδίο ορισμού το R* � k ' (x)=(ln jx3 + x j ) '= 3�2 + 1 για κάθε χ Ε R* . χ +χ ίίί. g(x)=log .J,_x.,--2 +-20_0_5 Πεδίο ορισμού το R. 1 " Ό τρόπος . � Λογαριθμική συνάρτηση με βάση διάφορη

του e . Αλλαγή βάσης : 1 Ι 2 2005 In.Jx2 + 2005 Τ ' og ν χ + = . οτε : ln l O � (log .Jx2 + 2005 ) '=( ιn.Jx2 + 2005 ) '= ln l O

-1- (ln .Jx2 + 2005 ) ' ln l O � Θα βρούμε την παράγωγο της συνάρτησης

: z(x)=ln .Jx2 + 2005 με τον κανόνα της αλυσίδας. ( Σύνθεση τριών συναρτήσεων )

Θέτουμε: κ=χ2+2005 , u=� τότε ψ=lnu � dψ = dψ . du . dk = _!_ . _1

_. 2χ = dx du dk dx u 2--Γκ.

1 1 1 χ =- ·--· 2χ =- · 2χ � --Γκ_ 2--Γκ. 2κ χ2 + 2005 ' ( ) χ z χ ----,---χ2 + 2005

ΕΥΚΛΕΙΔΗΣ Β ' τ.2/64

Page 67: Ευκλειδης Β 62

Μαθη ματικά για την Γ Λυκείου

> Άρα : (log .Jx2 + 2005 ) '= [ ιn.Jx2 + 2005 ) . ln l O

-1- (ln .Jx2 + 2005 ) '=

χ ln l O ln l O(x 2 + 2005)

> g(x)=log .Jx2 + 2005 =

_!_ 1 =log (x2 + 2005) 2 = - log (x 2 + 2005) .

2

Οπότε βρίσκουμε την παράγωγο (σύνθεση

δύο συναρτήσεων ) .

Να βρεθεί η παράγωγος των συναρτή­

σεων f και g με τύπους :

i. f(x)= 5χ και ii. g(x)=xlogx

i. f(x)= 5χ Πεδίο ορισμού το R. > Μορφή αχ με α σταθερό πραγματικό αριθ­

μό . ( 5x=exln5)

> Η f παραγωγίζεται στο R με : f' (x)=5xln5

ίί. g(x)=x1ogx Πεδίο ορισμού το (Ο,+ οο )

> Μορφή αχ με α=χ όχι σταθερό πραγματικό

αριθμό.

> Μετατροπή σε ισοδύναμη εκθετική με βά­

ση το e .

> Τότε : (xlogx)=(e'ogx· Inγ=e'ogx · Inx . ( logx· lnx) '=

=x1ogx . [(logx) ' lnx+logx(lnx) ' ]=

xlogx , ( �

+ logx

) xln l O χ

[(logαx) '= -1- ]

xlnα

> Θέτουμε : , lnx logx

K(x)=logx· lnx => k (x)= -- + --

n(x)=ex => n '(x)= ex Τότε :

xln l O χ

' ( ) ' (k( ) k '( ) Ιοgχ · Ιηχ ( lnx + logx

) c χ =η χ · χ =e · -- --

xln l O χ

= xlogx . ( � + logx )

xln l O χ

Ν. Ζανταρίδης, Π. Μυταρέλλης, Κ. Παπαδόπουλος Αν η συνάρτηση f: IR�IR είναι περιττή, γνησίως φθίνουσα στο IR και ισχύει f(f(x)) = χ

για κάθε χ ε ΙR να δείξετε ότι f(x) = -χ, χε ΙR.

Έστω ότι υπάρχει χ0 ε ΙR τέτοιος ώστε f(x0) -:;: -χ0, τότε θα είναι f(x0) > -χ0 ή f(x0) < -χ0 . Έστω ότι είναι f(x0) > -χ0 , τότε θα έχουμε :

f(xo) > -χο � f(f(xo)) < f(-xo) (αφού f γνησίως φθίνουσα στο IR) � � χ0 < f(-x0) (ισχύει f(f(xo)) = χ0 , αφού για κάθε χ ε ΙR ισχύει f(f(x)) = χ) � � χ0 < -f(x0) (ισχύει f( -χ0) = -f(x0) αφού η f είναι περιττή) � � f(xo) < -χο ΑΤΟΠΟ, αφού υποθέσαμε ότι f(x0) > -χ0 .

Ομοίως σε άτοπο καταλήγουμε αν υποθέσουμε ότι είναι f(x0) < -χ0. Άρα υποθέτοντας ότι υπάρχει x0 ε iR τέτοιος ώστε f(x0) -:;: -χ0 οδηγηθήκαμε σε άτοπο . Επομένως για κάθε χ ε ΙR είναι f(x) = -χ.

Η συνάρτηση f: IR�IR με f(IR)=IR είναι γνησίως μονότονη στο IR και η Cr διέρχεται από τα σημεία Α(1 , 5) και Β(2, 7).

i) Να βρεθεί το είδος μονοτονίας της f. ίί) Να λυθεί ως προς χεΙR η aνίσωση : Γ1 (3 + Γ1(2χ + 1)) > 1 .

ϊ·. , ) ; • � ii . i) Επειδή η Cr διέρχεται από τα σημεία Α( 1 , 5) και Β(2 , 7) έπεται ότι f(l) = 5 και f(2) = 7 . Παρατηρούμε ότι f(l) < f(2) . Επειδή η f είναι γνησίως μονότονη στο IR και είναι 1 < 2 και f(l) < f(2), έπεται ότι η f είναι γνησίως

αύξουσα στο IR. ίί) Επειδή η f είναι γνησίως μονότονη στο D1· = IR έπεται ότι η f είναι συνάρτηση 1 - 1 .

ΕΥΚΛΕΙΔΗΣ Β ' τ.2/65

Page 68: Ευκλειδης Β 62

Μαθηματικά για την Γ Λυκείου

Άρα η f έχει αντίστροφη συνάρτηση και το πεδίο ορισμού της Γ1 είναι το f(IR) = IR. Έχουμε : f - 1 (3 + Γ1 (2χ + 1 )) > 1 <::::> f (f - 1 (3 + Γ1 (2χ + 1 ))) > f( l ) (αφού f γν. αύξ. στο IR) <::::>

<::::> 3 + Γ1 (2χ + 1 ) > 5 (αφού f(Γ1 (y)) = y και f(l) = 5) <=>

<::::> f - 1 (2x + 1 ) > 2 <::::> f (Γ1 (2χ + 1 )) > f(2) (αφού f γν. αύξ. στο IR) <::::>

<::::> 2χ + 1 > 7 (αφού f(Γ1 (y) = y και f(2) = 7) <::::> χ > 3 .

Α 3 • Αν για τη συνάρτηση f: IR: �IR ισχύει f(x) - f(y);;::In � +x-y για κάθε χ, yε IR: και f(l)=l , Υ

i) να βρείτε τον τύπο της f.

. \ ί>ση : i) 'Ε χουμε f(x) - f(y) ;;:: 1n � + χ - y ( 1 ) .

Υ

ii) να μελετηθεί η f ως προς τη μονοτονία .

Από την ( 1 ) με εναλλαγή των γραμμάτων χ και y έχουμε ότι για κάθε χ, y ε IR: ισχύει:

Υ χ χ f(y) - f(x) ;;:: 1n - + y - χ � f(y) - f(x) ;;:: -1n- - χ + y �-(f(x) - f(y)) � -(ln- + χ - y) � χ Υ Υ

χ � (f(x) - f(y)) ::; 1n- + χ - y Υ Από ( 1 ) και (2) προκύπτει ότι για κάθε χ, y ε IR: ισχύει:

χ f(x) - f(y) = 1n - + χ - y Υ Από την (3) για y = 1 έχουμε ότι για κάθε χ ε IR: είναι :

χ t'( ι ) = ι f(x) - f(l) = 1n - + χ - 1 � f(x) - 1 = 1nx + χ - 1 � f(x) = 1nx + χ

1

ii) Έστω χ ι , χ2 ε (Ο,+οο) με Χ ι < χ2 . ( ιη χ γν. αύξ. ) { 1η χ ι < 1n x 2 ( + )

Έχουμε : Χ ι < χ2 � � 1ηχ ι + Χ ι < 1nx2 + χ2 � f(χ ι ) < f(x2). Χ ι < Χ 2

Άρα η f είναι γν. αύξουσα στο IR: .

(2)

(3)

Α-1-, Για τη συνάρτηση f: IR+�IR ισχύει f(λx)>f( i) για κάθε χ > Ο και λ > 1 . Να δείξετε ότι

η f είναι γνησίως αύξουσα στο IR: . λί>ση : 'Εχουμε f(λχ) > f( � ) , για κάθε χ > Ο και λ > 1 ( 1 ) Έστω χι , χ2 ε IR: με Χ ι < χ2 .

Θα πρέπει να χρησιμοποιήσουμε την ( 1 ) για κα­τάλληλα λ και χ, τέτοια ώστε να είναι: [� = χ , J {χ ' = χ , χ , { χ = Ν,

Λ = ----'- λ - ----'-λ � , Χ , � f χλ = χ , χ , - χ ,

(Λόγω της ( 1 ) , θέτοντας χ = Μ > Ο και

λ = � > 1 ν� (αφού Ο < Χ ι < χ2) , έχουμε ότι ισχύει:

ΕΥΚΛΕΙ Δ ΗΣ Β' τ.2/66

Page 69: Ευκλειδης Β 62

Μαθηματικά για την Γ Λυκείου

Άρα για κάθε χ ι , χ2 ε IR: με χ ι < χ2 ισχύει f(χ ι ) < f(x2) , οπότε η f είναι γνησίως αύξουσα στο IR: . Α� 3 " Για τη συνάρτηση f: IR�IR ισχύει: lim [2f { χ) + f { 2α - χ)] = .e ε IR . Να βρείτε το

χ--+α

lim f { χ) . χ--+α

λΙJ)ση : Θεωρούμε τη συνάρτηση h(x) = 2f(x) + f(2α - χ) ( 1 ) . Από την υπόθεση έχουμε lim h ( χ ) = .e .

χ ---tα Από την ( 1 ) , θέτοντας όπου χ το 2α - χ έχουμε ότι για κάθε χ ε ΙR ισχύει:

2f(2α - χ) + f(x) = h(2α - χ) (2) . {2f ( χ ) + f ( 2α - χ ) = h ( χ ) Οι ( 1 ) και (2) σχηματίζουν το σύστημα: ( ) (Σ) f ( χ ) + 2f ( 2α - χ ) = h 2α - χ

12 1 1 h ( x ) 1 Είναι: D =

1 2 = 3 :;t: O και Dr( x )

= h (2αχ ) 2

= 2h(x) - h(2α - x),

οπότε είναι : f ( x ) = Dr< x > = 2h ( χ ) - h (2α - χ ) = �h ( x ) _ .!_h (2α - χ ) D 3 3 3 (3)

Είναι: y=2α-χ

lim h ( χ ) = .e και lim h ( 2α - χ ) = lim h ( y) = .e χ �α χ ---tα χ �α οπότε λόγω της (3) , έχουμε :

lim f ( χ ) = lim (�h ( χ ) _ .!_ h (2α - χ )) = � lim h ( χ ) _ .!_ lim h (2α - χ ) = � .e - ! e = .!_ ( χ -->α χ --+α 3 3 3 χ -->α 3 χ -->α 3 3 3 Για τη συνάρτηση f: IR�IR ισχύει lim ( f {χ)) ν = Ο, { ν ε ΙΝ* , ν � 2) . Να δειχθεί ότι:

Χ-+ Χο

l im f { χ ) = Ο Χ -+ Χ ο

Για κάθε χε ΙR είναι:

-lf(x) l � f(x) � l f(x) l � -� � f ( x ) � vlr ( χ )Γ � - ν I ( f ( χ ) ) ν l � f ( x ) � ν I ( f (x ) ) " I ( 1 ) ,

Είναι: }��� ν l ( f ( x ) )" l = ν }��� � ( f ( x ))" l = ν ι :��, ( f ( x ) )" l = #ϊ = 0

και: }��� (- ν I( f ( Χ )) " I ) = - }��� ν I( f (Χ ) ) ν I = -0 = Ο , οπότε λόγω της ( 1 ) προκύπτει ότι: lim f ( χ ) = Ο , (κριτήριο παρεμβολής).

Χ -+ Χ ο

l x3 + χ - t l + l x - t l - t Α 7 " Να βρεθεί το όριο lim (αν υπάρχει).

χ -+ \ χ - 1

lx3 + χ - 1 1 + lx - 1 1 - 1 Έστω f ( χ ) = , Dr = IR - { 1 }

χ - 1

'Εχουμε αρχικά απροσδιόριστη μορφή ορίου Q ο

Είναι lim ( χ 3 + χ - 1 ) = 1 > Ο , οπότε είναι χ3 + χ - 1 > Ο κοντά στο χ0 = 1 . χ --+ 1

ΕΥΚΛΕΙΔΗΣ Β ' τ.2/67

Page 70: Ευκλειδης Β 62

Μ αθηματικά για την Γ Λυκείου

Είναι 1im ( χ - 1 ) = Ο (θα χρειαστούν πλευρικά όρια) . χ-->1 χ3 + χ - 1 + χ - 1 - 1 χ3 + 2χ - 3 (χ - 1) (χ2 + χ + 3) Κοντά στο χ0 = 1 με χ > 1 είναι: f(x)= = = = χ2 + χ + 3 , χ - 1 χ - 1 χ - 1

οπότε : 1im f ( x ) = 1iτη (χ 2 + χ + 3 ) = 5 . χ�ι+ χ 4 Ι

Κοντά στο χ0 = 1 με χ < 1 είναι: χ3 + χ - 1 - χ + 1 - 1 χ3 - 1 2 f(x) = =--= χ + χ + 1 χ - 1 χ - 1

οπότε : 1iτη f ( χ ) = 1im ( x2 + χ + 1 ) = 3 . X 4 l Χ 4 1

Επειδή είναι 1 iτη f ( χ ) * 1 im ( χ ) , έπεται ότι δεν υπάρχει το 1 im f ( χ ) . χ--+1 x-t l + χ 4 1

Άρα δεν υπάρχει το ζητούμενο όριο.

ΑΗ . Δίνεται η συνάρτηση f { x) = συν [ π( .Jx 2 + 2χ + 2 - χ)] . Να βρείτε το Δ� f { χ) .

Η 1 ση : 'Ε χουμε Dr = IR. Θεωρούμε τη συνάρτηση g ( χ ) = π ( .J χ 2 + 2χ + 2 - χ ) , Dg = IR. Το }i� g (χ ) οδηγεί

αρχικά σε απροσδιόριστη μορφή (+οο)+(-οο). Για κάθε χ > Ο είναι:

οπότε: π ( 2 + Ο) 1im g ( χ ) = = π .

HHC .Jι + ο + ο + 1 y=g( x ) Άρα: 1im f ( χ ) = 1im συν (g ( χ ) ) = 1imσυνy = συνπ = - 1 .

x -t +oo χ -+ +χ:ι y -+ π

A ':l . Δίνεται η συνάρτηση

f { x) = .J9x + 3 χ+ 1 + 1 - λ · 3 ' .

Ν α βρείτε τα όρια lim f (χ) και lim f (χ) για τις διάφορες τιμές του λ ε IR. χ�+οο χ----+-οο

. \Ίί1ση : Έχουμε Dr = IR. f ( x ) = .J9x + 3 · 3χ + 1 - λ · 3χ

"' 1im f ( χ ) χ -+ -::ο

Είναι: lim 3x = O και lim 9x = O (γιατί 3 > 1 και 9> 1 ) , οπότε : lim f ( x ) = .J0 + 3 · 0 + 1 - λ · 0 = 1 . X -+ -OC Χ -+ ---«> X -+ -OC

rJ 1im f ( χ ) . Η προηγούμενη διαδικασία δεν εφαρμόζεται για κάθε λε ΙR π. χ. για λ2':0 χ -+ +::ο

Γι' αυτό έχουμε: f ( x ) =

.-9χ---;(-1 +_3_�_: -+-

91χ-:-) - λ - 3' = 3' 1 + 3ω +ω - λ - 3' = 3' [ 1 + 3ω +ω - λ)

ΕΥΚΛΕΙΔΗΣ Β ' τ.2/68

Page 71: Ευκλειδης Β 62

Μαθηματικά για την Γ Λυκείου

Είναι: }i,� 3' �� και }�( 1 + 3 (Η + (Η - λ J � .Jι + 3 - 0 + 0 - λ � l - λ.

Διακρίνουμε : 1 ) 1 - λ > Ο <:=> λ < 1

Για λ < 1 είναι

2) 1 - λ < ο <:=> λ > 1 Για λ > 1 είναι

3) 1 - λ = ο <:=> λ = 1 Για λ = 1 είναι:

{ 1 -λ> Ο) lim f ( χ ) = ( +οο) ( 1 - λ) = + οο

X ---t +:::C

{ 1 -λ< Ο) lim f ( χ ) = ( +οο) ( 1 - λ) = - οο

X ---t+:O

3 . 3χ + 1 (9χ + 3 · 3χ + 1 ) - (3χ )2 f (x ) = -.fΨ + 3 · 3χ + 1 - 3χ =�======-'----''--­-./9' + 3 · 3χ + 1 + 3χ --�========�-- =

3χ 1 + 3 - (�)χ + (i )χ + 3χ

( 1 )χ ( Ι ) χ ( Ι Ι ) Είναι: lim - = Ο, Iim - = Ο αφού Ο < - < Ι και Ο < - < Ι Χ -ΗΟΟ 3 Χ - ΗΧ 9 3 9

οπότε: Iim f ( χ ) = 3 + 0 = i . χ-ΗΟΟ .J1 + ο + ο + Ι 2

+οο, αν λ < 1 Άρα: Iim f ( χ ) = -οο, αν λ > 1

x ---t+cc

Α ω. Δίνονται οι μιγαδικοί zι , Ζ2 με l z ι l = lz z l = ι . { lxz ι + Zz l - ι Θεωρούμε τη συνάρτηση f { χ) = χ '

ι, χ = Ο Αν η f είναι συνεχής στο χ0 = Ο, να δειχθεί ότι z1 = z2•

λίJση :

3 - , αν λ = 1 2

Το lim f ( χ ) οδηγεί αρχικά σε απροσδιόριστη μορφή .2. . Γι' αυτό έχουμε: χ�Ο 0 l xz 1 + z2 1 2 - 1 ( xz 1 + z2 ) ( xz1 + 22 ) - 1 x2 1 z 1 1 2 + x (z 1z2 + xz1 z2 ) + l z2 1 2 - Ι x + z 1z2 + z1 z2 f (χ) = = = = --;-----'-'"---;-....C......:... x ( l xz 1 + z2 1 + 1 ) x ( l xz 1 + z2 1 + 1 ) x ( l xz 1 + z2 1 + I ) l xz 1 + z2 1 + I '

Οπότε: ι. f ( ) z 1z2 + z1 z2 z1z2 + z1 z2 ιm χ = = . ΗΟ l z2 1 + 1 2

Επειδή η f είναι συνεχής στο χ0 = Ο έπεται ότι ισχύει:

ΕΥΚΛΕΙΔΗΣ Β ' τ.2/69

Page 72: Ευκλειδης Β 62

i)

Μ αθηματικά για την Γ Λυκείου

2 - (z , ,o) _ I Έχουμε, όμως, l z , l = 1 � lz , l = 1 � z 1 z1 = 1 � z1 =- (2)

z,

ο ' δ ' ' - 1 (3) μοιως απο εικνυεται οτι: z2 = -Zz

Από ( 1 ),(2) και (3) προκύπτει ότι:

Α ί Ι · Για τη συνάρτηση f: IR�IR ισχύει f(x + y) = f(x)f(y) - ex - eY (1) για κάθε χ, yε iR. ί) Αν η f είναι συνεχής στο χ1 = Ο να δειχθεί ότι η f είναι συνεχής στο IR. ίί) Αν η f είναι συνεχής σ' ένα σημείο αε ΙR, να δειχθεί ότι η f είναι συνεχής στο IR.

Επειδή η f είναι συνεχής στο σημείο χ 1 = Ο έπεται ότι ισχύει

Έστω χ0 ε IR τυχαίος. Θα δείξουμε ότι lim f ( χ ) = f ( χ0 ) • Χ�Χο Για το lim f ( χ ) θέτουμε u = χ - χ0, οπότε είναι χ = χ0 + u. Χ---+Χο Όταν χ�χ0 έχουμε u�O (είναι u =ι:. Ο κοντά στο χ0) .

lim f ( χ ) = f (Ο) Χ-->0 (2) .

( I ) ( 2 ) Άρα: l im f( χ) = limf( χ0 + u) = lim{f( x0 ) f ( u) - e'" - eιι ) = f ( x0 ) 1imf( u ) - e'• - limeu = χ�χ11 ιι---tΟ ιι4-Ο u�O ιι�Ο

( I ) = f ( x0 ) f (O) - e'" - e0 = f ( Χ0 + Ο) = f ( χ0 ) . Άρα για κάθε χ0ε ΙR ισχύει lim f ( χ ) = f ( χ0 ) , οπότε η f είναι συνεχής στο IR. Χ�Χο

ίί) Επειδή η f είναι συνεχής στο α έπεται ότι lim f ( χ ) = f (α ) (3 ) . χ-->α Έστω χ0 ε IR τυχαίος. Θα δείξουμε ότι lim f ( χ ) = f ( χ0 ) . Χ-+Χο Για το lim f ( χ ) θέτουμε: u = χ - χ0 + α οπότε είναι χ = χ0 - α + u. Χ-+Χο Όταν χ�χ0 έχουμε u�α. (είναι u =ι:. α κοντά στο χ0) . Άρα:

( I ) lim f ( χ ) = lim f (( x0 - α ) + u ) = lim { f ( χ0 - α ) f (u ) - e'u -α - eιι ) = χ-+χ0 ιι-+α ιι-+α ( 2 ) ( I ) = f ( x0 - α ) lim f (u ) - e'" -α - lim eιι = f ( x0 - α) f (Ο ) - e'u -α - eα = f (( χ0 - α ) + α ) = f ( x0 ) . u-+α u-+α

Άρα για κάθε Χο ε IR ισχύει lim f ( χ ) = f ( χ0 ) , οπότε η f είναι συνεχής σ' όλο το IR. Χ-+Χο

ν ( ν + ι ) Α 1 2 • Η συνάρτηση f: IR�IR είναι συνεχής και είναι: f(l) + f(2) + . . . + f(ν) =

2 .

Να δειχθεί ότι υπάρχει χ0ε [ 1 , ν] τέτοιος ώστε f(x0) = χ0 Λύση : Έστω ότι δεν υπάρχει χ0ε [ 1 , ν] τέτοιος ώστε να είναι f(x0) = χ0, τότε θα ισχύει f(x) =ι:. χ για κάθε

χ ε [ 1 , ν] . Έχουμε : f(x) =ι:. χ, για κάθε χε [l , ν] � f(x) - χ =ι:. Ο, για κάθε χε [ 1 , ν ] � g(x) =ι:. Ο, για κάθε χε [ I , ν ]

όπου g(x) = f(x) - χ, χ ε [ 1 ,ν] . Η συνάρτηση g(x) = f(x) - χ είναι συνεχής στο (διάστημα) [ 1 , ν] ως διαφορά συνεχών συναρτήσεων

και για κάθε χε [ 1 , ν] είναι g(x) =ι:. Ο . Επομένως η g διατηρεί σταθερό πρόσημο στο [ 1 , ν] . Δηλαδή θα είναι g(x) > Ο για κάθε χ ε [ 1 , ν] ή g(x) < Ο για κάθε χε [ 1 , ν] .

ΕΥΚΛΕΙΔΗΣ Β ' τ.2/70

Page 73: Ευκλειδης Β 62

Μαθη ματικά για την Γ Λυκείου

Έστω ότι είναι g(x) > Ο για κάθε χε [ 1 , ν] . Έχουμε : g(x) > Ο, χ ε [ 1 , ν] � f(x) - χ > Ο, χ ε [ 1 , ν ] � f(x) > χ, χ ε [ 1 , ν] ( 1 ) .

f ( 1 ) > 1 f ( 2 ) > 2

Λόγω της ( 1 ) έχουμε : ( + ) v ( v + 1 ) � f(l) + f(2) + . . . + f(v)> 1 +2+ . . . +v� f(l)+f(2)+ . . . + f(v)> .

f ( v) > v v ( v + 1 )

:C\τοπο . αφού δόθηκε ότι είναι: f(l) + f(2) + . . . + f(v) = . 2

2

Όμοίως σε άτοπο καταλήγουμε αν υποθέσουμε ότι ισχύει g(x) < Ο για κάθε χε [ 1 , ν] . Έτσι, υποθέτοντας ότι δεν υπάρχει χ0 ε [ 1 , ν] τέτοιος ώστε να είναι f(x0) = χ0, οδηγηθήκαμε σε άτο­

πο. Επομένως υπάρχει τουλάχιστον ένα χ0 ε [ 1 , ν] τέτοιος ώστε f(x0) = χ0.

Α U · Αν η συναρτηση f: IR�IR είναι συνεχής στο IR και ισχύει f(f(x)) = χ για κάθε χε ΙR, να δει­χθεί ότι υπάρχει χ0ε ΙR τέτοιος ώστε f(x0) = χ0 •

. \ ίJ σ η : . . . f(f(x)) = x ( 1 ).

Έστω ότι δεν υπάρχει χ0ε ΙR τέτοιος ώστε να ισχύει f(x0) = χ0, τότε θα ισχύει f(x) -::1:- χ για κάθε χε ΙR. Έχουμε: f(x) -::1:- χ � f(x) - χ -::1:- Ο � g(x) -:;:. Ο, όπου g(x) = f(x) - χ. Έτσι για την g(x) = f(x) - χ ισχύει g(x) -:;:. Ο για κάθε χε ΙR. Η συνάρτηση g(x) = f(x) - χ είναι συνεχής στο IR, ως διαφορά συνεχών συναρτήσεων.

Επειδή η συνάρτηση g είναι συνεχής στο (διάστημα) IR και για κάθε χε ΙR ισχύει g(x) -:;:. Ο έπεται ότι η g διατηρεί σταθερό πρόσημο στο IR οπότε θα είναι g(x) > Ο για κάθε χ ε ΙR ή g(x) < Ο για κάθε χ ε IR.

Έστω ότι ισχύει g(x) > Ο για κάθε χε ΙR. Τότε θα έχουμε g(x) > Ο � f(x) - χ > Ο � f(x) > χ. Δη­λαδή θα ισχύει f(x) > χ (2) για κάθε χε ΙR.

Από την ( 1 ) θέτοντας όπου χ το f(x) έχουμε ότι για κάθε χε ΙR ισχύει f(f(x))>f(x) · (3) .

Από τις (2) και (3) προκύπτει ότι f(f(x)) > χ για κάθε χ ε ΙR. Άτοπο, αφού δόθηκε ότι είναι f(f(x)) = χ για κάθε χ ε IR.

Ομοίως σε άτοπο καταλήγουμε και στην περί­πτωση να είναι g( χ) < Ο για κάθε χ ε IR.

Έτσι υποθέτοντας ότι δεν υπάρχει χ0 ε ΙR τέ­τοιος ώστε να ισχύει f(xo) = χ0 οδηγηθήκαμε σε άτοπο. Επομένως υπάρχει τουλάχιστον ένα x0 ε iR τέτοιο ώστε f(xo) = χο.

Π ρ οτε ιν{ ψεν ο θέμα : Για τη συνάρτηση f: (α, β) � IR ισχύει:

f(f(x)) = 2f(x) - χ και α < f(x) < β για κάθε χ ε ( α, β), να δειχθεί ότι: f(x) = χ, για κάθε χ ε ( α, β) .

Περ ιοδ ι χ q έκδοση I I '<C t\0!\ IΛ> ΚΛ Ι ΛΙΑΛΟ ' ΟΥ r�ΑΜαθηματικά

Υncύ8υνος 6ιδοσης D. Ε. Βισκαδο�ράκης

• λν ..-e t!Ji ανάγ>η jΙΑ μιιΙ οψjττρη � γόρω anό θ/μιmι ποu _,νν.., ­τοΥ δι::ιcπ.6λοιιwt�μαtwi:v

• Αν θtλ<ιe διlιμαrο της � - 0ΙΝ0δΙλφιw μα8rμαπιο)ν cιτ' � την EMdδa ""' anό την Κιίιφο

.. Αν � διtyμ:πα κca uλ!κό σπ6 1Jαθημαnιοι.ίς �on'ό.lo'IOVι<ό<rμo

.. Aν Θι\m να yfmt � wu �u rιροj!Ιη- γόρω οnό ιιι λ-Λιλιιιυ<ά Προy� Μ�, yύρω ατιό τη � των ΜαSι,μcπν.ών .. ,, .tuςπροmνι\1""< ,.-..;

. Αν θtλ<ιe """"ι'ένος � "" � � '/1(/ΙΙΙ ιιωΜ σος, ΊΟUς μ<i!ιτιtς σας ή ια:ι"ttΙuςφΟ.οuς οας

... Αν θθ.m w JSI'IDψOI'Itllήσειt IIJ3ημcmJΔ nαι.χνlδια '(10. ν11 Φ.οm το μαθημό "'< rno � "' αnαιολιομαι.6

• Av etλ<tew-.. σιω; μαθηιι<: "'< μ.φό; � μοθηl'!l"i< -

• Ανθιλm w � � διάοη""uςmim\μονος ιu:ι.τις�τοuςyiιρφ οτιό τα Μαθηιιmι.ά ιu:ι.τη ΜοΒηumιι\ Εισι.....,

ΟΛΑ ΑΠΑ ΚΑΙ ΑJ\ΛΑ ΠΟΛΛΑ ΠΑ

ΠΟΥ ΚΥΚΛΟΦΟΡΟΥΝ

ΕΥΚΛΕΙΔΗΣ Β ' τ.2/7 1

Ί;Ψ τeύχος 3 ' Νοtμ. 2006 Ιιι#Q(•fit!.:ι�ψι

Avri τφaλ6γου Το 'φ' ώριμοτι!""' της ο�ς Υ\Ο εnιιιαινωνια

Μαθηματικό V\0 όΛη την οοιογbmα 15 Απανιήσο•ς προβλημάποw 2ou πύχοuς Ζ3 ΠραιεLνdμινο nροβλι\μιπα 2J Πως arι:tοοξε ο θεαtπyιος τσ βεώρημ.α του 34 Γιατf οι Πpώτ<Μ Αριθμο� S3 Απειροσn\ π !I Δημιοuργικb; σελίδες Γεωμετρ(ας t8 Απ6δ<•ξη ανισuτwiιν aχtow. με τη βοήθεια ιωpτών σuνορτήαεωv nι Ποιας εtναι ο επ6μενος αριθμός; 84 ΛUση eξισώσeων μι:: ΟΥ\Ο'ότηm; 87 Προβολι\ διανύσμαπις 91 θέμαιu Ανόλuσης (λύ""ς πρσηγοόμ&VΟU m)x.) 15 θ!μιιιu Ανόλuσηc; (nρσmν6μεναj t6 Η αrώδειξη στο ιι:αλεtδοσκότuο 102 Επαγωγuο\ σuμnερααμιπολD'ι!α 1 1 1 Διδαιmκtς ΠΡΙΧ"'fΥiσε•ς {Μαθημιπuαj Ι'ΙΙαγωyή) 1 16 ΔΦλαγος μειαξύ Beπy Mazu, & Petυ Pesic 122 Μικρός ερeυνηnιιές μαθημαn.tς r.ρyαο!ες 133 Μαθηματικό ποιχνt&α 1311 Γιαή δεν tχει λύση (απανιι\Οε� προβ/ιιον 20<ι ιι:uχ )138 Μαθημαιu<Ο! διαγωνwμο{ σι ο Ηρ/Ικλw> ��� Μαθηματικό γu� 1.U Μνήμη ""' σκtψε� -<>Σμ<νόριο μαθημ<ml!ών 148 Ημερ(δα yιa τσν Στtλw Νεyρεπ6ντη 162 Μαθημαιu<Ο! διιrιω'Μμοi 111 θέμαιu Ροuμάνοοον μαθημ<ml!ών δ<αγωνι!J\Ιών 1111 H Eu.V..iδ<uι Γ""fJ8tpla cmι; B� Μαθημαπι<Ι.ς Ολuμιuό&ς 111 Το Βήμα """-'<: Μαθητέ<: μας 183 Κιm\θeση <δ<ών (μtοα από πραyμα111<ό &αγων/111) 191 Αφιtρωμα στον Sι:ιrge l..Bng 232 Σuντα<ΙU<!S και Μαθημαruιό 23\1 θολής + -· (ιαιΑοκαιρ•νές �ρ<ότητες) 2:111 Δώθεση ytQ αχόλια 205 Fle da Meda 3 2006 247 Βψλιοπαροuσiες και βιβλιοποροι.ισιά.σεις 1S4 Δυmιοeιιιλσytς ...

ΑΝΑΖΚ1'ΒΣΠ! ΤΟ ΣτΑ BIDΛioruiiii!IA Κ.. yισ - �ις � yισ αιι σιm � � 't� σra 21o-a2'7118 1\ Η48ΙΙδδ44δ

.....ι�: � .•

Page 74: Ευκλειδης Β 62

17 tΔ 2Υ (J{ [: :� 1: Δόρτσιος Κωνσταντίνος - Ζεμπίλης Κωνσταντίνος

Δίνεται ένα ισοσκελές τρίγωνο ΑΒΓ (ΑΒ=ΑΓ) με πλευρές ΑΒ=ΑΓ=5μ.μ. και ΒΓ= 8μ.μ. Ελαττώνουμε τη βάση σε 7μ.μ. και τα σκέλη του σε 4.95μ.μ.Τότε να συγκριθούν τα εμβαδά των δύο αυτών τριγώνων.

Το ενδιαφέρον στο πρόβλημα 1 αυτό είναι ότι και οι τρεις πλευρές του ελαττώνονται και θα περίμενε κανείς ότι το νέο τρίγωνο που προκύπτει θα έχει μικρότερο εμβαδόν από το αρχικό . Είναι όμως αυτή η προσδοκία μας αληθινή ;

Θα εργαστούμε αρχικά με τα σημερινά λογισμικά και ιδιαίτερα με το The Geometer's Sketchpad, το οποίο μας δίνει τη δυνατότητα να κινούμαστε άνετα μέσα σε ένα περιβάλλον με πολλές δυνατότητες.

Από το μενού «Μετασχηματισμός» και με την εντολή «Μεταφορά» μετακινώ το σημείο Β οριζόντια κατά 8 εκ. ώστε να λάβω το σημείο Γ. Με τον ίδιο τρόπο κατασκευάζω και ένα μήκος ΔΕ ίσο με 5εκ. Κατόπιν από το μενού «Κατασκευές» κατασκευάζω τους δύο κύκλους (Β,ΔΕ) και (Γ,ΔΕ) οι οποίοι τέμνονται σε δύο σημεία από τα οποία το ένα είναι το Α. Ενώνω το Α με το Β και μετά με το Γ και σχηματίζω το τρίγωνο ΑΒΓ. Τους κύκλους αυτούς με το δεύτερο σημείο τομής των τους aποκρύπτω από το μενού «Προβολή».

Στη συνέχεια κατασκευάζω το εσωτερικό του τριγώνου επιλέγοντας το κατάλληλο χρώμα και από το μενού «Μετρήσεις» υπολογίζω τα μήκη των πλευρών του καθώς επίσης την περίμετρο και το εμβαδόν του και πινακοποιώ τα αποτελέσματα όπως φαίνονται στο σχήμα 1 .

Με τον ίδιο ακριβώς τρόπο κατασκευάζω και το δεύτερο τρίγωνο το οποίο φαίνεται στο σχήμα 2 .

Α

/ L-- · - - ·-�-�- ---

Μή κος(τμήμα ΓΒ) 8 ,00 Μήκος(τμήμα ΑΓ) 5 ,00 Μήκος(τμήμα ΑΒ) 5 ,00

Π ερίμετρος(Πολύγωνο ΑΒΓ) 1 8 ,00 Εμβαδόν(Πολύγωνο ΑΒΓ) 1 2 ,00

Μήκος(τμήμα Β Γ ) 7 ,00 Μή κος(τμήμα ΑΓ) 4 ,95 Μή κος(τμή μ α Β Ά' ) 4 ,95

Π ερίμετρος(Πολύγωνο Α'Β Γ ) 1 6 ,90 Εμβαδόν(Πολύγωνο Α'Β Γ ) 1 2 ,25

Σχήμα 1

Σχήμα 2

Το δεύτερο τρίγωνο παρόλο ότι έχει όλες τις πλευρές του μικρότερες από τις αντίστοιχες πλευρές του πρώτου, εντούτοις έχει μεγαλύτερο εμβαδό, όπως αυτό απλά φαίνεται από τους πίνακες των δύο σχημάτων. Είναι δηλαδή :

Ε(ΑΒΓ)=12τ.εκ. και Ε(Α 'Β 'Γ ')=12.25 τ.εκ. ( !) ' Ί " ' , Πάλι μέσα από το ίδιο περιβάλλον του λογισμικού αυτού προσπαθούμε να αντιληφθούμε τη συμπεριφορά αυτών των τριγώνων γενικότερα. Για το λόγο αυτό αξιοποιούμε τη δυνατότητα του λογισμικού με αναφορά των τριγώνων σε ορθογώνιους άξονες.

1 Από άρθρο των Σ.Νεγρεπόντη, Β .Φαρμάκη με τίτλο: «Η "παράλογη" αποτελεσματικότητα των Μαθηματικών στις άλλες επιστήμες» (Διαδίκτυο)

ΕΥΚΛΕΙΔΗΣ Β ' τ.2/72

Page 75: Ευκλειδης Β 62

------------ Το Sketchpad σχεδιάζει μετράει και . . . προκαλεί -----------

/ ./

_./

_ ./· /// �--- - ·-

Σχήμα 3

Παρατηρούμε ότι το δεύτερο τρίγωνο έχει μεγαλύτερο ύψος, και το γεγονός αυτό επηρεάζει και τη συσχέτιση των εμβαδών των τριγώνων αυτών.

Θα αναζητήσουμε την τομή των πλευρών (ΑΒ, Α 'Β ') και (ΑΓ, Α 'Γ) . Δηλαδή τα σημεία Μ και Ν.

Αυτό γίνεται εύκολα από το μενού «κατασκευές» και από την εντολή «Σημείο σε τομή», αφού βέβαια επιλεγούν οι πλευρές αυτές με το βέλος επιλογής. (σχήμα 4) . Στη συνέχεια επιλέγω τα χωρία κατά τα οποία το δεύτερο τρίγωνο πλεονάζει καθώς και εκείνα στα οποία υστερεί τα μετρώ και πινακοποιώ τα αποτελέσματα όπως φαίνεται στο σχήμα 4 .

Εμβαδόν(Πολύγωνο ΜΑ'ΝΑ) Εμβαδόν(Πολύγωνο Β 'ΜΒ) Εμβαδόν(Πολύγωνο ΝΓ 'Γ )

Εμβαδόν(Πολύγωνο Β 'ΜΒ) + Εμβ . . . Σχήμα 4

Από τον πίνακα του ανωτέρω σχήματος φαίνεται ότι το εμβαδόν του μη κυρτού τετραπλεύρου ΜΑ 'ΝΑ είναι ίσο με 1 .00 τ .εκ. άρα μεγαλύτερο από το άθροισμα των εμβαδών των τριγώνων ΜΒΒ ' και ΝΓΓ που είναι ίσο με 0 .75τ .εκ.

' , , ' I " ·' ·-; � : Ll ' .� :· �-'

Μέχρι το σημείο αυτό η διευκόλυνση του λογισμικού ήταν θετική , ως προς την διαπίστωση της σχέσης των εμβαδών, αρχίζει όμως να δημιουργεί και νέα ερωτήματα. Αλλά πριν προχωρήσουμε ας δούμε πώς θα φθάναμε στη σύγκριση των εμβαδών των τριγώνων ΑΒΓ και Α 'Β 'Γ με θεωρητικό τρόπο, κάνοντας χρήση την Ευκλείδεια Γεωμετρία.

Α

5 4 .95

Σχήμα 5

/ Β 4 3 . 5

Από τ ο Πυθαγόρειο θεώρημα μπορούμε ν α υπολογίσουμε τ α ύψη των δύο τριγώνων. Έτσι για το

καθένα έχουμε : ΑΔ = .J ΑΓ2 - ΔΓ2 = .J52 - 42 = .J25 - 1 6 = J9 = 3 άρα ΑΔ=3εκ.

Α ' Δ ' = �(Α ' Γ ')2 - (Δ ' Γ ')2 = .J4.95 2 - 3 . 5 2 = .J24.5025 - 1 2 .25 = .J1 2 .2525 = 3 . 5004 Δηλαδή Α ' Δ '=3 . 5004. Άρα το δεύτερο τρίγωνο έχει μεγαλύτερο ύψος από το πρώτο . Τα εμβαδά αντίστοιχα είναι:

·

1 Ι Ε 1 = - (ΒΓ)(ΑΔ) = - · 8 · 3 = 1 2τ.εκ 2 2

ΕΥΚΛΕΙΔΗΣ Β ' τ.2/73

Page 76: Ευκλειδης Β 62

------------ Το Sketchpad σχεδιάζει μετράει και . . . προκαλεί -----------

: Ας προσπαθήσουμε να βρούμε τα όρια στα οποία μπορούν να κινηθούν οι ίσες πλευρές του αρχικού τριγώνου όταν η βάση του από 8 γίνει 7 μονάδες και απαιτούμε να συμβαίνει το ίδιο, δηλαδή να μεγαλώνει το εμβαδόν του δευτέρου τριγώνου .

Για τούτο ας θεωρήσουμε τα τρίγωνα ΑΒΓ και Α 'Β 'Γ (σχήμα 6) ώστε ΒΓ=8, ΑΒ=ΑΓ=5 και Β 'Γ=7, Α 'Β '=Α 'Γ= λ , με λ::;5

Β

Α Λ"­/ "-

5 / '�5 // ""

/_/ ___ _ _ _ _ __ _ __ � 8 7

il " c Γ::ω�u ::τ �Η �'i' {ις ) Για να έχουν τα δύο τρίγωνα το ίδιο εμβαδόν πρέπει:

(Β 'Γ)·(Α ' Δ ')=24 <=> Α' Δ '= 24 7

Σχήμα 6

άρα η τιμή του λ θα πρέπει να είναι: λ = (2)2 + (24

)2 =�3, 52 +3,42862 =J12, 25 + 1 1, 7553 = �24, 0053 = 4, 8995 2 7

Η τιμή αυτή του λ που βρέθηκε με προσέγγιση του δεκάκις χιλιοστού είναι η κατώτερη τιμή στην οποία μπορεί να μειωθεί η πλευρά ΑΓ του αρχικού τριγώνου ώστε το δεύτερο τρίγωνο να έχει το ίδιο εμβαδόν με το πρώτο. Αυτό γίνεται φανερό αν σκεφθούμε ότι οποιαδήποτε άλλη τιμή μεταξύ της τιμής 4,8995 και του 5 θα δίνει τρίγωνο με μεγαλύτερο εμβαδόν γιατί θα μεγαλώνει το ύψος Α ' Δ ' . Άρα: «Οποιαδήποτε τιμή του λ μεταξύ του 4,8995 και του 5 θα δίνει τρίγωνο με εμβαδόν μεγαλύτερο του εμβαδού του αρχικού τριγώνου».

τ 11 ι 6: r ω :ς ( υλγε �� ρ ι κίJς) Το εμβαδόν του δευτέρου τριγώνου δίνεται από τον τύπο :

Ε(λ) = .l_ · 7 - �λ2 - (7._)2 ( 1 ) 2 2

Μ λ ' ' ' ' I Η7 , ε ετουμε στη συνεχεια τη μονοτονια της συναρτησης: F(λ) = Ε(λ) - 1 2 = - · 7 · λ- - (-) ' - 1 2

2 2

Η σuνάρτηση αυτή έχει παράγωγο F'(λ) � 2 · π. )Ο , γιατί το λ είναι θετικός αριθμός. 2

2 λ2 - (2)2 2

Κατά

συνέπεια η συνάρτηση F είναι γνησίως αύξουσα στο σύνολο των θετικών αριθμών(ειδικότερα εδώ το πεδίο ορισμού της F είναι το σύνολο 0= { 7/2 , + οο } ) .

' ' 1 � � 24 Η F μηδενιζεται οταν : 2 . 7 · \j λ - (2) - 1 2 = Ο <=> \j λ - (2) = Ί <=> · · · <=> λ = 4, 8995

άρα για μεγαλύτερες τιμές του αριθμού αυτού (και μέχρι του 5 γιατί το πρόβλημα απαιτεί ελάττωση όλων των πλευρών του τριγώνου) το Ε(λ) είναι μεγαλύτερο του 1 2 . Τ : Αν ζητήσουμε πως θα μεταβάλλεται το δεύτερο τρίγωνο, αν ελαττώσουμε τη βάση κατά λ και τα σκέλη του κατά μ τότε θα έχουμε για μελέτη συνάρτηση με δύο μεταβλητές και η οποία θα πρέπει να μελετηθεί ανάλογα. Τi:τΗρτη : Ανάλογες ιδέες μπορούν να γίνουν όχι μόνο σε ισοσκελές τρίγωνο αλλά και σε τυχαίο . Δηλαδή μειώνοντας τις α,β,γ, ενός τριγώνου ΑΒΓ κατά τις ποσότητες χ,ψ,z αντίστοιχα, αναζητούμε τον τρόπο μεταβολής του εμβαδού του τριγώνου .

!Κ ω . , " " τΟ.ος . . . . Στο πρόβλημα αυτό βλέπουμε ότι οι περίμετροι των δύο αυτών τριγώνων έχουν αντίστροφη σχέση . Δηλαδή το δεύτερο τρίγωνο παρόλο ότι έχει μεγαλύτερο εμβαδόν από το πρώτο, εν τούτοις έχει μικρότερη περίμετρο από το πρώτο. Στην καθημερινή ζωή είναι άδικο κάποιος να διαθέτει ένα οικόπεδο σαν και το πρώτο τρίγωνο ΑΒΓ, δηλαδή να έχει σε σχέση με το δεύτερο τρίγωνο Α 'Β 'Γ μικρότερο εμβαδά και να χρειάζεται περισσότερα έξοδα για την περίφραξή του . Αυτό δεν μας ξαφνιάζει. Στον κόσμο συμβαίνουν τέτοια παράξενα. Ας σκεφθούμε μεταξύ άλλων και τη νιφάδα του Koch η οποία έχει άπειρο μήκος ως περίμετρο και περικλείει ένα συγκεκριμένο εμβαδόν !

ΕΥΚΛ ΕΙΔΗΣ Β' τ.2/74

Page 77: Ευκλειδης Β 62

Τα Μαθηματικά μας διασκεδάζουν Τα μαθηματικά αν και είναι επιστήμη που απαιτεί αυστηρή διατύπωση, έχουν τη μαγεία να α­

ποσπούν το ενδιαφέρον όλων των ανθρώπων. Επινοήσεις σε προβλήματα ή ασκήσεις με κατάλλη­λο τρόπο διατυπωμένα εξάπτουν το πνεύμα, διεγείρουν τη φαντασία και κεντρίζουν την περιέργεια. Πρώτοι οι Αρχαίοι Έλληνες όπως ο Δ ιόφαντος, ο Ζήνωνας κ. ά. μας δίδαξαν αυτά τα μαθηματικά. Στη στήλη αυτή θα παρουσιάζουμε θέματα τα οποία δεν απαιτούν ιδιαίτερες μαθηματικές γνώσεις αλλά μας διασκεδάζουν με την εκφώνησή τους ή τη λύση τους και είναι μια ευχάριστη και συναρ­παστική ασχο/,ία .

Επί χιλιάδες χρόνια Βασιλείς και στρατη­γοί αλλά και σήμερα οι πρωθυπουργοί, ήθελαν και θέλουν να έχουν ασφαλή και αποτελεσμα­τική επικοινωνία για ασφαλή διακυβέρνηση των χωρών τους. Το ίδιο οι εταιρίες, οι τράπε­ζες ακόμη και οι πολίτες σε απολυταρχικά κα­θεστώτα έχουν την ανάγκη να επικοινωνήσουν μυστικά. Γι' αυτό επινοούν όπως λέμε κώδι­κες επικοινωνίας. Όμως οι ανταγωνιστές ή οι κωδικοθραύστες καταφέρνουν πολλές φορές να τους σπάσουν όπως ακριβώς έγινε και με την γραφή Β, για την οποία με το πέρασμα των αιώνων είχε χαθεί η γλώσσα, αλλά και δι­αφόρων αρχαίων γραφών όπως των αιγυπτια­κών ιερογλυφικών κ.ά . .

Τ α μαθηματικά είχαν και έχουν τον πρώτο λόγο στην κρυπτογραφία αλλά και

στο σπάσιμο των γρίφων. (γι' αυτό σας προ­τείνουμε να διαβάστε το βιβλίο Κώδικες και Μυστικά του Sίmon Sίngh) .

Σήμερα όλοι μας επικοινωνούμε με τα κι­νητά τηλέφωνα, τα fax, τα email, τα sms, τα mms κ.ά. είναι ασφαλής αυτή η επικοινωνία μας; Φυσικά όχι. Και το ερώτημα είναι θα επι­λέξει τελικά η κοινωνία μας τον αστυνομικό έλεγχο (το μεγάλο αδελφό) ή την προστασία του ιδιωτικού απορρήτου;

Υπάρχουν κλειδιά για την προστασία του ιδιωτικού μας απορρήτου; Ναι υπάρχουν οι πρώτοι αριθμοί με μεγάλο αριθμό ψηφίων.

(Σας προτείνω να διαβάσετε το βιβλίο Η ΜΟ ΥΣΙΚΗ ΤΩΝ ΠΡΩ ΤΩΝ ΑΡΙΘΜΩΝ του Marcus Du Sautoy διότι ακόμη και σήμερα οι πρώτοι αριθμοί αποτελούν μυστήριο που προ­σπαθεί να εξιχνιάσει η ανθρώπινη γνώση) .

ι ί : . .

Επιμέλεια : Παναγιώτης Χριστόπουλος

«Πιο είναι το όνομα του πουλιού που απο­τελείται από 5 γράμματα και ότι, αν αποδοθεί στο καθένα αριθμητική τιμή ίση με τη σειρά που κατέχει στο αλφάβητο (1 έως 24) στις τι­μές θα έχουμε :

i) Το άθροισμα στα δύο πρώτα είναι 1 4 ii) Το άθροισμα στο 3 ° και 4° είναι 2 1 iii) Η διαφορά του αθροίσματος των 3 τε­

λευταίων από το άθροισμα των 2 πρώτων είναι 1 9

iν) Το γινόμενο του δεύτερου γράμματος επί τη διαφορά του από το 1 7 είναι 33/8 του γινομένου του τρίτου επί τη διαφορά του από το 1 7» .

:i ,UTO Κάποτε μια διαφήμιση έλεγε : «μόνο με 10

Ευρώ μπορείτε να αποκτήσετε ποδήλατο των 50 Ευρώ» .

Στο τηλέφωνο εξηγούσαν : με τα 1 0 Ευρώ θα πάρετε 4 κουπόνια θα τα πουλήσετε σε 4 φί­λους και έτσι με τα 40 Ευρώ και τα 1 0 που δώ­σατε για τα κουπόνια θα πάρετε το ποδήλατο.

Αλλά και ο κάθε φ ίλος μπορεί να ανταλλά­ξει το κουπόνι που αγόρασε με 5 άλλα τα οποία θα πουλήσει για να πάρει και αυτός ποδήλατο. Κ. ο. κ. Υπάρχει κάποια απάτη ;

, , ; π ω δ ω i: Ρώτησαν οι μαθητές τον καθηγητή τους

πόσο χρόνων είναι ο γιος του και τους απάντη­σε: <<Είχα 4 φορές την ηλικία του γιου μου ό­

ταν είχα την ηλικία που έχει αυτός σήμερα. Όταν φτάσει στην ηλικία που έχω εγώ σήμερα και οι δυο μας θα έχουμε άθροισμα 119 έτη.»

-ι -"· τ : � - .' ι

Μια παρέα από Πιθηκάκια θορυβούσε. Από τη θορυβώδη παρέα το ένα όγδο στο τε-

ΕΥΚΛΕΙΔΗΣ Β" τ.2/75

Page 78: Ευκλειδης Β 62

Τα Μαθηματικά μας Διασκεδάζουν

τράγωνο χοροπηδούσε μέσα στο δάσος, ενώ οι άλλοι δώδεκα φώναζαν στην κορυφή του λό­φου με τις καρύδες. Πόσοι ήταν οι Πίθηκοι;

Κάποιος αγόρασε με 30 Ευρώ 30 πουλιά (Καναρίνια, Παπαγάλους, Περιστέρια) .

Τρία Καναρίνια αξίζουν 2 Ευρώ. Επίσης 2 Παπαγάλοι και 1 Περιστέρι αξίζουν 2 Ευρώ. Πόσα πουλιά πήρε από κάθε είδος;

Έχω τρείς δίσκους τον Α, τον Β και τον Γ. Στον Α δίσκο έχω κέρματα 2€, 1€, 0.50€,

0.20€, 0.10€ το ένα πάνω στο άλλο από το με­γαλύτερο στο μικρότερο. Μπορείτε να τα με­ταφέρετε στο Γ δίσκο με την ίδια σειρά;

Σε κάθε κίνηση μεταφέρουμε μόνο ένα νόμισμα. Δεν επιτρέπεται να τοποθετούμε μεγαλύ­τερο νόμισμα πάνω σε μικρότερο. Μόνο προσωρινή χρήση γίνεται του δί­σκου Β .

_ , i · �: η .,( / ιΟΙ ι . .({ (& � (U�.:; μ � .� , ς Συμπληρώστε με ψηφία τους αστερίσκους

στον πολλαπλασιασμό : * 1 * 3 * 2

* 3 * 3 * 2 *

* 2 * 5

1 * 8 * 3 0

Στα Φιλιτρά της Μεσσηνίας υπάρχει ένα ομοίωμα του πύργου του Αϊφελ με ύψος 1 5 μέτρα. Ο πύργος του Αϊφελ στο Παρίσι έχει ύψος 300 μέτρα και το ατσάλι από το οποίο είναι κατασκευασμένος έχει βάρος 8 .000 τό­νους . Πόσους τόνους ζυγίζει το ομοίωμα στα Φιλιατρά;

Ο Οκτώβρης έχει 3 Ι μέρες. Αρα ο πατέρας του Πέτρου θα δώσει στο γιο του 3Ι Χ Ι ΟΟ = 3 100 Ευ­ρώ. Ο γιος πρέπει να του επιστρέψει :

2 3 1 - 1 Ι +2+2 2 +23

• • • +20 30 = Ι =2 3 1 - 1 2 - 1

λεπτά. (γεωμετρική πρόοδος) . Δηλαδή 2Ι . 474. 63Ι , 67 Ευρώ!!! Φυσικά δεν πρέπει να δε-

χτεί ο Πέτρος την πρόταση του Πατέρα του. r� , �f. ;·�ρτ(r, �Ι ! � � ·-:� α

500+200+ΙΟΟ+50+20+ΙΟ+5+2+Ι +� 50+�2 Ο+Ο, Ι 0+0, 05+0, 02+0, 0Ι = 888, 88

Αν πάρουμε ένα από κάθε νόμισμα θα έχουμε 888, 88 αν πάρουμε ακόμη Ι 00+ Ι Ο+ Ι +0, Ι 0+0, ΟΙ +0, ΟΙ θα έχουμε Ι 000 Ευ­ρώ. Τεμάχια 2 Ι .

Αν κυκλοφορούσε σε Ι , 3 , 5 και παίρναμε ένα από κάθε νόμισμα θα είχαμε 999, 99 Ευρώ και ένα ακόμη Ο, ΟΙ δίνουν 1 000 Ευρώ. Τεμάχια Ι 6.

Είναι ο 64 = 82 = 43 Τριψήφιος είναι ο 729 = 2 72 = 93

Από το Ι" κελί θα περάσουν όλοι οι φύλακες δηλαδή ζυγός αριθμός άρα θα παραμείνει κλειστό. Από τα 2" και 3° θα περάσει ο φύλακας με αριθμό ένα και ο φύλακας με αριθμό δύο και τρία αντίστοι­χα άρα θα παραμείνουν κλειστά. Από το 4" θα πε­ράσουν τρεις άρα θα παραμείνει ανοικτό. Κ. ο. κ. δη­λαδή θα μείνουν ανοικτά τα κελιά των τετράγωνων αριθμών.

Το i θα είναι το2025. Δηλαδή το χ=45. Επο­μένως 2025 - 2006 = Ι9 χρόνια άρα σήμερα είναι 45 - Ι9=26 χρόνων.

Έστω R η ακτίνα της Γης (σε μέτρα) στον Ιση­μερινό. Επομένως το καλώδιο έχει μήκος Κ = 2π(R+5) = 2πR+ΙΟπ

Μέσα στη Γη σε βάθος 5 μέτρα θα έχει μήκος Κ ' = 2π(R-5) = 2πR-Ι Οπ. Δηλαδή θα εξοικονομήσει Κ-Κ ' = 20π = 62, 8 μέτρα περίπου και φυσικά είναι ανεξάρτητο από την ακτίνα άρα το ίδιο ισχύει σε κάθε σφαιρικό σώμα.

Δ ιότι 7ΧΙ IXJ3 = Ι ΟΟΙ . Κάθε αριθμός που πολλαπλασιάζεται με Ι ΟΟΙ επαναλαμβάνει τα ψηφία του.

Διόρθωση Πρώτα πρέπει να πολλαπλασιάσει το νούμερο επί δυο και μετά να προσθέσει 5 μονά­δες . .

Είναι ο παππούς ο πατέρας και η κόρη. Ο πατέ­ρας του Γιώργου είναι παππούς της Ελένης.

'_ J ψ'Η'rτyΗ}:; Το ποσό ήταν 4000 Ευρώ.

Έστω p 1 = 7 και p 2 = 1 1 τότε το Α = 5305 7

που αντιστοιχεί στο Ι έτος έως το Α = 53Ι57 που αντιστοιχεί στα Ι 00 έτη.

ΕΥΚΛΕΙΔΗΣ Β ' τ.2!76

Page 79: Ευκλειδης Β 62

Ο Ευκλεiδης

((Η καρδιά των μαθηματικών είναι τα προβλήματα και οι λύσεις και

ο κύριος λόγος ύπαρξης του μαθηματικού είναι να λύνει προβλήματω>.

P. R. HALMOS

Επιμέλεια: Α. Κυριακόπουλος, Γ. Στρατής, Γ. Τριάντος, Ν. Αντωνόπουλος

8 Ι . Δίνεται το εμβαδόν Ε ενός τριγώνου καθώς και το μέγεθος φ μιας εκ των γωνιών του. Να βρείτε τα μήκη των πλευρών α και β, ώστε το μήκος της πλευράς γ που βρίσκεται απέναντι από τη γωνία φ να είναι το ελάχιστο δυνατό. (επροτάθη από τον Ακαδημαϊκό Ν. Αρτεμιάδη) Λύση (από τον συνάδελφο Σ. Σκοτίδα)

Από το νόμο των συνημιτόνων έχουμε:

γ2=α2+β2-2αβσυνφ:Ξ:2αβ-2αβσυνφ = 2αβ( 1 -συνφ) 2 2 φ ΔΕ ημ -= - ( 1 - συνφ ) = ΔΕ 2 = 4Εσφ!.

ημφ 2ημ .τ_συν.τ_ 2 2 2 Άρα η ελάχιστη τιμή της πλευράς γ είναι

2�Εσφ φ και επιτυγχάνεται όταν α = β = J 2Ε 2 η�

Λύσεις έστειλαν επίσης οι συνάδελφοι Καλάκος Αθα­

νάσιος- Κάτω Π ατήσια, Η λιόπουλος Γι{ιννης - Καλα­

μiπα, Ι ωαννίδης Αντιόνης - Λάρισα, Σταματογιάννης

Γι fιννης - Δροσιά Αττικής, Τσαπακίδης Γιώργος -

Αγρίνιο, Π απαϊωάννου Εμμανουέλα- Ρέθυμνο, οι κύ­

ριοι Ροδόλφος Μπόρης - Δάφνη Αττικής, Λνδρης Ι ­

ωάννης - μηχανικός Αθήνα και ο μαθητής Σακελλά­

ρης Γιώργος - Μουσικό Λύκειο Βόλου.

82. Να αποδείξετε ότι το πολυώνυμο Ρ(χ)=2χ8+3χ6-5χ3-4χ+6 δεν έχει πραγματικές ρίζες. (Επροτάθη από τον καθηγητή Θεόδωρο Μπόλη) Λύση (Από τον κ. Ρ. Μπόρη - Δάφνη)

Αν f(x)=2x8-4x και g(x)=3x6-5x3+6, τότε έχουμε P(x)=f(x)+g(x) . Η συνάρτηση g είναι τριώνυμο ως προς χ3, οπότε η ελάχιστη τιμή της είναι

4 7 1 2

Για τη συνάρτηση f(x) 2χ8-4χ είναι f' (x)= 1 6x7-4=4(4x7- 1 ) οπότε εύκολα βρίσκουμε

' ' ζ λ ' λ ' 1 οτι παρουσια ει ο ικο ε αχιστο στο Χ0 = 7 r; .

�4 Το ολικό ελάχιστο της f είναι

f min = f ( Χ0 ) = 2Χ0 ( χ: - 2 ) = 2χο ( � - 2) = -� Χ ο

Άρα για κάθε χ ε R έχουμε 47 7 47 - 42χ ' 47 Ρ ( χ ) ;?:---Χ0 = 0

> 0, διοτι Χ0 < 1 < -1 2 2 1 2 42

οπότε το πολυώνυμο δεν έχει πραγματικές ρίζες. Λύσεις έστειλαν επίσης οι συνάδελφοι Τσαπακίδης

Γιώργος - Αγρίνιο, Καλάκος Αθανάσιος- Κάτω Π α­

τήσια, Η λιόπουλος Γιάννης - Καλαμ{Ηα, Ι ωαννίδης

Αντώνης - Λάρ ισα, Μ άγκος Αθαν{ισιος - Κοζάνη

και ο μαθητής Σακι:λλάρης Γιώργος - \'Ι ουσικό Λ1J­

κειο ΒίJλου.

83. Θεωρούμε ένα τρίγωνο ΑΒΓ και στις προε­κτάσεις των πλευρών ΒΓ (προς το Γ), Γ Α (προς το Α) και ΑΒ (προς το Β) παίρνουμε αντιστοί­χως τα σημεία Δ, Ε και Ζ έτσι, ώ­στε:Γ Δ=ΑΕ=ΒΖ.

ΜιΑ Α(ιDΠιδ"ηι Π pDTA6"tt yιa TD δ"ΧΙΙΑιΙCD lιιίιlnμα

ΕΚΑΟΤ ΚΟ Γ. Βλάχος Λ.Τ. 1 4,50€

Σόλωνος 1 00 • 1 06 80 Αθιiνα Τηλ.: 2 1 0 3646 1 25 • Fax: 2 1 0 363 1 363

WEB:http://www.eosk_gr • E-mail: [email protected]

ΕΥΚΛΕΙΔΗΣ Β' τ.Ι /77

Page 80: Ευκλειδης Β 62

Ο Ευκλείδης προτείνει . . . Ευκλείδη . • . και Διόφαντο

Να αποδείξετε ότι, αν το τρίγωνο ΔΕΖ είναι ισό­πλευρο, τότε και το τρίγωνο ΑΒΓ είναι ισόπλευρο. (Επροτάθη από τονΑντώνη Κυριακόπουλο) Λί>ση (από τον Γιώργο Αποστολόπουλο - '\1εσολόγ­γι) Θεωρούμε ότι το τρίγωνο δεν είναι ισόπλευρο. Χωρίς βλάβη της γενικότητας υποθέτουμε ότι α::Ξβ::Ξγ με τη μια τουλάχιστον από τις δυο ανισότη­τες (έστω β<γ) να είναι γνήσια.

Τότε έχουμε : α � β � Α. � :Β � 1 80° - ω � 1 80° - φ � ω � φ και α<γ�ΒΓ<ΑΒ�ΒΓ +Γ Δ<ΑΒ+ΒΖ, οπότε ΒΔ<ΑΖ

Ε Από την τε-

z οπότε θα είναι ΕΡ�ΖΔ ( 1 )

λευταία ανισότη­τα προκύπτει ότι υπάρχει σημείο Ρ στο τμήμα ΑΖ ώστε να ισχύει ΑΡ=ΒΔ

Τα τρίγωνα Δ ΑΕΡ και ΒΔΖ

έχουν: ΑΕ=ΒΖ, ΑΡ=ΒΔ και ω�φ,

Η γωνία ω είναι αμβλεία και τα σημεία Ρ, Ζ είναι στο ίδιο Ζ ημιεπίπεδο ως προς την ευθεία από το Ε την κάθετη στην ΑΖ με το σημείο Ζ να απέχει από το ίχνος της Ε στην ΑΖ, από­σταση μεγαλύ­τερη από ότι το Δ

Ρ δηλ. ΑΖ>ΑΡ, οπότε ΕΖ>ΕΡ και δεδομένου ισχύ­ει η ( 1 ) έχουμε ΕΖ>ΖΔ που είναι άτοπο, διότι το τρίγωνο ΔΕΖ είναι ισόπλευρο. Άρα το τρίγωνο ΑΒΓ είναι ισόπλευρο . Β 'τρ όπος Γ. Τασσόπουλος

Τα τρίγωνα ΒΔΕ, ΓΕΖ, ΑΔΖ έχουν δύο πλευρές ίσες. Αν λοιπόν α > β � γ τότε Δ ι > Ε ι � Ζι οπότε: � � Ε2 >�. Άρα Ει + Ε2 > Ζι +Δ2 . Άρα: ω' >Α ( 1 )

� � � � � � � Α + Β + Γ � Αλλά Α > Β � Γ � Α > � Α > 60° (2) 3

Οι ( 1 ), (2) οδηγούν σε άτοπο. Δ ι α φ ο ρ ετ ι κά :

� � � ( ι ) Α > Β � Γ� 60° > Α > Β � Γ� 1 80° > Α + Β + Γ

άτοπο. Όμοια καταλήγουμε σε άτοπο αν α � β > γ . Άρα α=β=γ. Λύσεις έστειλαν επίσης οι συνάδελφοι Καλάκος Α­θανάσιος- Κάτω Πατήσια, Χυτήρης - Κέρκυρα, Η-

λίας Κώστας -Αλιβέρι, Ιωαννίδης Αντώνης - Λάρι­σα, Τσαπακίδης Γιώργος - Αγρίνιο, Σαμπάς Θεόδω­ρος - Πάτρα, Μάγκος Αθανάσιος - Κοζάνη και ο κύριος Ροδόλφος Μπόρης - Δάφνη Αττικής. 84. Να βρείτε την ελάχιστη τιμή της παράστα-

σης Α = (--\- + --\- + --\-) · (α + ι) {β + ι) ( γ + ι) για α β γ

τις διάφορες θετικές τιμές των αριθμών α, β, γ. (Επροτάθη από το συνάδελφο Γιώργο Αποστολόπου­λο - Μεσολόγγι) Λί>ση ( από το συνάδελφο Καλάκο Αθανάσιο- Κάτω Π ατήσια) Από την ανισότητα του αριθμητικού - γεωμετρι­κού μέσου έχουμε :

r-'-----1 1 1 Α � 3 · 3 � -β2 ·"7 (α + 1 ) (β + 1 ) (γ + 1 ) με την ισότητα να ισχύει μόνον όταν α=β=γ , (α + 1 )3 (β + 1 )3 (γ + 1 )3 Άρα Α � 3 3 . .....:...._---,--:-α2 β2 γ2

( χ + 1 )3 Θεωρούμε τη συνάρτηση f(x) = 1 χ > Ο . χ-( χ + 1 )2 Ισχύει: f ' ( χ ) = 3 ( χ - 2 ) η f παρουσιάζει ο-χ

λικό ελάχιστο για χ=2, ίσο με f ( 2 ) = 27 . 4

Άρα για κάθε χ>Ο ισχύει f ( χ ) � 27 . 'Ετσι έχουμε: 4

Α � 3ψ (α) f(β) f (γ) � 3�(�)' = � οπότε η ελάχιστη τιμή της παράστασης Α είναι: Α 8 1 ' ' β 2 min = 4 και επιτυγχανεται οταν α= =γ= Λύσεις έστειλαν επίσης οι συνάδελφοι Σωτήρης Σκο­τίδας - Καρδίτσα, Ηλίας Κώστας - Αλιβέρι, Ιωαννί­δης Αντώνης - Λάρισα, Τσαπακίδης Γιώργος - Αγρί­νιο, Μάγκος Αθανάσιος - Κοζάνη ο κύριος Άνδρης Ιωάννης - μηχανικος Αθήνα και ο κύριος Ροδόλφος Μπόρης - Δάφνη Αττικής.

85. Δίνεται η συνάρτηση f: [O, +oo)->R για την οποία ισχύει f(x)er<x>=x για κάθε χ�Ο. Να αποδεί­ξετε ότι:

ι . Για κάθε χ�Ο, ισχύει f(x)�O 2. Η συνάρτηση f είναι γνησίως αύξουσα

στο [Ο,+οο) 3. Η συνάρτηση f αντιστρέφεται και να

βρείτε την f1 4. Η συνάρτηση f είναι συνεχής στο [Ο,+οο) 5. lim f ( χ) = +οο

χ--++οο

6. Η συνάρτηση f είναι παραγωγίσιμη στο [Ο,+οο)

ΕΥΚΛΕΙΔΗΣ Β' τ.2178

Page 81: Ευκλειδης Β 62

Ο Ευκλείδης προτείνει .. . Ευκλείδη . . . και Διόφαντο

( Επροτ{ιΟη ιι.π6 τον l:ιιμπι'ι. Θι :6όωρο - l l ιίτfJU) .

Λύση (από το συνάδελφο Γ.Τσαπακίδη - Αγρίνιο) Είναι: f(x)e'{x >=x�O, για κάθε χ�Ο, οπότε f(x)�O . Αν η f δεν ήταν γν. αύξουσα στο [Ο,+οο) , τότε θα υπήρχαν χ 1 , χ2 ε [Ο,+οο) με χ ι < χ2 ώστε f (χ l ) �f(x2) . Τότε όμως θα είχαμε και e ' ( x , ) 2': et ( x , J

, οπότε f ( χ 1 ) e '·

( x,) 2': f ( χ 2 ) e '· ( x , ) <=> χ 1 2': χ 2 , άτοπο.

Άρα η f είναι γνησίως αύξουσα στο [Ο,+οο) . Η f ως γνησίως αύξουσα είναι « 1 - 1 » οπότε αντι­στρέφεται. Αν y=f(x), τότε έχουμε yeY=x, y�O, ο­πότε f 1 (x)=exx, χ�Ο . Η συνάρτηση f 1 είναι συνεχής στο διάστημα [Ο,+οο) , οπότε και η aντίστροφή της, δηλαδή η f είναι συνεχής . Έστω ότι υπάρχει Μ>Ο με f(χ)::ΞΜ, τότε erι x > :::;eM και f( x )e 1i x l :::;Me\ 1<::::> x:::;Me\·t για κάθε χ�Ο, που αποκλείεται . Άρα η f δεν είναι φραγμένη και είναι γνη σίω; αύ�ουσα. οπότε 1 ί ιη f ( χ ) = +:χ> .

Χ � + Χ

Η συ\ 'άρτη ση f 1 ( x )= xe' είναι παραγωγίσιμη στο [Ο .�οο) με ( f 1 ( x ) ) '= e' +xe'=( x+ l )e':;tO, οπότε η αντιστροφ1i τη;. που είναι η f είναι παραγωγίσιμη στο [0 .-οο ) . Λύσεις έστειλαν επίσης ο ι συνάδελφοι Καλάκος Αθα­νάσιος- Κάτω Πατήσια, Κούρτης Χρυσόστομος - Λά­ρισα, Ηί.ιόπουλος Γιάννης - Καλαμάτα, Τσόπελας Ιωάννης- Αμαλιάδα, Ιωαννίδης Αντώνης - Λάρισα, Μάγκος Αθανάσιος - Κοζάνη και ο κύριος Ροδόλφος Μπόρης - .λάφνη Αττικής. 86. Αν για τους πραγματικούς αριθμούς χ, y, z

ο 1 1 1 ισχύουν οι σχεσεις -- + -- + -- = 1 και

x + l y + l z + l χ Υ z

------=-] + ] + ] = Ο να αποδείξετε ( x + l ) ' (y + l )' (z + l ) .

ότι: χ3+ y3+ z3=6 i Ε π ιιuτιίθη από το μηzυn κ{� ί\ \'Ι) μη Β �·J(inη - . υ ιΙιν ιι. ) Λύση (από τον Γ. Σταματογιάννη - Δροσιά Αττικής)

1 1 1 Αν θέσουμε -- = α, -- = β και -- = γ τότε

x + l y + l z + l οι δοσμένες ισότητες γίνονται: α+β+γ= Ι και α2+β2+γ2= α3+β3+/, διότι

χ χ ( 1 ) 2 χ + 1 _ 1 ( 1 ) 2

( χ + Ι ) 3 = x + l x + l

= x + l x + l

= ( Ι - α )2 = α2 - α3 . Η γνωστή ταυτότητα α3+β3+γ3=(α+β+γ) (α2+β2+γ

2-αβ-βγ-γα)+3αβγ, εξαιτίας των παραπάνω ισοτήτων γράφεται: αβ+βγ+γα=3αβγ

<=> ± + � + � = 3 <=> (± - r) + (� - I ) + [� - ι ) = ο οπότε ( ± - 1 )' + (i - 1 )' + ( � - !)' 3 ( ± - r} (� - �} ( � -ι) = 3 ·

α�

γ ( 1 - α) ( Ι - β) ( Ι - γ)

3 = - ( ! + 3αβγ - 1 - αβγ) = 6 αβγ

διότι α+β+γ= Ι και αβ+βγ+γα=3αβγ. Αλλά: Ι Ι I 1

-- = α <::::> - - 1 = χ , - - 1 = y, - - 1 = z, x + l α β γ

οπότε χ3+ y3+ z3=6 Λύσεις έστειλαν επίσης οι συνάδελφοι Καλάκος Α­θανάσιος- Κάτω Πατήσια, Ιωαννίδης Αντώνης -Λάρισα, Τσαπακίδης Γιώργος - Αγρίνιο, Σαμπάς Θεόδωρος - Πάτρα, Μάγκος Αθανάσιος - Κοζάνη, Τσόπελας Ιωάννης- Αμαλιάδα, Αποστολόπουλος Γεώργιος - Μεσολόγγι, ο μαθητής Σακελλάρης Γιώργος - Μουσικό Λύκειο Βόλου και Κούρτης Χρυσόστομος - Λάρισα. 87. Δίνεται η εξίσωση 3(x2+y2)+2(xy-33 1 )=2. Να βρείτε τις ακέραιες λύσεις της (Επροτάθη από το συνάδελφο Τριάντο Γιώργο -Αθήνα) . \ 1>ση ( υπι'. το συνάδελφο \ Ι ι'ι.γκο Λ Η ιιν6σω - κο;(ι.­νη ) Η εξίσωση γράφεται: 3x2+2yx+3y2-664=0. Για να έχει ακέραια λύση , πρέπει η διακρίνουσα του ως προς χ τριωνύμου να είναι τετράγωνο θετι­κού ακεραίου . Είναι: Δ= 1 6 ·2(249- /) και για να είναι τέλειο τε­τράγωνο, αρκεί να υπάρχει θετικός ακέραιος κ ώ­στε να ισχύει 2(249- y2)=κ2

f1��HMAfiKA Αποκτήστε το Β' τεύχος του νέου βιβλίου των --�":":::) Τάκη Δρούτσο - Νίκου Πονουσόκη Νέα έκδοση

� ι ··� Qf '"''f '"'

θετικής κο ι Τεχνολογικής κατεύθυνσης με έκπτωση 1 Ο% ΚΟΙ πόρτε μοζί ΚΟΙ ΤΟ Α' τεύχος 13�ii#Jt)f!1!1�fιH8 Η προσφορά ισχύει μέχρι εξαντλήσεως της Α' έκδοσης.

ΕΚΔΟΣΕΙΣ ΚΟΚΟΠΑΚΗ

ΑθΗΝΑ: Μεσολογγίου 1 , τηλ. : 2 10 3804347 θΕΣΣΑΛΟΝΙΚΗ: Πλάτωνος 33, τηλ . : 231 0 230 31 0

Page 82: Ευκλειδης Β 62

Ο Ευκλείδης προτείνει .•• Ευκλείδη . . . και Διόφαντο

Από την τελευταία ισότητα προκύπτει ότι κ άρτιος, στα σημεία Ε, Ζ και Η, Θ αντίστοιχα να αποδείξε-οπότε υπάρχει λεΝ* ώστε κ=2λ και η ισότητα γί- τε ότι α) ΕΗ//ΔΓ β) 1 3 (ΕΖΗ)=20(ΕΘΗ) νεται 2λ2=249- y2<:=> y2=249-2λ2 (Προτείνεται από το συνάδελφο Γιώργο Αποστολό-

οπότε λ2< 1 24,5 δηλ. Ι λ Ι ε { 1 ,2 , . . . , 1 1 } πουλο - Μεσολόγγι)

Εύκολα πλέον διαπιστώνουμε ότι η ισότητα y2=249-2λ2 ικανοποιείται μόνο όταν I λ I =8 ή 1 0, οπότε y ε {- 1 1 ,-7,7, 1 1 } Με αντικατάσταση στην αρχική εξίσωση βρί­σκουμε τις αντίστοιχες τιμές του χ και τελικά οι ακέραιες λύσεις της εξίσωσης είναι οι:

(7 , 1 1 ) , (-7,- 1 1 ) , ( 1 1 ,7), (- 1 1 ,-7) Λύσεις έστειλαν Καλάκος Αθανάσιος- Κάτω Πατήσια, Ιωαννίδης Αντώνης - Λάρισα, Τσαπακίδης Γιώργος ­Αγρίνιο, Ηλιόπουλος Ιωάννης - Καλαμάτα, Κούρτης Χρυσόστομος - Λάρισα, Αποστολόπουλος Γεώργιος -Μεσολόγγι, Χυτήρης - Κέρκυρα, Ροδόλφος Μπόρης -Δάφνη Αττικής, Άνδρης Ιωάννης - Αθήνα. 88. Έστω οι ν το πλήθος θετικοί ακέραιοι

Κι, Κ2, • • • , Κν• Να αποδείξετε ότι υπάρχουν ί και j με 1� ί � j � ν τέτοιοι, ώστε το άθροισμα Κϊ+ι +Κϊ+2+ • • • + Kj να είναι πολλαπλάσιο του ν. ( Επροτύ.Ηη από l:ωτή ρη Σ ιωτίδα - Καρδ ίτσα) Λύση (από Δημήτρη Καράβοτα- Κ. Αχάία)

Θεωρούμε τα αθροίσματα:

S ι= κ ι S2 =κ ι+ Κ2 s3 =κι+ κ2+κ3

Sv =κι + κ2+ . . . +κv

Α ν για κάποιο από τα αθροίσματα αυτά, έστω το sί, i= 1 ,2, . . . ,ν ισχύει:

S i= κ ι+κ2+ . . . +Κv = O(mod ν) τότε προφανώς οι ζητούμενοι αριθμοί είναι οι Κ ι , κ2, . . . , Ki Αν κανένα από τα αθροίσματα Si δεν είναι πολλα­πλάσιο του ν, τότε επειδή το πλήθος των αθροι­σμάτων είναι ν και το πλήθος των υπολοίπων της διαίρεσης ενός αριθμού που δεν είναι πολλαπλάσιο του ν, είναι ν-1 , θα υπάρχουν τουλάχιστον δυο α­θροίσματα έστω Sκ , Sλ με κ> λ τα οποία διαιρούμε­να με το ν θα δίνουν το ίδιο υπόλοmο. Άρα Sκ - Sλ = O (mod· ν ) , οπότε

ακ+ Ι + ακ+2 + . . . + αλ = O (mod· ν ) Λύσεις έστειλαν επίσης Καλάκος Αθανάσιος- Κάτω Πατήσια, I . Ηλιόπουλος - Καλαμάτα, Ρ. Μπόρης.

Ασκήσεις για λύση 89. Στις πλευρές ΔΓ και ΓΒ τετραγώνου ΑΒΓΔ θεωρούμε τα σημεία Κ, Λ και Μ, Ν αντίστοιχα ώστε να ισχύουν ΔΚ=ΚΛ=ΛΓ=ΓΜ=ΜΝ=ΝΒ. Αν οι ευθείες ΑΚ, ΚΑ τέμνονται από τις ΔΝ, ΔΜ

90. Σε τρίγωνο ΑΒΓ η γωνία Β είναι διπλάσια από τη γωνία Γ. Να βρείετε τη σχέση που συνδέει τις πλευρές του τριγώνου. (Προτείνεται από το συνάδελφο Θανάση Κυριακό­πουλο - Αθήνα)

9 1 . Δίνεται ισοσκελές τρίγωνο ΑΒΓ (ΑΒ=ΑΓ) και σημεία Δ, Ε στις πλευρές ΑΓ, ΑΒ ώστε να ισχύουν ΓΔ=ΔΕ=ΑΕ=ΒΓ. Να υπολογίσετε τη γωνία Α του τριγώνου . (Προτείνεται από το συνάδελφο Γκουντουβά Σωτή­ρη- Αργυρούπολη)

92. Αν για τους θετικούς αριθμούς χ, y, z ισχύει χ+ y+z= 1 , να βρείτε τη μέγιστη τιμή του αθροίσματος S=xy(x+y )+yz(y+z )+zx(z+x) (Προτείνεται από το συνάδελφο Τσαπακίδη Γεώργιο - Αγρίνιο)

93. Οι διχοτόμοι ΑΔ, ΒΕ, ΓΖ των γωνιών τριγώ­νου ΑΒΓ τέμνουν τον περιγεγραμμένο κύκλο στΑ σημεία Α ' , Β ' , Γ . Α ν ΑΑ ' =da. ΒΒ '=dβ και ΓΓ =dy , να αποδείξετε

' 1 1 1 Οτι _α_ + _β_ + __ Υ - > 3

ημ2 Α ημ2Β ημ2Γ -

Ό δ δβ δ

που 1 = -α 1 = - 1 = -γ α d ' β d ' γ d α β γ (Προτείνεται από τον Χρ. Κούρτη - Λάρισα)

94. Ν α βρεθούν οι τρεις μικρότεροι φυσικοί διαδοχικοί αριθμοί των οποίων το άθροισμα είναι τέλειο τετράγωνο και τέλειος κύβος. (Προτείνεται από φοιτητή του ΕΜΠ Μ. Θ. Ρασσιά) 95. Έστω yεΖ* . Αν Χι, x2, . . . ,Xn E Z - { 1 } με η ε Ν και (χι· Χ2· . . . ·xn)2·y:Ξ22(n+ι) και χ ι · χ2 · . . . ·xn ·y=z+ 1 , με zε Ν , να αποδειχθεί ότι τουλάχιστον ένας από τους ακεραίους αριθ­μούς χ ι , χ2 , . . . ,xn, z είναι πρώτος. (Προτείνεται από φοιτητή του ΕΜΠ Μ. Θ. Ρασσιά) 96. Έστω δυο περιττοί πρώτοι ακέραιοι p ι , p2 . Έστω, επίσης, α, η ακέραιοι με α> 1 και η> 1 . ( - 1 )Ρι ( + 1 )Ρι Αν η εξίσωση � + � = αη δεν έχει ακέραιες λύσεις όταν p ι = p2, τότε δεν έχει επίσης ακέραιες λύσεις στην περίπτωση Ρ ι:f Ρ2· (Προτείνεται από φοιτητή του ΕΜΠ Μ. Θ. Ρασσιά)

ΕΥΚΛΕΙΔΗΣ Β ' τ.2/80

Page 83: Ευκλειδης Β 62

,

ι α από τις εκδόσεις <<Εν Δυνάμει>>

Για την Γ. Λυκείου • Φυαική ΙΚcιτ'cύβυνσπς (ταλαντώσεις - Κuματα) - Γ. Θ. Ντοuβαλης

• Φυσική Κατcύβυνσηc: (Μηχανική του ατερεοu σώματος) - Γ. Θ. Ντοuβαλης

• Φuσική ΙΚcιτ'cύβυνσης (Κροuσεις - Φαινόμενο Doppler) - Γ. Θ. Ντοuβαλης • Ασκήσeις Βιολοyiας Γενικής Παι6εiας - Κ. Ρ. Παπαζήσης

• Μα8ημσnκ6 Κατcύ8υνσης (Παρόγωγοι) - Γ. & Π. Λοuκόποuλος

• Μα8πμcmικ6 Κατεύβυνσης (Ολοκληρώματα) - Γ. & Π. Λοuκόποuλος

•Έκφραση -ΊΞκecση - Α. Καλλή - Γ. Σοuλτάνης • Αν6nιυξπ sφαρμοyών σs προyραμμcmιcnικό πeρι86λλον - Κ. Ν. Ιορδανόποuλος

• Αρχ&ς Οικονομικής Θι:ωρiας - Κ. Γαροφαλάκης

Για την Α' Λυκείου • Φυσική - Κ. Ρ. Παπαζήσης

• 'Άλyι:8ρα - Γ. & Π. Λοuκόποuλος

Ειι Δ�ιιόμιsι Ε Κ Δ Ο Σ Ε Ι Σ

ΧΡΥΣΙ Π ΠΟΥ 1 & ΟΥΛΟΦ ΠΑΛΜΕ • ΖΩΓΡΑΦΟΥ Τηλ. : 2 1 Ο 74 88 030, fax : 2 1 Ο 74 83 03 1

Page 84: Ευκλειδης Β 62